Hostname: page-component-76fb5796d-x4r87 Total loading time: 0 Render date: 2024-04-27T09:05:44.544Z Has data issue: false hasContentIssue false

$\mathbf {\Sigma }_1$-definability at higher cardinals: Thin sets, almost disjoint families and long well-orders

Part of: Set theory

Published online by Cambridge University Press:  17 November 2023

Philipp Lücke
Affiliation:
Fachbereich Mathematik, Universität Hamburg, Bundesstraße 55, Hamburg, 20146, Germany; E-mail: philipp.luecke@uni-hamburg.de Departament de Matemàtiques i Informàtica, Universitat de Barcelona, Gran Via de les Corts Catalanes 585, Barcelona, 08007, Spain
Sandra Müller
Affiliation:
Institut für Diskrete Mathematik und Geometrie, Technische Universität Wien, Wiedner Hauptstraße 8-10/104, Wien, 1040, Austria; E-mail: sandra.mueller@tuwien.ac.at

Abstract

Given an uncountable cardinal $\kappa $, we consider the question of whether subsets of the power set of $\kappa $ that are usually constructed with the help of the axiom of choice are definable by $\Sigma _1$-formulas that only use the cardinal $\kappa $ and sets of hereditary cardinality less than $\kappa $ as parameters. For limits of measurable cardinals, we prove a perfect set theorem for sets definable in this way and use it to generalize two classical nondefinability results to higher cardinals. First, we show that a classical result of Mathias on the complexity of maximal almost disjoint families of sets of natural numbers can be generalized to measurable limits of measurables. Second, we prove that for a limit of countably many measurable cardinals, the existence of a simply definable well-ordering of subsets of $\kappa $ of length at least $\kappa ^+$ implies the existence of a projective well-ordering of the reals. In addition, we determine the exact consistency strength of the nonexistence of $\Sigma _1$-definitions of certain objects at singular strong limit cardinals. Finally, we show that both large cardinal assumptions and forcing axioms cause analogs of these statements to hold at the first uncountable cardinal $\omega _1$.

Type
Foundations
Creative Commons
Creative Common License - CCCreative Common License - BY
This is an Open Access article, distributed under the terms of the Creative Commons Attribution licence (https://creativecommons.org/licenses/by/4.0/), which permits unrestricted re-use, distribution, and reproduction in any medium, provided the original work is properly cited.
Copyright
© The Author(s), 2023. Published by Cambridge University Press

1 Introduction

Mathematical objects whose existence are usually proved with the axiom of choice are often referred to as pathological sets. Important examples of such objects are Hamel bases of the vector space of real numbers over the field of rational numbers, nonprincipal ultrafilters on infinite sets and bistationary, that is, stationary and costationary) subsets of uncountable regular cardinals. For many types of pathological sets of real numbers, it is possible to use results from descriptive set theory to show that these objects cannot be defined by simple formulas in second-order arithmetic. Moreover, many canonical extensions of the axioms of ${\mathrm {ZFC}}$ prove that these objects are not definable in second-order arithmetic at all and this implication is often viewed as a desirable feature of such extensions because it allows us to clearly separate pathological sets of real numbers from the explicitly constructed sets of reals.

In this paper, we study the set-theoretic definability of pathological sets of higher cardinalities. More specifically, we aim to generalize classical nondefinability results for sets of real numbers to subsets of the power set ${\mathcal {P}}({\kappa })$ of an uncountable cardinal $\kappa $ that are definable by $\Sigma _1$ -formulasFootnote 1 with parameters in ${\mathrm {H}}(\kappa )\cup \{\kappa \}$ . This bound on the complexity of the used formulas is motivated by the observation that the assumption ${\mathrm {V}}={\mathrm {HOD}}$ implies the $\Sigma _2$ -definability of various pathological sets (see [Reference Lücke34, Proposition 3.9]), and this assumption is compatible with many canonical extensions of ${\mathrm {ZFC}}$ . The restriction of the set of parameters is motivated by the existence of highly potent coding forcings at uncountable regular cardinals $\kappa $ that can be used to make highly pathological subsets of ${\mathcal {P}}({\kappa })$ definable by a $\Sigma _1$ -formula with parameters in ${\mathrm {H}}(\kappa ^+)$ . For example, the results of [Reference Lücke33, Section 3] show that, if $\kappa $ is an uncountable cardinal satisfying $\kappa =\kappa ^{{<}\kappa }$ and A is a subset ${\mathcal {P}}({\kappa })$ , then, in some cofinality-preserving forcing extension ${\mathrm {V}}[G]$ of the ground model ${\mathrm {V}}$ , the sets A and ${\mathcal {P}}({\kappa })^{{\mathrm {V}}[G]}\setminus A$ are definable by $\Sigma _1$ -formulas with parameters in ${\mathrm {H}}(\kappa ^+)$ . Moreover, the main result of [Reference Holy and Lücke22] shows that for every cardinal $\kappa $ with these properties, there is a cofinality-preserving forcing extension in which a well-ordering of ${\mathcal {P}}({\kappa })$ is definable by a $\Sigma _1$ -formula with parameters in ${\mathrm {H}}(\kappa ^+)$ , and the results of [Reference Friedman, Holy and Lücke13] show that various large cardinal properties of $\kappa $ can be preserved by such coding forcings. Finally, results of Caicedo and Veličković in [Reference Caicedo and Veličković5] show that the bounded proper forcing axiom ${\mathrm {BPFA}}$ outright implies the existence of a well-ordering of ${\mathcal {P}}({\omega _1})$ that is definable by a $\Sigma _1$ -formula with parameters in ${\mathrm {H}}(\omega _2)$ (see the proof of Proposition 10.6 below).

Previous work in this direction (see [Reference Lücke34], [Reference Lücke, Schindler and Schlicht37] and [Reference Welch51]) has already provided important examples that show that we can achieve the above aim when we work in one of the following scenarios:

  • The cardinal $\kappa $ is a limit of cardinals possessing certain large cardinal properties, like measurability.

  • The cardinal $\kappa $ is the first uncountable cardinal $\omega _1$ and either certain large cardinals exist above $\kappa $ or strong forcing axioms hold.

In the following, we will derive structural results for simply definable sets that will allow us to prove the nondefinability of several types of pathological sets in the above settings. These implications can again be seen as desirable features of the corresponding axiom systems. Moreover, for most of our results about singular limits of large cardinals, we prove that the used large cardinal assumption is optimal for the corresponding nondefinability statement at singular cardinals.

The starting point of our work is a perfect set theorem for $\Sigma _1$ -definable sets at limits of measurable cardinals. In order to formulate this result, we generalize some basic topological concepts to higher function spaces and power sets. Given a cardinal $\kappa>0$ and an infinite cardinal $\mu $ , we equip the set ${}^\mu \kappa $ of all functions from $\mu $ to $\kappa $ with the topology whose basic open sets consists of all functions that extend a given function ${s}:{\xi }\longrightarrow {\kappa }$ with $\xi <\mu $ . In the same way, we equip the power set ${\mathcal {P}}({\nu })$ of an infinite cardinal $\nu $ with the topology whose basic open sets consists of all subsets of $\nu $ whose intersection with a given ordinal $\eta <\nu $ is equal to a fixed subset of $\eta $ . We then say that an injection ${\iota }:{{}^\mu \kappa }\longrightarrow {{\mathcal {P}}({\nu })}$ is a perfect embedding if it induces a homeomorphism between ${}^\mu \kappa $ and the subspace ${{\mathrm {ran}}(\iota )}$ of ${\mathcal {P}}({\nu })$ . The following result now shows that, analogously to the perfect set property of analytic sets of reals, simply definable thin sets of subsets of limits of measurable cardinals have small cardinality.

Theorem 1.1. Let $\kappa $ be a limit of measurable cardinals, and let D be a subset of ${\mathcal {P}}({\kappa })$ that is definable by a $\Sigma _1$ -formula with parameters in ${\mathrm {H}}(\kappa )\cup \{\kappa \}$ . If D has cardinality greater than $\kappa $ , then there is a perfect embedding ${\iota }:{{}^{{{\mathrm {cof}}(\kappa )}}\kappa }\longrightarrow {{\mathcal {P}}({\kappa })}$ with ${{\mathrm {ran}}(\iota )}\subseteq D$ .

In the case of singular limits of measurable cardinals, we will use core model theory developed in [Reference Koepke31] and, for example, in [Reference Zeman53] to show that the consistency strength of the assumption of this theorem is optimal for its conclusion.

Theorem 1.2. Let $\kappa $ be a singular strong limit cardinal with the property that, for every subset D of ${\mathcal {P}}({\kappa })$ of cardinality greater than $\kappa $ that is definable by a $\Sigma _1$ -formula with parameters in ${\mathrm {H}}(\kappa )\cup \{\kappa \}$ , there is a perfect embedding ${\iota }:{{}^{{{\mathrm {cof}}(\kappa )}}\kappa }\longrightarrow {{\mathcal {P}}({\kappa })}$ with ${{\mathrm {ran}}(\iota )}\subseteq D$ . Then there is an inner model with a sequence of measurable cardinals of length ${{\mathrm {cof}}(\kappa )}$ .

The next type of pathological sets that we will study in this paper are almost disjoint families of large cardinalities. Given an infinite cardinal $\kappa $ , a set A of unbounded subsets of $\kappa $ is an almost disjoint family in ${\mathcal {P}}({\kappa })$ if $x\cap y$ is bounded in $\kappa $ for all distinct $x,y\in A$ . In addition, we say that such a family A is maximal if for every unbounded subset x of $\kappa $ , there exists $y\in A$ with the property that $x\cap y$ is unbounded in $\kappa $ . Motivated by a classical result of Mathias in [Reference Mathias39] that shows that all analytic maximal almost disjoint families in ${\mathcal {P}}({\omega })$ are finite and many additional influential results on maximal almost disjoint families by Mathias, A. Miller, Törnquist, Horowitz and Shelah, Neeman and Norwood, Bakke-Haga, Fischer, Schrittesser, Weinert, and others (see [Reference Bakke Haga, Schrittesser and Törnquist3, Reference Fischer, Schrittesser and Weinert12, Reference Horowitz and Shelah23, Reference Mathias39, Reference Miller40, Reference Neeman and Norwood42, Reference Schrittesser and Törnquist45, Reference Törnquist50]), we will use the techniques developed in the proof of Theorem 1.1 to prove that, if a cardinal $\kappa $ possesses sufficiently strong large cardinal properties, then every simply definable almost disjoint family in ${\mathcal {P}}({\kappa })$ has cardinality at most $\kappa $ . In particular, by a simple diagonalization argument, all simply definable maximal almost disjoint families in ${\mathcal {P}}({\kappa })$ have cardinality less than $\kappa $ in this case.

In order to reduce the large cardinal assumptions used in our arguments, we recall the notion of iterable cardinals, introduced by Sharpe and Welch in [Reference Sharpe and Welch46] and studied extensively in [Reference Gitman and Welch17]. An uncountable cardinal $\kappa $ is iterable if for every subset x of $\kappa $ , there exists a transitive model M of ${\mathrm {ZFC}}^-$ of cardinality $\kappa $ with $\kappa ,x\in M$ and a weakly amenable M-ultrafilter U on $\kappa $ such that the structure $\langle M,U\rangle $ is iterable. Note that all iterable cardinals are weakly compact and all Ramsey cardinals are iterable (see, for example, [Reference Gitman16, Theorem 1.3]). In particular, all measurable limits of measurable cardinals satisfy the assumptions of the following result.

Theorem 1.3. Let $\kappa $ be an iterable cardinal that is a limit of measurable cardinals, and let A be a subset of ${\mathcal {P}}({\kappa })$ that is definable by a $\Sigma _1$ -formula with parameters in ${\mathrm {H}}(\kappa )\cup \{\kappa \}$ . If A has cardinality greater than $\kappa $ , then there exist distinct $x,y\in A$ with the property that $x\cap y$ is unbounded in $\kappa $ .

The third type of pathological sets studied in this paper are long well-orders, that is, well-orderings of subsets of the power set ${\mathcal {P}}({\kappa })$ of an infinite cardinal $\kappa $ of order-type at least $\kappa ^+$ . The study of the definability of these objects is motivated by the classical fact that projective determinacy implies that all well-orderings definable in second-order arithmetic have countable length. In the case of limits of measurable cardinals $\kappa $ , it is possible to use arguments contained in the proof of [Reference Lücke and Schlicht38, Lemma 1.3] to show that for every well-ordering of $\kappa $ , the collection of proper initial segments of the given order is not definable by a $\Sigma _1$ -formula with parameters in ${\mathrm {H}}(\kappa )\cup \{\kappa \}$ . In Section 7 below, we will show that it is possible to use classical results of Dehornoy in [Reference Dehornoy11] to show that for all such limits $\kappa $ , no well-ordering of ${\mathcal {P}}({\kappa })$ is definable in the above way (see Corollary 7.4). We will then proceed by using ideas from the proof of Theorem 1.1 to prove results about well-orderings whose domain is a large proper subset of ${\mathcal {P}}({\kappa })$ . The following theorem provides a scenario in which such orders have no simple definition.

Theorem 1.4. Let $\kappa $ be a cardinal of countable cofinality that is a limit of measurable cardinals. If there exists a well-ordering of a subset of ${\mathcal {P}}({\kappa })$ of cardinality greater than $\kappa $ that is definable by a $\Sigma _1$ -formula with parameter $\kappa $ , then there is a $\mathbf {\Sigma }^1_3$ -well-ordering of the reals.

In addition, the theory developed in this paper allow us to determine the exact consistency strength of the nonexistence of $\Sigma _1$ -definable long well-orderings of subsets of a singular strong limit cardinal of countable cofinality. The following theorem is proven by combining our techniques with results about short core models from [Reference Koepke31] in one direction and diagonal Prikry forcing in the other direction.

Theorem 1.5. The following statements are equiconsistent over ${\mathrm {ZFC}}$ :

  1. (i) There exist infinitely many measurable cardinals.

  2. (ii) There exists a singular cardinal $\kappa $ with the property that no well-ordering of a subset of ${\mathcal {P}}({\kappa })$ of cardinality greater than $\kappa $ is definable by a $\Sigma _1$ -formula with parameters in ${\mathrm {H}}(\kappa )\cup \{\kappa \}$ .

We now continue by considering analogues of the above results for pathological sets consisting of subsets of the first uncountable cardinal $\omega _1$ . Using results of Woodin in [Reference Woodin52], a perfect subset theorem for subsets of ${\mathcal {P}}({\omega _1})$ definable by a $\Sigma _1$ -formula with parameters in ${\mathrm {H}}(\aleph _1)\cup \{\omega _1\}$ is provided by [Reference Lücke, Schindler and Schlicht37, Theorem 4.9] that shows that if the nonstationary ideal on $\omega _1$ is saturated and there is a measurable cardinal, then every such subset either contains a continuous image of ${}^{\omega _1}\omega _1$ or is a subset of ${\mathrm {L}}({\mathbb {R}})$ . As observed in [Reference Lücke, Schindler and Schlicht37], it is, in general, not possible to strengthen the second alternative to state that the given set has cardinality at most $\aleph _1$ , because the failure of ${\mathrm {CH}}$ implies that $\{{x\in {}^{\omega _1}\omega _1}~\vert ~{\forall \alpha <\omega _1 ~ x(\omega +\alpha )=0}\}$ is a subset of ${}^{\omega _1}\omega _1$ of cardinality greater than $\aleph _1$ that is definable by a $\Sigma _1$ -formula with parameter $\omega _1$ and does not contain a perfect subset. The following result now shows that analogs of Theorems 1.3 and 1.4 for $\omega _1$ follow both from strong large cardinal assumptions and the validity of strong forcing axioms.

Theorem 1.6. Assume that either there is a measurable cardinal above infinitely many Woodin cardinals or Woodin’s axiom $(*)$ holds.

  1. (i) No well-ordering of a subset of ${\mathcal {P}}({\omega _1})$ of cardinality greater than $\aleph _1$ is definable by a $\Sigma _1$ -formula with parameters in ${\mathrm {H}}(\aleph _1)\cup \{\omega _1\}$ .

  2. (ii) If A is a set of cardinality greater than $\aleph _1$ that consists of unbounded subsets of $\omega _1$ and is definable by a $\Sigma _1$ -formula with parameters in ${\mathrm {H}}(\aleph _1)\cup \{\omega _1\}$ , then there exist distinct $x,y\in A$ with the property that $x\cap y$ is unbounded in $\omega _1$ .

We will end this paper by observing that the above results cannot be generalized from $\omega _1$ to $\omega _2$ . More specifically, we will show that all large cardinal assumptions are compatible with the existence of an almost disjoint family of cardinality $2^{\aleph _2}$ in ${\mathcal {P}}({\omega _2})$ that is definable by a $\Sigma _1$ -formula with parameter $\omega _2$ (see Proposition 10.6 below).

2 A perfect subset theorem for limits of measurable cardinals

In this section, we prove Theorem 1.1 with the help of iterated ultrapowers. Throughout this paper, we will use two types of iterated ultrapower constructions for transitive ${\mathrm {ZFC}}^-$ -models M: iterated ultrapowers of M constructed using a single weakly amenable M-ultrafilter (as defined in [Reference Kanamori28, Section 19]) and iterated ultrapowers of M constructed using a set of normal measures in M (as defined in [Reference Steel49, Section 3]). In order to establish notation, we now discuss some details of the second type of construction. Given a transitive model M of ${\mathrm {ZFC}}^-$ and $\mathcal {E}\in M$ with

$$ \begin{align*}M\models{\text{"}\hspace{0.3ex}{\mathcal{E}\ \textit{consists of normal ultrafilters on measurable cardinals}}\hspace{0.3ex}\text{"}}, \end{align*} $$

a linear iteration of $\langle M,\mathcal {E}\rangle $ is a sequence $I=\langle {U_\alpha }~\vert ~{\alpha <\lambda }\rangle $ with $\lambda>0$ and the property that there exists a directed system

$$ \begin{align*}\langle\langle{M_\alpha}~\vert~{\alpha<\lambda}\rangle,\langle{{i_{\alpha,\beta}}:{M_\alpha}\longrightarrow{M_\beta}}~\vert~{\alpha\leq\beta<\lambda}\rangle\rangle\end{align*} $$

of transitive ${\mathrm {ZFC}}^-$ -models and elementary embeddings such that the following statements hold:

  1. (i) $M_0=M$ .

  2. (ii) $U_\alpha \in i_{0,\alpha }(\mathcal {E})$ for all $\alpha <\lambda $ .

  3. (iii) If $\alpha $ is an ordinal with $\alpha +1<\lambda $ , then $M_{\alpha +1}$ is the (transitive collapse) of the ultrapower of $M_\alpha $ constructed using $U_\alpha $ and $i_{\alpha ,\alpha +1}$ is the corresponding ultrapower embedding.

  4. (iv) If $\eta <\lambda $ is a limit ordinal, then $\langle M_\eta ,\langle {i_{\alpha ,\eta }}~\vert ~{\alpha <\eta }\rangle \rangle $ is a direct limit of the directed system $\langle \langle {M_\alpha }~\vert ~{\alpha <\eta }\rangle ,\langle {{i_{\alpha ,\beta }}:{M_\alpha }\longrightarrow {M_\beta }}~\vert ~{\alpha \leq \beta <\eta }\rangle \rangle $ .

The ordinal $\lambda $ is then called the length of I, and we use ${\mathrm {lh}}({I})$ to refer to this ordinal.

It is easy to see that the above system is uniquely determined by the sequence I, and therefore we write $U^I_\alpha =U_\alpha $ , $M^I_\alpha =M_\alpha $ and $i^I_{\alpha ,\beta }=i_{\alpha ,\beta }$ for all $\alpha \leq \beta <{\mathrm {lh}}({I})$ . We then let $\langle M^I_\infty ,\langle {{i^I_{\alpha ,\infty }}:{M^I_\alpha }\longrightarrow {M^I_\infty }}~\vert ~{\alpha <{\mathrm {lh}}({I})}\rangle \rangle $ denote the direct limit of the above system and, if the model $M^I_\infty $ is well-founded, then we identify it with its transitive collapse. Finally, the pair $\langle M,\mathcal {E}\rangle $ is called linearly iterable if the model $M^I_\infty $ is well-founded for every linear iteration I of $\langle M,\mathcal {E}\rangle $ . Note that [Reference Steel49, Theorem 3.3] shows that, if every element of $\mathcal {E}$ is $\sigma $ -complete in ${\mathrm {V}}$ , then the pair is $\langle M,\mathcal {E}\rangle $ is linearly iterable. In particular, the pair $\langle {\mathrm {V}},\mathcal {E}\rangle $ is linearly iterable for every set $\mathcal {E}$ of normal ultrafilters.

Given a transitive $ZFC^-$ -model M and $U\in M$ with

$$ \begin{align*}M\models{\text{"}\hspace{0.3ex}{U\ \textit{is a normal ultrafilters on a measurable cardinal}}\hspace{0.3ex}\text{"}}, \end{align*} $$

the set U is a weakly amenable M-ultrafilter and the pair $\langle M,U\rangle $ is iterable (in the sense of [Reference Kanamori28, Section 19]) if and only if the pair $\langle M,\{U\}\rangle $ is linearly iterable (in the above sense). Moreover, if $\langle M,U\rangle $ is iterable,

$$ \begin{align*}\langle\langle{M_\alpha}~\vert~{\alpha\in{\mathrm{On}}}\rangle,\langle{{j_{\alpha,\beta}}:{M_\alpha}\longrightarrow{M_\beta}}~\vert~{\alpha\leq\beta\in{\mathrm{On}}}\rangle\rangle\end{align*} $$

denotes the iteration of $\langle M,U\rangle $ (as defined in [Reference Kanamori28, Section 19]) and $\lambda>0$ is an ordinal, then $\langle {j_{0,\alpha }(U)}~\vert ~{\alpha <\lambda }\rangle $ is the unique linear iteration $I(U,\lambda )$ of $\langle M,\{U\}\rangle $ of length $\lambda $ and we have $M^{I(U,\lambda )}_\alpha =M_\alpha $ and $i^{I(U,\lambda )}_{\alpha ,\beta }=j_{\alpha ,\beta }$ for all $\alpha \leq \beta <\lambda $ .

The following technical lemma about the existence of certain systems of linear iterations is the starting point of the proofs of most of the results about limits of measurable cardinals stated in the introduction:

Lemma 2.1. Let $\mu $ be an infinite regular cardinal, let $\kappa $ be a limit of measurable cardinals with ${{\mathrm {cof}}(\kappa )}=\mu $ and let $\mathcal {E}$ denote the collection of all normal ultrafilters on cardinals smaller than $\kappa $ . Given an element z of ${\mathrm {H}}(\kappa )$ and a subset D of ${\mathcal {P}}({\kappa })$ of cardinality $\kappa ^+$ , there exists

  • an element x of D,

  • a system $\langle {\nu _s}~\vert ~{s\in {}^{{<}\mu }\kappa }\rangle $ of inaccessible cardinals smaller than $\kappa $ ,

  • a system $\langle {\kappa _s}~\vert ~{s\in {}^{{<}\mu }\kappa }\rangle $ of measurable cardinals smaller than $\kappa $ ,

  • a system $\langle {U_s}~\vert ~{s\in {}^{{<}\mu }\kappa }\rangle $ of elements of $\mathcal {E}$ , and

  • a system $\langle {I_s}~\vert ~{s\in {}^{{<}\mu }\kappa }\rangle $ of linear iterations of $\langle {\mathrm {V}},\mathcal {E}\rangle $ of length less than $\kappa $

such that the following statements hold for all $s,t\in {}^{{<}\mu }\kappa $ :

  1. (i) $z\in {\mathrm {H}}(\nu _\emptyset )$ and $\mu <\kappa $ implies that $\mu <\nu _\emptyset $ .

  2. (ii) $U_s$ is an ultrafilter on $\kappa _s$ .

  3. (iii) $I_s$ is a linear iteration of $\langle {\mathrm {V}},\{{U_{s\restriction \xi }}~\vert ~{\xi \in {{\mathrm {dom}}(s)}}\}\rangle $ .

  4. (iv) The sequence $\langle {\min \{{\kappa _s}~\vert ~{s\in {}^\xi \kappa }\}}~\vert ~{\xi <\mu }\rangle $ is cofinal in $\kappa $ .

  5. (v) If $I_s$ is nontrivial, then ${\mathrm {lh}}({I_s})\in {\mathrm {Lim}}$ .

  6. (vi) If $s\subsetneq t$ , then ${\mathrm {lh}}({I_s}){}<\nu _s<\kappa _s<\nu _t$ .

  7. (vii) $i^{I_s}_{0,\infty }(\nu _s)=\nu _s$ and $i^{I_s}_{0,\infty }(\kappa _s)=\kappa _s$ .

  8. (viii) $i^{I_s}_{0,\infty }(\mu )=\mu $ , $i^{I_s}_{0,\infty }(\kappa )=\kappa $ and $i^{I_s}_{0,\infty }(z)=z$ .

  9. (ix) If $s\subseteq t$ , then ${\mathrm {lh}}({I_s}){}\leq {\mathrm {lh}}({I_t}){}$ and $U^{I_s}_\alpha =U^{I_t}_\alpha $ for all $\alpha <{\mathrm {lh}}({I_s}){}$ .Footnote 2

  10. (x) If $s\subseteq t$ with ${\mathrm {lh}}({I_s}){}<{\mathrm {lh}}({I_t}){}$ , then ${\mathrm {H}}(\kappa _s)^{M^{I_s}_\infty }={\mathrm {H}}(\kappa _s)^{M^{I_t}_\infty }$ and

    $$ \begin{align*}i^{I_t}_{{\mathrm{lh}}({I_s}){},\infty}\restriction{\mathrm{H}}(\kappa_s)^{M^{I_s}_\infty} ~ = ~ {\mathrm{id}}_{{\mathrm{H}}(\kappa_s)^{M^{I_s}_\infty}}.\end{align*} $$
  11. (xi) If $\xi \in {{\mathrm {dom}}(s)}\cap {{\mathrm {dom}}(t)}$ satisfying $s\restriction \xi =t\restriction \xi $ and $s(\xi )< t(\xi )$ , then $\nu _{s\restriction (\xi +1)}\leq \nu _{t\restriction (\xi +1)}$ and

    $$ \begin{align*}i^{I_s}_{0,\infty}(x)\cap \nu_{s\restriction(\xi+1)} ~ \neq ~ i^{I_t}_{0,\infty}(x)\cap\nu_{s\restriction(\xi+1)}.\end{align*} $$

Proof. Pick a strictly increasing, cofinal sequence $\langle {\kappa _\xi }~\vert ~{\xi <\mu }\rangle $ of measurable cardinals in $\kappa $ with the property that $\mu <\kappa $ implies that $\mu <\kappa _0$ . Given $\xi <\mu $ , fix a normal ultrafilter $U_\xi $ on $\kappa _\xi $ and let

$$ \begin{align*}\langle\langle{N^\xi_\alpha}~\vert~{\alpha\in{\mathrm{On}}}\rangle,\langle{{j^\xi_{\alpha,\beta}}:{N^\xi_\alpha}\longrightarrow{N^\xi_\beta}}~\vert~{\alpha\leq\beta\in{\mathrm{On}}}\rangle\rangle\end{align*} $$

denote the iteration of $\langle {\mathrm {V}},U_\xi \rangle $ . Given $\xi <\mu $ , we then have $j^\xi _{0,\kappa }(\kappa _\xi )=\kappa $ and $j^\xi _{0,\alpha }(\kappa )=\kappa $ for all $\alpha <\kappa $ . In particular, we know that $\vert {{\mathcal {P}}({\kappa })^{N^\xi _\kappa }}\vert =\kappa $ holds for all $\xi <\mu $ . Therefore, we can find $x\in D$ with $x\notin N^\xi _\kappa $ for all $\xi <\mu $ . Given $\xi <\mu $ , we then have $x\neq j^\xi _{0,\kappa }(x)\cap \kappa $ and hence we know that

(1) $$ \begin{align} x\cap j^\xi_{0,\lambda}(\kappa_\xi) ~ \neq ~ j^\xi_{0,\lambda}(x\cap\kappa_\xi) \end{align} $$

holds for all sufficiently large $\lambda <\kappa $ .

By earlier remarks, the pair $\langle {\mathrm {V}},\mathcal {E}\rangle $ is linearly iterable. In the following, we inductively construct systems with the properties listed above while also ensuring that for every $s\in {}^{{<}\mu }\kappa $ , there exists ${{\mathrm {dom}}(s)}\leq \xi <\mu $ with $\kappa _s=\kappa _\xi $ and $U_s=U_\xi $ . Note that this additional property will directly ensure that (iv) holds in the end.

First, we define $I_\emptyset $ to be the trivial linear iteration of $\langle {\mathrm {V}},\mathcal {E}\rangle $ . Moreover, we pick some inaccessible cardinal $\nu _\emptyset <\kappa $ such that $z\in {\mathrm {H}}(\nu _\emptyset )$ and $\mu <\kappa $ implies $\mu <\nu _\emptyset $ .

Next, assume that $\zeta \in {\mathrm {Lim}}\cap \mu $ and the objects $\nu _t$ , $\kappa _t$ , $U_t$ and $I_t$ are defined for all $t\in {}^{{<}\zeta }\kappa $ . Fix $s\in {}^\zeta \kappa $ , and define $I_s$ to be the unique linear iteration of $\langle {\mathrm {V}},\{{U_{s\restriction \eta }}~\vert ~{\eta <\zeta }\}\rangle $ of length $\sup _{\eta <\zeta }{\mathrm {lh}}({I_{s\restriction \eta }}){}<\kappa $ with the property that $U^{I_s}_\alpha =U^{I_{s\restriction \eta }}_\alpha $ holds for all $\eta <\zeta $ and $\alpha <{\mathrm {lh}}({I_{s\restriction \eta }}){}$ . In addition, define $\nu _s$ to be an inaccessible cardinal smaller than $\kappa $ and bigger than both $\sup _{\eta <\zeta }\kappa _{s\restriction \eta }$ and ${\mathrm {lh}}({I_s}){}$ . This setup ensures that ${\mathrm {lh}}({I_s})>1$ implies that ${\mathrm {lh}}({I_s})\in {\mathrm {Lim}}$ , and therefore we know that (v) holds. Moreover, these definitions directly ensure that the relevant parts of (vi) and (vii) hold in this case. In addition, since ${\mathrm {lh}}({I_s})<\nu _s$ and $I_s$ only makes use of ultrafilter on cardinals contained in the interval $(\nu _\emptyset ,\nu _s)$ , the fact that the cofinality of $\kappa $ is not contained in this interval allows us to conclude that (viii) holds in this case. Next, notice that our construction directly ensures that (ix) holds in this case. Moreover, if $\eta <\zeta $ with ${\mathrm {lh}}({I_{s\restriction \eta }}){}<{\mathrm {lh}}({I_s}){}$ , then the fact that (v) and (x) hold for all $\eta <\rho <\zeta $ ensures that

$$ \begin{align*}{\mathrm{H}}(\kappa_{s\restriction\eta})^{M^{I_{s\restriction\eta}}_\infty} ~ = ~ {\mathrm{H}}(\kappa_{s\restriction\eta})^{M^{I_s}_\infty}\end{align*} $$

and

$$ \begin{align*}i^{I_s}_{{\mathrm{lh}}({I_{s\restriction\eta}}){},\infty}\restriction{\mathrm{H}}(\kappa_{s\restriction\eta})^{M^{I_s}_\infty} ~ = ~ {\mathrm{id}}_{{\mathrm{H}}(\kappa_{s\restriction\eta})^{M^{I_s}_\infty}}.\end{align*} $$

By the definition of $I_s$ , this shows that (x) also holds in this case. Finally, pick $t\in {}^{{<}\mu }\kappa $ with ${{\mathrm {dom}}(t)}\leq \zeta $ and $\xi \in {{\mathrm {dom}}(t)}$ with $s\restriction \xi =t\restriction \xi $ and $s(\xi )\neq t(\xi )$ . Set $\rho =\min (\nu _{s\restriction (\xi +1)},\nu _{t\restriction (\xi +1)})$ . Since we know that $\rho <\min (\kappa _{s\restriction (\xi +1)},\kappa _{t\restriction (\xi +1)})$ , we can use (x) and (xi) to show that

$$ \begin{align*}i^{I_s}_{0,\infty}(x)\cap\rho ~ = ~ i^{I_{s\restriction(\xi+1)}}_{0,\infty}(x)\cap\rho ~ \neq ~ i^{I_{t\restriction(\xi+1)}}_{0,\infty}(x)\cap\rho ~ = ~ i^{I_t}_{0,\infty}(x)\cap\rho.\end{align*} $$

By the properties of $\nu _{s\restriction (\xi +1)}$ ensured by our induction hypothesis, these computations show that (xi) also holds in this case.

Now, assume that $\zeta <\mu $ and the objects $\nu _t$ , $\kappa _u$ , $U_u$ and $I_t$ are defined for all $t,u\in {}^{{<}\mu }\kappa $ with ${{\mathrm {dom}}(t)}\leq \zeta $ and ${{\mathrm {dom}}(u)}<\zeta $ . Fix $s\in {}^\zeta \kappa $ , and pick $\zeta \leq \xi <\mu $ with $\kappa _\xi>\nu _s$ . Set $\kappa _s=\kappa _\xi $ and $U_s=U_\xi $ . By Equation (1), there exists a limit ordinal $\kappa _s<\lambda <\kappa $ with the property that

(2) $$ \begin{align} x\cap j^\xi_{0,\lambda}(\kappa_s) ~ \neq ~ j^\xi_{0,\lambda}(x\cap\kappa_s). \end{align} $$

Let $\langle {\lambda _\beta }~\vert ~{\beta <\kappa }\rangle $ denote the unique continuous sequence of ordinals with $\lambda _0=0$ and $\lambda _{\beta +1}=\lambda _\beta +j^\xi _{0,\lambda _\beta }(\lambda )$ for all $\beta <\lambda $ . Since $\kappa $ is a limit of inaccessible cardinals, we know that $\lambda _\beta <\kappa $ holds for all $\beta <\kappa $ . Given $\beta <\kappa $ , define $I_{s^\frown \langle \beta \rangle }$ to be the unique linear iteration of $\langle {\mathrm {V}},\mathcal {E}\rangle $ of length ${\mathrm {lh}}({I_s})+i^{I_s}_{0,\infty }(\lambda _\beta )$ with $U^{I_{s^\frown \langle \beta \rangle }}_\alpha =U^{I_s}_\alpha $ for all $\alpha <{\mathrm {lh}}({I_s}){}$ and $U^{I_{s^\frown \langle \beta \rangle }}_\alpha =i^{I_{s^\frown \langle \beta \rangle }}_{0,\alpha }(U_s)$ for all ${\mathrm {lh}}({I_s}){}\leq \alpha <{\mathrm {lh}}({I_{s^\frown \langle \beta \rangle }}){}$ . That means we linearly iterate $U_s$ on top of what we already have to obtain $I_{s^\frown \langle \beta \rangle }$ . Moreover, for every $\beta <\kappa $ , we define $\nu _{s^\frown \langle \beta \rangle }$ to be the least inaccessible cardinal greater than ${\mathrm {lh}}({I_{s^\frown \langle \beta +1\rangle }}){}$ . These definitions then directly ensure that (v) and (ix) hold. In addition, for all $\beta <\kappa $ , we have

$$ \begin{align*}{\mathrm{lh}}({I_s}) ~ < ~ \nu_s ~ < ~ \kappa_s ~ < ~ \lambda ~ \leq ~ {\mathrm{lh}}({I_{s^\frown\langle\beta+1\rangle}}) ~ < ~ \nu_{s^\frown\langle\beta\rangle}\end{align*} $$

and this can be used to conclude that $i^{I_s}_{0,\infty }(\kappa _s)=\kappa _s$ and $i^{I_{s^\frown \langle \beta \rangle }}_{0,\infty }(\nu _{s^\frown \langle \beta \rangle })=\nu _{s^\frown \langle \beta \rangle }$ . This shows that the relevant instances of (vi) and (vii) hold in this case. Moreover, the fact that all linear iterations of the form $I_{s^\frown \langle \beta \rangle }$ with $\beta <\kappa $ have length less than $\kappa $ and only make use of ultrafilters on cardinals contained in the interval $[\kappa _0,\kappa )$ directly implies that (viii) holds in this case as $\mu < \kappa _0$ in case $\mu < \kappa $ . Next, notice that, if $0<\beta <\kappa $ and ${\mathrm {lh}}({I_s}){}\leq \alpha <{\mathrm {lh}}({I_{s^\frown \langle \beta \rangle }}){}$ , then our construction ensures that

$$ \begin{align*}{\mathrm{H}}(\kappa_s)^{M^{I_s}_\infty} ~ = ~ {\mathrm{H}}(\kappa_s)^{M^{I_{s^\frown\langle\beta\rangle}}_{{\mathrm{lh}}({I_s}){}}} ~ = ~ {\mathrm{H}}(\kappa_s)^{M^{I_{s^\frown\langle\beta\rangle}}_\alpha}\end{align*} $$

and

$$ \begin{align*}i^{I_{s^\frown\langle\beta\rangle}}_{{\mathrm{lh}}({I_s}){},\alpha}\restriction{\mathrm{H}}(\kappa_s)^{M^{I_s}_\infty} ~ = ~ {\mathrm{id}}_{{\mathrm{H}}(\kappa_s)^{M^{I_s}_\infty}}.\end{align*} $$

This directly implies that (x) holds in this case. Finally, fix $\beta <\gamma <\kappa $ . Then ${\mathrm {lh}}({I_{s^\frown \langle \beta +1\rangle }}){}<{\mathrm {lh}}({I_{s^\frown \langle \gamma +1\rangle }}){}$ , and hence we know that $\nu _{s^\frown \langle \beta \rangle }\leq \nu _{s^\frown \langle \gamma \rangle }$ .

Claim. $i^{I_{s^\frown \langle \beta \rangle }}_{0,\infty }(x)\cap \nu _{s^\frown \langle \beta \rangle } ~ \neq ~ i^{I_{s^\frown \langle \gamma \rangle }}_{0,\infty }(x)\cap \nu _{s\frown \langle \beta \rangle }$ .

Proof of the claim

Let

$$ \begin{align*}\langle\langle{N_\alpha}~\vert~{\alpha\in{\mathrm{On}}}\rangle,\langle{{j_{\alpha_0,\alpha_1}}:{N_{\alpha_0}}\longrightarrow{N_{\alpha_1}}}~\vert~{\alpha_0\leq\alpha_1\in{\mathrm{On}}}\rangle\rangle\end{align*} $$

denote the iteration of $\langle M^{I_s}_\infty ,i^{I_s}_{0,\infty }(U_s)\rangle $ . Given $\delta <\kappa $ and $\alpha <i^{I_s}_{0,\infty }(\lambda _\delta )$ , the definition of $I_{s^\frown \langle \delta \rangle }$ ensures that the following statements hold:

  • $M^{I_{s^\frown \langle \delta \rangle }}_{{\mathrm {lh}}({I_s})+\alpha }=N_\alpha $ and $M^{I_{s^\frown \langle \delta \rangle }}_\infty =N_{i^{I_s}_\infty (\lambda _\delta )}$ .

  • $i^{I_{s^\frown \langle \delta \rangle }}_{0,{\mathrm {lh}}({I_s})+\alpha }=j_{0,\alpha }\circ i^{I_s}_{0,\infty }$ and $i^{I_{s^\frown \langle \delta \rangle }}_{0,\infty }=j_{0,i^{I_s}_{0,\infty }(\lambda _\delta )}\circ i^{I_s}_{0,\infty }$ .

Now, set $M_*=M^{I_{s^\frown \langle \beta \rangle }}_\infty $ , $x_*=i^{I_{s^\frown \langle \beta \rangle }}_{0,\infty }(x)$ , $\kappa _*=i^{I_{s^\frown \langle \beta \rangle }}_{0,\infty }(\kappa _s)$ , $\lambda _*=i^{I_{s^\frown \langle \beta \rangle }}_{0,\infty }(\lambda )$ and $U_*=i^{I_{s^\frown \langle \beta \rangle }}_{0,\infty }(U_s)$ . Note that elementarity ensures that

$$ \begin{align*}i_{0,\infty}^{I_s}(j^\xi_{0,\lambda_\beta}(\lambda)) ~ = ~ j_{0,i^{I_s}_{0,\infty}(\lambda_\beta)}(i^{I_s}_{0,\infty}(\lambda)) ~ = ~ i^{I_{s^\frown\langle\beta\rangle}}_{0,\infty}(\lambda) ~ = ~ \lambda_*\end{align*} $$

and this allows us to conclude that

$$ \begin{align*}i^{I_s}_{0,\infty}(\lambda_\gamma) ~ \geq ~ i^{I_s}_{0,\infty}(\lambda_{\beta+1}) ~ = ~ i^{I_s}_{0,\infty}(\lambda_\beta+j^\xi_{0,\lambda_\beta}(\lambda)) ~ = ~ i^{I_s}_{0,\infty}(\lambda_\beta)+\lambda_*.\end{align*} $$

In particular, we know that

(3) $$ \begin{align} {\mathrm{lh}}({I_{s^\frown\langle\beta\rangle}}){}+\lambda_*\leq{\mathrm{lh}}({I_{s^\frown\langle\gamma\rangle}}){}. \end{align} $$

Now, define

$$ \begin{align*}\langle\langle{N^*_\alpha}~\vert~{\alpha\in{\mathrm{On}}}\rangle,\langle{{j^*_{\alpha_0,\alpha_1}}:{N^*_{\alpha_0}}\longrightarrow{N^*_{\alpha_1}}}~\vert~{\alpha_0\leq\alpha_1\in{\mathrm{On}}}\rangle\rangle\end{align*} $$

to be the iteration of $\langle M_*,U_*\rangle $ . Given ordinals $\alpha _0\leq \alpha _1$ , we then have $N^*_{\alpha _0}=N_{i^{I_s}_\infty (\lambda _\beta )+\alpha _0}$ and $j^*_{\alpha _0,\alpha _1}=j_{i^{I_s}_\infty (\lambda _\beta )+\alpha _0,i^{I_s}_\infty (\lambda _\beta )+\alpha _1}$ . In particular, we can use Equation (3) to find an ordinal $\alpha \geq \lambda _*$ with $M^{I_{s^\frown \langle \gamma \rangle }}_\infty =N^*_\alpha $ and $i^{I_{s^\frown \langle \gamma \rangle }}_{0,\infty }=j^*_{0,\alpha }\circ i^{I_{s^\frown \langle \beta \rangle }}_{0,\infty }$ .

By elementarity, the inequality (2) implies that

$$ \begin{align*}x_*\cap j^*_{0,\lambda_*}(\kappa_*) ~ \neq ~ j^*_{0,\lambda_*}(x_*\cap\kappa_*).\end{align*} $$

Moreover, our setup ensures that

$$ \begin{align*}\nu_{s^\frown\langle\beta\rangle} ~> ~ {\mathrm{lh}}({I_{s^\frown\langle\beta+1\rangle}}){} ~ \geq ~ i^{I_s}_{0,\infty}(\lambda_{\beta+1}) ~ \geq ~ i^{I_s}_{0,\infty}(j^\xi_{0,\lambda_\beta}(\lambda)) ~ = ~ \lambda_* ~ > ~ \kappa_*\end{align*} $$

and

$$ \begin{align*}\nu_{s^\frown\langle\beta\rangle} ~ = ~ j^*_{0,\lambda_*}(\nu_{s^\frown\langle\beta\rangle}) ~> ~ j^*_{0,\lambda_*}(\kappa_*).\end{align*} $$

Since for $\alpha \geq \lambda _*$ as above

$$ \begin{align*}i^{I_{s^\frown\langle\gamma\rangle}}_{0,\infty}(x) ~ = ~ j^*_{0,\alpha}(i^{I_{s^\frown\langle\beta\rangle}}_{0,\infty}(x)) ~ = ~ (j^*_{\lambda_*,\alpha}\circ j^*_{0,\lambda_*})(x_*)\end{align*} $$

and

$$ \begin{align*}j^*_{\lambda_*,\alpha}\restriction(j^*_{0,\lambda_*}(\kappa_*)) ~ = ~ {\mathrm{id}}_{j^*_{0,\lambda_*}(\kappa_*)},\end{align*} $$

we know that

$$ \begin{align*} \begin{aligned} i^{I_{s^\frown\langle\gamma\rangle}}_{0,\infty}(x)\cap j^*_{0,\lambda_*}(\kappa_*) ~ & = ~ j^*_{0,\lambda_*}(x_*)\cap j^*_{0,\lambda_*}(\kappa_*) ~ = ~ j^*_{0,\lambda_*}(x_*\cap\kappa_*) \\ & \neq ~ x_*\cap j^*_{0,\lambda_*}(\kappa_*) ~ = ~ i^{I_{s^\frown\langle\beta\rangle}}_{0,\infty}(x)\cap j^*_{0,\lambda_*}(\kappa_*) \end{aligned} \end{align*} $$

and the fact that $\nu _{s^\frown \langle \beta \rangle }> j^*_{0,\lambda _*}(\kappa _*)$ then yields the statement of the claim.

The above claim now shows that (xi) also holds in this case. This completes the proof of the lemma.

We now extend the above construction to obtain linear iterations indexed by sequences of length equal to the cofinality of the given limit of measurable cardinals. In addition, we also allow these sequences to exist in small forcing extensions of the ground model.

Lemma 2.2. In the situation of Lemma 2.1, let $\lambda \leq \mu $ be a limit ordinal, let ${\mathbb {P}}$ be a partial orderFootnote 3 and let G be ${\mathbb {P}}$ -generic over ${\mathrm {V}}$ . Given a function $c\in ({}^\lambda \kappa )^{{\mathrm {V}}[G]}$ with the property that all of its proper initial segments are contained in ${\mathrm {V}}$ , we let $I_c$ denote the unique linear iteration of $\langle {\mathrm {V}},\{{U_{c\restriction \xi }}~\vert ~{\xi <\lambda }\}\rangle $ of length $\sup _{\xi <\lambda }{\mathrm {lh}}({I_{c\restriction \xi }}){}$ in ${\mathrm {V}}[G]$ with $U^{I_c}_\alpha =U^{I_{c\restriction \xi }}_\alpha $ for all $\xi <\lambda $ and $\alpha <{\mathrm {lh}}({I_{c\restriction \xi }}){}$ .

If either ${\mathbb {P}}$ is an element of ${\mathrm {H}}(\kappa _\emptyset )$ or forcing with ${\mathbb {P}}$ does not add bounded subsets of $\kappa $ , then the following statements hold in ${\mathrm {V}}[G]$ for all functions $c,d\in {}^\lambda \kappa $ with the property that all of their proper initial segments are contained in ${\mathrm {V}}$ :

  1. (i) $M^{I_c}_\infty $ is well-founded.

  2. (ii) $i^{I_c}_{0,\infty }(\mu )=\mu $ , $i^{I_c}_{0,\infty }(\kappa )=\kappa $ and $i^{I_c}_{0,\infty }(z)=z$ .

  3. (iii) If $\xi <\lambda $ with $c\restriction \xi =d\restriction \xi $ and $c(\xi )\neq d(\xi )$ , then

    $$ \begin{align*}i^{I_c}_{0,\infty}(x)\cap\kappa_{c\restriction\xi} ~ = ~ i^{I_d}_{0,\infty}(x)\cap\kappa_{c\restriction\xi}\end{align*} $$
    and there is $\rho <\min (\kappa _{c\restriction (\xi +1)},\kappa _{d\restriction (\xi +1)})$ with
    (4) $$ \begin{align} i^{I_c}_{0,\infty}(x)\cap\rho ~ \neq ~ i^{I_d}_{0,\infty}(x)\cap\rho. \end{align} $$

Proof. Work in ${\mathrm {V}}[G]$ , pick a function $c\in {}^\lambda \kappa $ with the desired properties and define $I_c$ as above. If ${\mathbb {P}}$ is contained in ${\mathrm {H}}(\kappa _\emptyset )$ , then we can use the Lévy–Solovay theorem to show that for all $\xi <\lambda $ , the set $\{{B\in {\mathcal {P}}({\kappa _\xi })}~\vert ~{\exists A\in U_{c\restriction \xi } ~ A\subseteq B}\}$ is a normal ultrafilter on $\kappa _\xi $ , and therefore we know that $U_{c\restriction \xi }$ itself is a $\sigma $ -complete ${\mathrm {V}}$ -ultrafilter. Since the same conclusion obviously holds true if forcing with ${\mathbb {P}}$ does not add bounded subsets of $\kappa $ , we can apply [Reference Steel49, Theorem 3.3] to conclude that the pair $\langle {\mathrm {V}},\{{U_{c\restriction \xi }}~\vert ~{\xi <\lambda }\}\rangle $ is linearly iterable, and therefore we know that $M^{I_c}_\infty $ is well-founded.

Next, since (x) of Lemma 2.1 ensures that

$$ \begin{align*}{\mathrm{H}}(\kappa_{c\restriction\xi})^{M^{I_c}_{{\mathrm{lh}}({I_{c\restriction\xi}}){}}} ~ = ~ {\mathrm{H}}(\kappa_{c\restriction\xi})^{M^{I_{c\restriction\xi}}_\infty} ~ = ~ {\mathrm{H}}(\kappa_{c\restriction\xi})^{M^{I_{c\restriction\zeta}}_\infty} ~ = ~ {\mathrm{H}}(\kappa_{c\restriction\xi})^{M^{I_c}_{{\mathrm{lh}}({I_{c\restriction\zeta}}){}}}\end{align*} $$

and

$$ \begin{align*}i^{I_c}_{{\mathrm{lh}}({I_{c\restriction\xi}}){},{\mathrm{lh}}({I_{c\restriction\zeta}}){}}\restriction{\mathrm{H}}(\kappa_{c\restriction\xi})^{M^{I_{c\restriction\xi}}_\infty} ~ = ~ i^{I_{c\restriction\zeta}}_{{\mathrm{lh}}({I_{c\restriction\xi}}){},\infty}\restriction{\mathrm{H}}(\kappa_{c\restriction\xi})^{M^{I_{c\restriction\xi}}_\infty} ~ = ~ {\mathrm{id}}_{{\mathrm{H}}(\kappa_{c\restriction\xi})^{M^{I_{c\restriction\xi}}_\infty}}\end{align*} $$

hold for all $\xi <\zeta <\lambda $ with ${\mathrm {lh}}({I_{c\restriction \xi }}){}<{\mathrm {lh}}({I_{c\restriction \zeta }}){}<{\mathrm {lh}}({I_c}){}$ , we know that

$$ \begin{align*}{\mathrm{H}}(\kappa_{c\restriction\xi})^{M^{I_{c\restriction\xi}}_\infty} ~ = ~ {\mathrm{H}}(\kappa_{c\restriction\xi})^{M^{I_c}_\infty}\end{align*} $$

and

(5) $$ \begin{align} i^{I_c}_{{\mathrm{lh}}({I_{c\restriction\xi}}){},\infty}\restriction{\mathrm{H}}(\kappa_{c\restriction\xi})^{M^{I_{c\restriction\xi}}_\infty} ~ = ~ {\mathrm{id}}_{{\mathrm{H}}(\kappa_{c\restriction\xi})^{M^{I_{c\restriction\xi}}_\infty}} \end{align} $$

hold for all $\xi <\lambda $ with ${\mathrm {lh}}({I_{c\restriction \xi }}){}<{\mathrm {lh}}({I_c}){}$ . In particular, it follows that $i^{I_c}_{0,\infty }(z)=z$ and, if $\mu <\kappa $ , then $i^{I_c}_{0,\infty }(\mu )=\mu $ . In addition, for all $\xi <\lambda $ with the property that ${\mathrm {lh}}({I_{c\restriction \xi }}){}<{\mathrm {lh}}({I_c}){}$ , we have $i^{I_c}_{0,{\mathrm {lh}}({I_{c\restriction \xi }}){}}=i^{I_{c\restriction \xi }}_{0,\infty }$ and therefore

(6) $$ \begin{align} i^{I_c}_{0,{\mathrm{lh}}({I_{c\restriction\xi}}){}}(\alpha) ~ < ~ i^{I_c}_{0,{\mathrm{lh}}({I_{c\restriction\xi}}){}}(\kappa_{c\restriction\xi}) ~ = ~ i^{I_{c\restriction\xi}}_{0,\infty}(\kappa_{c\restriction\xi}) ~ = ~ \kappa_{c\restriction\xi} \end{align} $$

for all $\alpha <\kappa _{c\restriction \xi }$ . In particular, a combination of Equations (5) and (6) allows us to conclude that $i^{I_c}_{0,\infty }[\kappa _{c\restriction \xi }]\subseteq \kappa _{c\restriction \xi }$ holds for all $\xi <\lambda $ . If the sequence $\langle {\kappa _{c\restriction \xi }}~\vert ~{\xi <\lambda }\rangle $ is cofinal in $\kappa $ , then this observation directly implies that $i^{I_c}_{0,\infty }(\kappa )=\kappa $ . In the other case, if the above sequence is bounded by $\rho <\kappa $ , then $I_c$ is a linear iteration of length less than $\kappa $ that only uses ultrafilters on measurable cardinals in the interval $[\kappa _\emptyset ,\rho ]$ and, since $\kappa $ is a limit of inaccessible cardinals whose cofinality is not contained in this interval, we also know that $i^{I_c}_{0,\infty }(\kappa )=\kappa $ holds in this case.

Finally, pick functions $c,d\in {}^\lambda \kappa $ whose proper initial segments are all contained in ${\mathrm {V}}$ and $\xi <\lambda $ with $c\restriction \xi =d\restriction \xi $ and $c(\xi )\neq d(\xi )$ . Then Equation (5) implies that

$$ \begin{align*}i^{I_c}_{0,\infty}(x)\cap\kappa_{c\restriction\xi} ~ = ~ i^{I_{c\restriction\xi}}_{0,\infty}(x)\cap\kappa_{c\restriction\xi} ~ = ~ i^{I_d}_{0,\infty}(x)\cap\kappa_{c\restriction\xi}.\end{align*} $$

If we now define

$$ \begin{align*}\rho ~ = ~ \min(\nu_{c\restriction(\xi+1)},\nu_{d\restriction(\xi+1)}) ~ < ~ \min(\kappa_{c\restriction(\xi+1)},\kappa_{d\restriction(\xi+1)}),\end{align*} $$

then statement (xi) of Lemma 2.1 directly implies that Equation (4) holds.

We now use the above constructions to derive the desired perfect subset result for $\Sigma _1$ -definable subsets of power sets of limits of measurable cardinals.

Proof of Theorem 1.1

Let $\mu $ be an infinite regular cardinal, let $\kappa $ be a limit of measurable cardinals with ${{\mathrm {cof}}(\kappa )}=\mu $ , let z be an element of ${\mathrm {H}}(\kappa )$ and let D be a subset of ${\mathcal {P}}({\kappa })$ of cardinality greater than $\kappa $ that is definable by a $\Sigma _1$ -formula with parameters $\kappa $ and z. An application of Lemma 2.2 with the trivial partial order now yields $x\in D$ and systems $\langle {\kappa _s}~\vert ~{s\in {}^{{<}\mu }\kappa }\rangle $ , $\langle {U_s}~\vert ~{s\in {}^{{<}\mu }\kappa }\rangle $ and $\langle {I_c}~\vert ~{c\in {}^\mu \kappa }\rangle $ such that the following statements hold for all $s,t\in {}^{{<}\mu }\kappa $ and all $c,d\in {}^\mu \kappa $ :

  • $\kappa _s$ is a measurable cardinal smaller than $\kappa $ .

  • $U_s$ is a normal ultrafilter on $\kappa _s$ .

  • $I_c$ is a linear iteration of $\langle {\mathrm {V}},\{{U_{c\restriction \xi }}~\vert ~{\xi <\mu }\}\rangle $ with $M^{I_c}_\infty $ well-founded.

  • The sequence $\langle {\kappa _{c\restriction \xi }}~\vert ~{\xi <\mu }\rangle $ is cofinal in $\kappa $ .

  • $i^{I_c}_{0,\infty }(\mu )=\mu $ , $i^{I_c}_{0,\infty }(\kappa )=\kappa $ and $i^{I_c}_{0,\infty }(z)=z$ .

  • If $\xi <\mu $ with $c\restriction \xi =d\restriction \xi $ and $c(\xi )\neq d(\xi )$ , then

    $$ \begin{align*}i^{I_c}_{0,\infty}(x)\cap\kappa_{c\restriction\xi} ~ = ~ i^{I_d}_{0,\infty}(x)\cap\kappa_{c\restriction\xi}\end{align*} $$
    and
    $$ \begin{align*}i^{I_c}_{0,\infty}(x)\cap\rho ~ \neq ~ i^{I_d}_{0,\infty}(x)\cap\rho,\end{align*} $$
    where $\rho =\min (\kappa _{c\restriction (\xi +1)},\kappa _{d\restriction (\xi +1)})$ .

We now define

$$ \begin{align*}{\iota}:{{}^\mu\kappa}\longrightarrow{{\mathcal{P}}({\kappa})};~{c}\longmapsto{i^{I_c}_{0,\infty}(x)}.\end{align*} $$

Then $\iota $ is an injection.

Claim. The map $\iota $ is a perfect embedding.

Proof of the claim

Fix $c\in {}^\mu \kappa $ . Given $\alpha <\kappa $ , there is $\xi <\mu $ with $\kappa _{c\restriction \xi }\geq \alpha $ and, if $d\in {}^\mu \kappa $ with $c\restriction \xi =d\restriction \xi $ , then $\iota (c)\cap \kappa _{c\restriction \xi }=\iota (d)\cap \kappa _{c\restriction \xi }$ . In the other direction, fix $\xi <\nu $ and $d\in {}^\mu \kappa $ with $\iota (c)\cap \kappa _{c\restriction \xi }=\iota (d)\cap \kappa _{c\restriction \xi }$ . Assume, towards a contradiction, that $c\restriction \xi \neq d\restriction \xi $ . Then there is $\eta <\xi $ with $c\restriction \eta =d\restriction \eta $ and $c(\eta )\neq d(\eta )$ . Our construction then ensures that $\iota (c)\cap \kappa _{c\restriction (\eta +1)}\neq \iota (d)\cap \kappa _{c\restriction (\eta +1)}$ and therefore $\iota (c)\cap \kappa _{c\restriction \xi }\neq \iota (c)\cap \kappa _{c\restriction \xi }$ , a contradiction. This proves the statement of the claim.

Claim. ${{\mathrm {ran}}(\iota )}\subseteq D$ .

Proof of the claim

Fix $c\in {}^\mu \kappa $ . Pick a $\Sigma _1$ -formula $\varphi (v_0,v_1,v_2)$ such that

$$ \begin{align*}D ~ = ~ \{{y\subseteq\kappa}~\vert~{\varphi(\kappa,y,z)}\}.\end{align*} $$

As $x \in D$ , $\varphi (\kappa ,x,z)$ holds in V and hence the properties listed above ensure that $\varphi (\kappa ,\iota (c),z)$ holds in $M^{I_c}_\infty $ . Since the upwards absoluteness of $\Sigma _1$ -statements directly implies that $\varphi (\kappa ,\iota (c),z)$ holds in ${\mathrm {V}}$ , we can conclude that $\iota (c)$ is an element of D.

This completes the proof of the theorem.

Note that, in general, the conclusion of Theorem 1.1 cannot be extended to $\Sigma _1$ -formulas using arbitrary subsets of the cardinal $\kappa $ as parameters. For example, if $\kappa $ is a regular limit of measurable cardinals, then, in a generic extension by some ${<}\kappa $ -closed forcing, there exists a subset of ${\mathcal {P}}({\kappa })$ that does not contain the range of a perfect embedding and is definable by a $\Sigma _1$ -formula with parameters in ${\mathcal {P}}({\kappa })$ (see [Reference Lücke33, Corollary 7.9]).

3 Local complexity of canonical inner models

As a first application of the results of Section 2, we prove a result that shows that canonical inner models with infinitely many measurable cardinals are not locally $\Sigma _1$ -definable. Note that Gödel’s constructible universe ${\mathrm {L}}$ , the Dodd–Jensen core model and Kunen’s model ${\mathrm {L}}[U]$ all possess the property that for every uncountable cardinal $\kappa $ , the ${\mathrm {H}}(\kappa ^+)^M$ of the corresponding inner model M is definable by a $\Sigma _1$ -formula with parameters in $\kappa +1$ (see, for example, the proof of [Reference Lücke34, Lemma 4.13]). In particular, these models satisfy the assumptions of the next theorem.

Theorem 3.1. Assume that M is a class term with the property that ${\mathrm {ZFC}}$ proves the following statements:Footnote 4

  1. (i) The class M is a transitive model of ${\mathrm {ZFC}}+{{\mathrm {V}}=M}$ that contains all ordinals.

  2. (ii) M is forcing invariant under Cohen forcing ${\mathrm {Add}}({\omega },{1})$ .

  3. (iii) If $\kappa $ is a limit of measurable cardinals with ${{\mathrm {cof}}(\kappa )}=\omega $ and $(\kappa ^+)^M=\kappa ^+$ , then there is a subset of $M\cap {\mathcal {P}}({\kappa })$ of cardinality greater than $\kappa $ that is definable by a $\Sigma _1$ -formula with parameters in ${\mathrm {H}}(\kappa )^M\cup \{\kappa \}$ .

Then ${\mathrm {ZFC}}$ proves that M contains only finitely many measurable cardinals.

Proof. Assume, towards a contradiction, that the above conclusion fails. Then we may work in a model ${\mathrm {V}}$ of ${\mathrm {ZFC}}+{{\mathrm {V}}=M}$ that contains infinitely many measurable cardinals. Let $\kappa $ be the least limit of measurable cardinals, and let G be ${\mathrm {Add}}({\omega },{1})$ -generic over ${\mathrm {V}}$ . An application of (ii) and (iii) in ${\mathrm {V}}[G]$ now yields $z\in {\mathrm {H}}(\kappa )^{\mathrm {V}}$ and a $\Sigma _1$ -formula $\varphi (v_0,v_1,v_2)$ with the property that the set $\{{y\in {\mathrm {V}}[G]}~\vert ~{{\mathrm {V}}[G]\models \varphi (\kappa ,y,z)}\}$ is a subset of ${\mathcal {P}}({\kappa })^{\mathrm {V}}$ that has cardinality greater than $\kappa $ in ${\mathrm {V}}[G]$ . In this situation, the homogeneity of ${\mathrm {Add}}({\omega },{1})$ in ${\mathrm {V}}$ ensures that these statements about the class defined by the formula $\varphi $ and the parameters $\kappa $ and z hold in every ${\mathrm {Add}}({\omega },{1})$ -generic extension of ${\mathrm {V}}$ .

Now, let $G_0\times G_1$ be $({\mathrm {Add}}({\omega },{1})\times {\mathrm {Add}}({\omega },{1}))$ -generic over ${\mathrm {V}}$ . Then the above observations show that

$$ \begin{align*}D ~ = ~ \{{y\in{\mathrm{V}}[G_0]}~\vert~{{\mathrm{V}}[G_0]\models\varphi(\kappa,y,z)}\}\end{align*} $$

is a subset of ${\mathcal {P}}({\kappa })^{\mathrm {V}}$ that has cardinality greater than $\kappa $ in ${\mathrm {V}}[G_0]$ . An application of Lemma 2.1 in ${\mathrm {V}}[G_0]$ then yields $x\in D$ and systems $\langle {\nu _s}~\vert ~{s\in {}^{{<}\omega }\kappa }\rangle $ , $\langle {\kappa _s}~\vert ~{s\in {}^{{<}\omega }\kappa }\rangle $ , $\langle {U_s}~\vert ~{s\in {}^{{<}\omega }\kappa }\rangle $ and $\langle {I_s}~\vert ~{s\in {}^{{<}\omega }\kappa }\rangle $ satisfying the statements listed in the lemma with respect to z and some subset of D of cardinality $\kappa ^+$ . Define

$$ \begin{align*}c ~ = ~ \bigcup G_1 ~ \in ~ ({}^\omega 2)^{V[G_0,G_1]}\setminus{\mathrm{V}}[G_0],\end{align*} $$

and let I denote the unique linear iteration of $\langle {\mathrm {V}}[G_0],\{{U_{c\restriction n}}~\vert ~{n<\omega }\}\rangle $ of length $\sup _{n<\omega }{\mathrm {lh}}({I_{c\restriction n}}){}$ in ${\mathrm {V}}[G_0,G_1]$ with $U^I_\alpha =U^{I_{c\restriction n}}_\alpha $ for all $n<\omega $ and $\alpha <{\mathrm {lh}}({I_{c\restriction n}}){}$ . Then Lemma 2.2 shows that $M^I_\infty $ is well-founded. Set $x_*=i^I_{0,\infty }(x)$ . Since Lemma 2.2 ensures that $i^I_{0,\infty }(\kappa )=\kappa $ and $i^I_{0,\infty }(z)=z$ , we can use the elementarity of $i^I_{0,\infty }$ and $\Sigma _1$ -upwards absoluteness to conclude that $\varphi (\kappa ,x_*,z)$ holds in ${\mathrm {V}}[G_0,G_1]$ . Since ${\mathrm {V}}[G_0,G_1]$ is an ${\mathrm {Add}}({\omega },{1})$ -generic extension of ${\mathrm {V}}$ , our earlier observations allow us to conclude that $x_*$ is an element of ${\mathcal {P}}({\kappa })^{\mathrm {V}}$ . If we now pick $n<\omega $ and set $\rho =\min (\kappa _{(c\restriction n)^\frown \langle 0\rangle },\kappa _{(c\restriction n)^\frown \langle 1\rangle })$ , then clause (iii) of Lemma 2.2 shows that $c(n)$ is the unique $i<2$ with

$$ \begin{align*}x_*\cap\rho ~ = ~ i^{I_{(c\restriction n)^\frown\langle i\rangle}}_{0,\infty}(x)\cap\rho.\end{align*} $$

This conclusion implies that c is an element of ${\mathrm {V}}[G_0]$ , a contradiction.

The above result can easily be shown to be optimal, in the sense that there exists a class term M satisfying the above three properties that can consistently contain any finite number of measurable cardinals. Ideas from the proof of Theorem 1.2 for singular cardinals of countable cofinality will allow us to prove the following result in Section 4.

Proposition 3.2. There exists a class term M with the following properties:

  1. (i) There is a $\Sigma _1$ -formula $\varphi (v_0,v_1,v_2)$ with the property that ${\mathrm {ZFC}}$ proves the following statements:

    1. (a) The class M is a transitive model of ${\mathrm {ZFC}}+{{\mathrm {V}}=M}$ that contains all ordinals.

    2. (b) M is forcing invariant.

    3. (c) If $\kappa $ is an uncountable cardinal, then there exists $z\in {\mathrm {H}}(\kappa )^M$ with ${\mathrm {H}}(\kappa ^+)^M=\{{y}~\vert ~{\varphi (\kappa ,y,z)}\}$ .

  2. (ii) Given a natural number n, if the theory

    $$ \begin{align*}{\mathrm{ZFC}} ~ + ~ {\text{"}\hspace{0.3ex}{\textit{There exist}\ n\ \textit{measurable cardinals}}\hspace{0.3ex}\text{"}} \end{align*} $$
    is consistent, then so is the theory
    $$ \begin{align*}{\mathrm{ZFC}} ~ + ~ {\mathrm{V}}=M ~ + ~ {\text{"}\hspace{0.3ex}{\textit{There exist}\ n\ \textit{measurable cardinals}.}\hspace{0.3ex}\text{"}} \end{align*} $$

4 The lower bound for singular cardinals of countable cofinality

In this section, we will prove the following result that covers the case of singular strong limit cardinals of countable cofinality in the statement of Theorem 1.2:

Theorem 4.1. Assume that there is no inner model with infinitely many measurable cardinals, and let $\kappa $ be a singular strong limit cardinal of countable cofinality. Then there is a subset D of ${\mathcal {P}}({\kappa })$ of cardinality greater than $\kappa $ that is definable by a $\Sigma _1$ -formula with parameters in ${\mathrm {H}}(\kappa )\cup \{\kappa \}$ such that there is no continuous injection ${\iota }:{{}^\omega \kappa }\longrightarrow {{\mathcal {P}}({\kappa })}$ with ${{\mathrm {ran}}(\iota )}\subseteq D$ .

The proof of the above theorem relies on the theory of short core models developed by Koepke in [Reference Koepke31] and generalizations of basic concepts from classical descriptive set theory to simply definable collections of subsets of singular cardinals of countable cofinality. In the following, we will briefly introduce these generalized notions.

Definition 4.2. Let $\kappa $ be a limit cardinal of countable cofinality, and let $0<n<\omega $ be a natural number.

  1. (i) A subset T of $({}^{{<}\omega }\kappa )^n$ is a subtree of $({}^{{<}\omega }\kappa )^n$ if the following statements hold for all ${\langle t_0,\ldots ,t_{n-1}\rangle \in T}$ :

    1. (a) ${\mathrm {lh}}({t_0}) = \ldots = {\mathrm {lh}}({t_{n-1}})$ .

    2. (b) If $m<{\mathrm {lh}}({t_0})$ , then $\langle t_0\restriction m,\ldots ,t_{n-1}\restriction m\rangle ~ \in ~ T$ .

  2. (ii) If T is a subtree of $({}^{{<}\omega }\kappa )^n$ , then we define $[T]$ to be the set of all elements $\langle x_0,\ldots ,x_{n-1}\rangle $ of $({}^\omega \kappa )^n$ with the property that $\langle x_0\restriction m,\ldots ,x_{n-1}\restriction m\rangle \in T$ holds for all $m<\omega $ .

  3. (iii) A subset X of $({}^\omega \kappa )^n$ is a $\mathbf {\Sigma }^1_1$ -subset if there exists a subtree T of $({}^{{<}\omega }\kappa )^{n+1}$ with

    $$ \begin{align*}X ~ = ~ p[T] ~ = ~ \{{\langle x_0,\ldots,x_{n-1}\rangle\in({}^\omega\kappa)^n}~\vert~{\exists y ~ \langle x_0,\ldots,x_{n-1},y\rangle\in[T]}\}.\end{align*} $$
  4. (iv) A subset of $({}^\omega \kappa )^n$ is a $\mathbf {\Pi }^1_1$ -subset if its complement in $({}^\omega \kappa )^n$ is a $\mathbf {\Sigma }^1_1$ -subset.

As in the classical case, we can use universal sets to show that the classes of $\mathbf {\Sigma }^1_1$ - and $\mathbf {\Pi }^1_1$ -subsets do not coincide at singular strong limits of countable cofinality.

Proposition 4.3. If $\kappa $ is a singular strong limit cardinal of countable cofinality, then there exists a $\mathbf {\Sigma }^1_1$ -subset of ${}^\omega \kappa $ that is not a $\mathbf {\Pi }^1_1$ -subset.

Proof. Pick a strictly increasing sequence $\langle {\kappa _m}~\vert ~{m<\omega }\rangle $ of infinite cardinals that is cofinal in $\kappa $ . In addition, fix an enumeration $\langle {a_\alpha }~\vert ~{\alpha <\kappa }\rangle $ of ${\mathrm {H}}(\kappa )$ . Define U to be the set of all pairs $\langle s,t\rangle $ in ${}^{{<}\omega }\kappa \times {}^{{<}\omega }\kappa $ with the property that ${\mathrm {lh}}({s})={\mathrm {lh}}({t})$ and $\langle s\restriction l,t\restriction l\rangle \in a_{s(m)}$ for all $l\leq m<{\mathrm {lh}}({s})$ with $s[l]\cup t[l]\subseteq \kappa _m$ . Then it is easy to see that U is a subtree of ${}^{{<}\omega }\kappa \times {}^{{<}\omega }\kappa $ . Assume, towards a contradiction, that there exists a subtree T of ${}^{{<}\omega }\kappa \times {}^{{<}\omega }\kappa $ with $p[T]={}^\omega \kappa \setminus p[U]$ . Pick a function $x\in {}^\omega \kappa $ with the property that $a_{x(m)}={\mathrm {H}}(\kappa _m)\cap T$ holds for all $m<\omega $ .

Now, assume that there is $y\in {}^\omega \kappa $ with $\langle x,y\rangle \in [T]$ . Then $\langle x,y\rangle \notin [U]$ and there exists $l<\omega $ with $\langle x\restriction l,y\restriction l\rangle \notin U$ . Then there exists $l\leq m<\omega $ with $x[l]\cup y[l]\subseteq \kappa _m$ and $\langle x\restriction l,y\restriction l\rangle \notin a_{x(m)}={\mathrm {H}}(\kappa _m)\cap T$ . But, this yields a contradiction because $\langle x\restriction l,y\restriction l\rangle $ is an element of T. This shows that there is $y\in {}^\omega \kappa $ with $\langle x,y\rangle \in [U]$ . Then $\langle x,y\rangle \notin [T]$ and there is $l<\omega $ with $\langle x\restriction l,y\restriction l\rangle \notin T$ . Pick $l\leq m<\omega $ with $x[l]\cup y[l]\subseteq \kappa _m$ . Then the fact that $\langle x,y\rangle \in [U]$ implies that $\langle x\restriction l,y\restriction l\rangle \in a_{x(m)}\subseteq T$ , a contradiction.

The proof of Theorem 4.1 relies on a generalization of the Boundedness Lemma to singular cardinals of countable cofinality. Below, we introduce the definitions needed in the formulation of this result.

Definition 4.4. Let $\kappa $ be an infinite cardinal, let $\vec {\kappa }=\langle {\kappa _\xi }~\vert ~{\xi <{{\mathrm {cof}}(\kappa )}}\rangle $ be a strictly increasing sequence of ordinals that is cofinal in $\kappa $ and let $\vec {a}=\langle {a_\alpha }~\vert ~{\alpha <\kappa }\rangle $ be a sequence of elements of ${\mathrm {H}}(\kappa )$ .

  1. (i) Given $z\subseteq \kappa $ , we define $\lhd _z$ to be the unique binary relation on $\kappa $ with the property that

    $$ \begin{align*}\alpha\lhd_z\beta ~ \Longleftrightarrow ~ {\prec}{\alpha},{\beta}{\succ}\in z\end{align*} $$
    holds for all $\alpha ,\beta <\kappa $ .Footnote 5
  2. (ii) We define $\mathcal {W}\mathcal {O}_\kappa $ to be the set of all $z\in {\mathcal {P}}({\kappa })$ with the property that $\lhd _z$ is a well-ordering of $\kappa $ .

  3. (iii) We let $\mathsf {WO}(\vec {\kappa },\vec {a})$ denote the set of all $x\in {}^{{{\mathrm {cof}}(\kappa )}}\kappa $ with the property that there exists $y\in \mathcal {W}\mathcal {O}_\kappa $ such that $y\cap \kappa _\xi =a_{x(\xi )}$ holds for all $\xi <{{\mathrm {cof}}(\kappa )}$ .

  4. (iv) Given an element x of $\mathsf {WO}(\vec {\kappa },\vec {a})$ , we let $\|x\|_{\vec {a}}$ denote the order-type of the resulting well-order $\langle \kappa ,\lhd _{\hspace {0.9pt}\bigcup \{{a_{x(\xi )}}~\vert ~{\xi <{{\mathrm {cof}}(\kappa )}}\}}\rangle $ .

Lemma 4.5. Let $\kappa $ be a singular strong limit cardinal of countable cofinality, let $\vec {\kappa }=\langle {\kappa _m}~\vert ~{m<\omega }\rangle $ be a strictly increasing sequence of cardinals that is cofinal in $\kappa $ and let $\vec {a}=\langle {a_\alpha }~\vert ~{\alpha <\kappa }\rangle $ be an enumeration of ${\mathrm {H}}(\kappa )$ . If B is a $\mathbf {\Sigma }^1_1$ -subset of ${}^\omega \kappa $ with $B\subseteq \mathsf {WO}(\vec {\kappa },\vec {a})$ , then there exists an ordinal $\gamma <\kappa ^+$ with $\|y\|_{\vec {a}}<\gamma $ for all $y\in B$ .

Proof. Assume, towards a contradiction, that the set $\{{\|y\|_{\vec {a}}}~\vert ~{y\in B}\}$ is unbounded in $\kappa ^+$ . Pick a subtree S of ${}^{{<}\omega }\kappa \times {}^{{<}\omega }\kappa $ with $p[S]=B$ . By Proposition 4.3, there exists a subtree T of ${}^{{<}\omega }\kappa \times {}^{{<}\omega }\kappa $ with the property that the set $A={}^\omega \kappa \setminus p[T]$ is not a $\mathbf {\Sigma }^1_1$ -subset of ${}^\omega \kappa $ . Given $x\in {}^\omega \kappa $ , set

$$ \begin{align*}T_x ~ = ~ \{{t\in{}^{{<}\omega}\kappa}~\vert~{\langle x\restriction{\mathrm{lh}}({t}),t\rangle\in T}\}.\end{align*} $$

Then $T_x$ is a subtree of ${}^{{<}\omega }\kappa $ for all $x\in {}^\omega \kappa $ and $A=\{{x\in {}^\omega \kappa }~\vert ~{[T_x]=\emptyset }\}$ . By standard arguments (see [Reference Kechris30, Section 2.E]), we now know that a function $x\in {}^\omega \kappa $ is contained in A if and only if there exists an ordinal $\gamma <\kappa ^+$ and a function ${r}:{T_x}\longrightarrow {\gamma }$ with $r(s)>r(t)$ for all $s,t\in T_x$ with $s\subsetneq t$ . Since our assumption implies that for every $\gamma <\kappa ^+$ , there is $y\in B$ with the property that there exists an order-preserving embedding of $\langle \gamma ,<\rangle $ into $\langle \kappa ,\lhd _{\hspace {0.9pt}\bigcup \{{a_{y(\xi )}}~\vert ~{\xi <{{\mathrm {cof}}(\kappa )}}\}}\rangle $ , we know that A consists of all $x\in {}^\omega \kappa $ with the property that there exists $y\in B$ and a function ${f}:{T_x}\longrightarrow {\kappa }$ such that for all $s,t\in T_x$ with $s\subsetneq t$ and all $m<\omega $ with $f(s),f(t)<\kappa _m$ , we have ${\prec }{f(t)},{f(s)}{\succ }\in a_{y(m)}$ .

Below, we aim to derive a contradiction from the above assumption by constructing a subtree U of ${}^{{<}\omega }\kappa \times {}^{{<}\omega }\kappa $ with the property that a pair $\langle x,y\rangle $ in ${}^\omega \kappa \times {}^\omega \kappa $ is an element of $[U]$ if and only if y codes (in some fixed canonical way) functions ${c}:{T_x}\longrightarrow {\omega }$ , ${f}:{T_x}\longrightarrow {\kappa }$ and ${u,v}:{\omega }\longrightarrow {\kappa }$ such that $f(p)<\kappa _{c(p)}$ for all $p\in T_x$ , the pair $\langle u,v\rangle $ is an element of $[S]$ and ${\prec }{f(q)},{f(p)}{\succ }\in a_{u(\max (c(p),c(q)))}$ holds for all $p,q\in T_x$ with $p\subsetneq q$ . Note that if U is a tree with these properties, $\langle x,y\rangle $ is an element of U and c, f, u and v are the functions coded by y, then u is an element of B and the functions f and u witness that x is an element of A. But this means that, if we succeed in constructing such a tree U, then we derive a contradiction because the properties of U ensure that $p[U]=A$ and hence such a tree U witnesses that A is a $\mathbf {\Sigma }^1_1$ -subset of ${}^\omega \kappa $ .

We now show that our assumptions allow us to construct a tree U with the properties described above. For every $s\in {}^{{<}\omega }\kappa $ , we set

$$ \begin{align*}T_s ~ = ~ \{{t\in{}^{{<}\omega}\kappa}~\vert~{{\mathrm{lh}}({t})\leq{\mathrm{lh}}({s}), ~ \langle s\restriction{\mathrm{lh}}({t}),t\rangle\in T}\}.\end{align*} $$

Define U to be the subset of ${}^{{<}\omega }\kappa \times {}^{{<}\omega }\kappa $ consisting of pairs $\langle s,t\rangle $ with ${\mathrm {lh}}({s})={\mathrm {lh}}({t})$ and the property that for all $m<{\mathrm {lh}}({s})$ , there exist $c_m,f_m,u_m,v_m\in {\mathrm {H}}(\kappa )$ such that $a_{t(m)}=\langle c_m,f_m,u_m,v_m\rangle $ and the following statements hold for all $l\leq m$ :

  • $\langle u_l,v_l\rangle ,\langle u_m,v_m\rangle \in S$ , ${\mathrm {lh}}({u_m})=m+1$ , $u_l=u_m\restriction (l+1)$ and $v_l=v_m\restriction (l+1)$ .

  • ${c_m}:{{\mathrm {H}}(\kappa _m)\cap T_{s\restriction m}}\longrightarrow {\omega }$ with $c_m\restriction {{\mathrm {dom}}(c_l)}=c_l$ .

  • ${f_m}:{\{{w\in {\mathrm {H}}(\kappa _m)\cap T_{s\restriction m}}~\vert ~{c_m(w)\leq m}\}}\longrightarrow {\kappa _m}$ with $f_m\restriction {{\mathrm {dom}}(f_l)}=f_l$ and ${\prec }{f_m(q)},{f_m(p)}{\succ }\in a_{u_m(m)}$ for all $p,q\in {{\mathrm {dom}}(f_m)}$ with $p\subsetneq q$ .

Then U is a subtree of ${}^{{<}\omega }\kappa \times {}^{{<}\omega }\kappa $ .

Claim. $p[U] = A$ .

Proof of the Claim

First, fix $\langle x,y\rangle \in [U]$ . Then there are ${c}:{T_x}\longrightarrow {\omega }$ , ${f}:{T_x}\longrightarrow {\kappa }$ and $\langle u,v\rangle \in [S]$ with the property that for all $m<\omega $ , the set $a_{y(m)}$ is equal to the quadruple

$$ \begin{align*}\langle c\restriction({\mathrm{H}}(\kappa_m)\cap T_{x\restriction m}),f\restriction\{{w\in{\mathrm{H}}(\kappa_m)\cap T_{x\restriction m}}~\vert~{c(w)\leq m}\},u\restriction(m+1),v\restriction(m+1)\rangle.\end{align*} $$

Then $u\in B$ and ${\prec }{f(q)},{f(p)}{\succ }\in a_{u(m)}$ holds for all $p,q\in T_x$ with $p\subsetneq q$ and all $m<\omega $ with $f(p),f(q)<\kappa _m$ . By earlier observations, this shows that $x\in A$ .

Now, pick $x\in A$ . Then we can find $\langle u,v\rangle \in S$ and a function ${f}:{T_x}\longrightarrow {\kappa }$ such that for all $p,q\in T_x$ with $p\subsetneq q$ and all $m<\omega $ with $f(p),f(q)<\kappa _m$ , we have ${\prec }{f(q)},{f(p)}{\succ }\in a_{u(m)}$ . Let ${c}:{T_x}\longrightarrow {\omega }$ denote the unique function with $c(p)=\min \{{m<\omega }~\vert ~{f(p)<\kappa _m}\}$ . If we then pick $y\in {}^\omega \kappa $ such that the set $a_{y(m)}$ is equal to the quadruple

$$ \begin{align*}\langle c\restriction({\mathrm{H}}(\kappa_m)\cap T_{x\restriction m}),f\restriction\{{w\in{\mathrm{H}}(\kappa_m)\cap T_{x\restriction m}}~\vert~{c(w)\leq m}\},u\restriction(m+1),v\restriction(m+1)\rangle\end{align*} $$

for all $m<\omega $ , then we can conclude that $\langle x,y\rangle \in [U]$ .

The above computations allow us to conclude that $A=p[U]$ , contradicting the fact that A is not a $\mathbf {\Sigma }^1_1$ -subset of ${}^\omega \kappa $ .

We are now ready to prove the main result of this section.

Proof of Theorem 4.1

Assume that there is no inner model with infinitely many measurable cardinals. Then [Reference Koepke31, Theorem 2.14] implies that $0^{\text {long}}$ (as defined in [Reference Koepke31, Definition 2.13]) does not exist. Let $U_{\text {can}}$ denote the canonical sequence of measures, and let ${\mathrm {K}}[U_{\text {can}}]$ denote the canonical core model (as defined in [Reference Koepke31, Definition 3.15]). Then our assumption implies that ${{\mathrm {dom}}(U_{\text {can}})}$ is finite and [Reference Koepke31, Theorem 3.23] shows that there is a generic extension ${\mathrm {K}}[U_{\text {can}},G]$ of ${\mathrm {K}}[U_{\text {can}}]$ by finitely many Prikry forcings with the property that for every ordinal $\tau \geq \omega _2$ and every $X\subseteq \tau $ such that $\vert {X}\vert $ is a regular cardinal smaller than $\vert {\tau }\vert $ , there exists $Z\in {\mathcal {P}}({\tau })^{{\mathrm {K}}[U_{\text {can}},G]}$ with $X\subseteq Z$ and $\vert {Z}\vert ^{{\mathrm {K}}[U_{\text {can}},G]}<\tau $ .

Now, let $\kappa $ be a singular strong limit cardinal of countable cofinality. Then $\kappa $ is singular in ${\mathrm {K}}[U_{\text {can}},G]$ and $\kappa ^+=(\kappa ^+)^{{\mathrm {K}}[U_{\text {can}},G]}$ . Moreover, since forcing with a finite iteration of Prikry forcings preserves all cardinals, we also know that $\kappa ^+=(\kappa ^+)^{{\mathrm {K}}[U_{\text {can}}]}$ . Set $U={U_{\text {can}}}\restriction {\kappa }$ and ${\mathrm {K}}={\mathrm {K}}[U]$ (see [Reference Koepke31, Definition 3.1]). Then [Reference Koepke31, Theorem 3.2] shows that ${\mathrm {K}}$ is an inner model of ${\mathrm {ZFC}}$ . Moreover, we can use [Reference Koepke31, Theorem 3.9.(iii)] to conclude that ${\mathcal {P}}({\kappa })^{{\mathrm {K}}[U_{\text {can}}]}\subseteq {\mathrm {K}}$ and therefore we know that $\kappa ^+=(\kappa ^+)^{\mathrm {K}}$ .

Next, let $<_{\mathrm {K}}$ denote the canonical well-ordering of ${\mathrm {K}}$ given by [Reference Koepke31, Theorem 3.4]). For every $\kappa \leq \gamma <\kappa ^+$ , let ${b_\gamma }:{\kappa }\longrightarrow {\gamma }$ denote the $<_{\mathrm {K}}$ -least bijection between $\kappa $ and $\gamma $ , and set $y_\gamma =\{{{\prec }{\alpha },{\beta }{\succ }}~\vert ~{\alpha ,\beta <\kappa , ~ b_\gamma (\alpha )<b_\gamma (\beta )}\}$ . Finally, we define $D=\{{y_\gamma }~\vert ~{\kappa \leq \gamma <\kappa ^+}\}$ . Then D is a subset of $\mathcal {W}\mathcal {O}_\kappa $ of cardinality $\kappa ^+$ .

Claim. The set D is definable by a $\Sigma _1$ -formula with parameters in ${\mathrm {H}}(\kappa )\cup \{\kappa \}$ .

Proof of the claim

First, note that our assumption implies that U is an element of ${\mathrm {H}}(\kappa )^{\mathrm {K}}$ . By arguing as in the proof of [Reference Lücke and Schlicht38, Lemma 2.3], we can combine [Reference Koepke31, Theorem 2.7] with [Reference Koepke31, Theorem 2.10] to conclude that the collection of all initial segments of the restriction of $<_{\mathrm {K}}$ to ${\mathrm {H}}(\kappa ^+)^{\mathrm {K}}$ is definable by a $\Sigma _1$ -formula with parameters $\kappa $ and U. This conclusion directly implies the statement of the claim.

In the following, assume, towards a contradiction, that there is a continuous injection ${\iota }:{{}^\omega \kappa }\longrightarrow {{\mathcal {P}}({\kappa })}$ with ${{\mathrm {ran}}(\iota )}\subseteq D$ . Fix a strictly increasing sequence $\vec {\kappa }=\langle {\kappa _m}~\vert ~{m<\omega }\rangle $ of cardinals that is cofinal in $\kappa $ and an enumeration $\vec {a}=\langle {a_\alpha }~\vert ~{\alpha <\kappa }\rangle $ of ${\mathrm {H}}(\kappa )$ . Define T to be the set of all pairs $\langle s,t\rangle $ in ${}^{{<}\omega }\kappa \times {}^{{<}\omega }\kappa $ such that ${\mathrm {lh}}({s})={\mathrm {lh}}({t})$ , and the following statements hold for all $l\leq m<{\mathrm {lh}}({s})$ :

  • $a_{s(m)}\subseteq \kappa _m$ and $a_{s(l)}=a_{s(m)}\cap \kappa _l$ .

  • $a_{t(l)},a_{t(m)}\in {}^{{<}\omega }\kappa $ with $l\leq {\mathrm {lh}}({a_{t(l)}})\leq {\mathrm {lh}}({a_{t(m)}})$ , $a_{t(l)}=a_{t(m)}\restriction {\mathrm {lh}}({a_{t(l)}})$ and $\iota (u)\cap \kappa _m=a_{s(m)}$ for all $u\in {}^\omega \kappa $ with $a_{t(m)}\subseteq u$ .

This definition directly ensures that T is a subtree of ${}^{{<}\omega }\kappa \times {}^{{<}\omega }\kappa $ . Pick $\langle x,y\rangle \in [T]$ . Set $u=\bigcup \{{a_{y(m)}}~\vert ~{m<\omega }\}\in {}^\omega \kappa $ and $v=\bigcup \{{a_{x(m)}}~\vert ~{m<\omega }\}\subseteq \kappa $ . By the definition of T, we then have $\iota (u)=v\in D\subseteq \mathcal {W}\mathcal {O}_\kappa $ and this shows that x is an element of $\mathsf {WO}(\vec {\kappa },\vec {a})$ . This shows that $p[T]\subseteq \mathsf {WO}(\vec {\kappa },\vec {a})$ , and therefore Lemma 4.5 yields an ordinal $\gamma <\kappa ^+$ with $\|x\|_{\vec {a}} < \gamma $ for all $x\in p[T]$ .

Since, for every ordinal $\kappa \leq \delta <\kappa ^+$ , there is a unique element y of D with ${{\mathrm {otp}}\left (\kappa ,\lhd _y\right )}=\delta $ , we know that the map

$$ \begin{align*}{i}:{{}^\omega\kappa}\longrightarrow{\kappa^+};~{u}\longmapsto{{{\mathrm{otp}}\ \left(\kappa,\lhd_{\iota(u)}\right)}}\end{align*} $$

is an injection and we can find $u\in {}^\omega \kappa $ with ${{\mathrm {otp}}\left (\kappa ,\lhd _{\iota (u)}\right )}>\gamma $ . Pick $x\in {}^\omega \kappa $ with $a_{x(m)}=\iota (u)\cap \kappa _m$ for all $m<\omega $ . In addition, pick $y\in {}^\omega \kappa $ with the property that for all $l\leq m<\omega $ , we have $a_{y(l)},a_{y(m)}\in {}^{{<}\omega }\kappa $ , $l\leq {\mathrm {lh}}({a_{y(l)}})\leq {\mathrm {lh}}({a_{y(m)}})$ , $a_{y(l)}\subseteq a_{y(m)}=u\restriction {\mathrm {lh}}({a_{y(m)}})$ and $\iota (w)\cap \kappa _m=\iota (u)\cap \kappa _m$ for all $w\in {}^\omega \kappa $ with $u\restriction {\mathrm {lh}}({a_{y(m)}})\subseteq w$ . Note that this is possible as $\iota $ is a continuous injection. Then $\langle x,y\rangle \in [T]$ and $x\in p[T]\subseteq \mathsf {WO}(\vec {\kappa },\vec {a})$ with $\|x\|_{\vec {a}}={{\mathrm {otp}}\left (\kappa ,\lhd _{\iota (u)}\right )}>\gamma $ , a contradiction.

We close this section by using ideas from the above proof to show that the assumptions of Theorem 3.1 are optimal. These arguments make use of the following observation that can be seen as a special case of the generic absoluteness of the core model as, for example, in [Reference Schimmerling43, Theorem 3.4], [Reference Jensen and Steel26] or [Reference Steel47].

Lemma 4.6. Assume that $0^{\text {long}}$ does not exist. If ${\mathrm {V}}[G]$ is a generic extension of the ground model ${\mathrm {V}}$ , then ${\mathrm {K}}[U_{can}]^{\mathrm {V}}={\mathrm {K}}[U_{can}]^{{\mathrm {V}}[G]}$ .

Proof. The statement of the lemma will be a direct consequence of the following two claims:

Claim. If ${\mathrm {V}}[G]$ is a generic extension of the ground model ${\mathrm {V}}$ , then ${\mathrm {K}}[U_{can}]^{\mathrm {V}}={\mathrm {K}}[U_{can}^{\mathrm {V}}]^{{\mathrm {V}}[G]}$ .

Proof of the claim

Since the property of being a U-mouse is upwards absolute between transitive models of ${\mathrm {ZFC}}$ with the same ordinals, we know that ${\mathrm {K}}[U_{can}]^{\mathrm {V}}\subseteq {\mathrm {K}}[U_{can}^{\mathrm {V}}]^{{\mathrm {V}}[G]}$ . Next, observe that the fact that ${\mathrm {V}}[G]$ is a set forcing extension of ${\mathrm {V}}$ implies that all sufficiently large singular cardinals in ${\mathrm {V}}[G]$ are singular in ${\mathrm {V}}$ . Moreover, an application of [Reference Koepke31, Theorem 3.23] shows that all sufficiently large singular cardinals in ${\mathrm {V}}$ are singular in ${\mathrm {K}}[U_{can}]^{\mathrm {V}}$ . In combination, this shows that for all sufficiently large singular cardinals $\lambda $ of uncountable cofinality in ${\mathrm {V}}[G]$ , every closed unbounded subset of $\lambda $ in ${\mathrm {V}}[G]$ contains an element that is singular in ${\mathrm {K}}[U_{can}]^{\mathrm {V}}$ . This observation allows us to use [Reference Koepke31, Theorem 3.24(ii)] to conclude that ${\mathrm {K}}[U_{can}^{\mathrm {V}}]^{{\mathrm {V}}[G]}\subseteq {\mathrm {K}}[U_{can}]^{\mathrm {V}}$ .

Claim. Let ${\mathbb {P}}$ be a weakly homogeneous partial order. If G is ${\mathbb {P}}$ -generic over ${\mathrm {V}}$ , then ${\mathrm {K}}[U_{can}]^{\mathrm {V}}={\mathrm {K}}[U_{can}]^{{\mathrm {V}}[G]}$ .

Proof of the claim

First, the weak homogeneity of ${\mathbb {P}}$ in ${\mathrm {V}}$ ensures that

$$ \begin{align*}U_{can}^{{\mathrm{V}}[G]} ~ \subseteq ~ {\mathrm{K}}[U_{can}]^{{\mathrm{V}}[G]} ~ \subseteq ~ {\mathrm{HOD}}^{{\mathrm{V}}[G]} ~ \subseteq ~ {\mathrm{V}}.\end{align*} $$

In particular, we know that the set $U_{can}^{{\mathrm {V}}[G]}(\kappa )\cap {\mathcal {P}}({\kappa })^{\mathrm {V}}$ is an element of ${\mathrm {V}}$ for every $\kappa \in {{\mathrm {dom}}(U_{can}^{{\mathrm {V}}[G]})}$ . In this situation, we can now use the first claim to inductively show that the definition of the canonical measure sequence ensures that $U_{\text {can}}^{\mathrm {V}}\restriction \xi =U_{can}^{{\mathrm {V}}[G]}\restriction \xi $ holds for all $\xi \in {\mathrm {On}}$ .

Now, let ${\mathbb {P}}$ be a partial order, and let G be ${\mathbb {P}}$ -generic over ${\mathrm {V}}$ . Pick a sufficiently large cardinal $\delta $ such that ${\mathbb {P}}\times {\mathrm {Col}}({\omega },{\delta })$ densely embeds into ${\mathrm {Col}}({\omega },{\delta })$ , and let H be ${\mathrm {Col}}({\omega },{\delta })$ -generic over ${\mathrm {V}}[G]$ . Since ${\mathrm {Col}}({\omega },{\delta })$ is weakly homogeneous in both ${\mathrm {V}}$ and ${\mathrm {V}}[G]$ , we can now use the above claim twice to conclude that ${\mathrm {K}}[U_{can}]^{\mathrm {V}}={\mathrm {K}}[U_{can}]^{{\mathrm {V}}[G,H]}={\mathrm {K}}[U_{can}]^{{\mathrm {V}}[G]}$ .

Proof of Proposition 3.2

Let M denote the class term with the property that ${\mathrm {ZFC}}$ proves the following statements:

  • If either $0^{\text {long}}$ exists or $0^{\text {long}}$ does not exist and the model ${\mathrm {K}}[U_{can}]$ contains infinitely many measurable cardinals, then M is equal to the constructible universe ${\mathrm {L}}$ .

  • Otherwise, M is equal to ${\mathrm {K}}[U_{can}]$ .

Then the standard results about ${\mathrm {L}}$ together with [Reference Koepke31, Theorem 3.2] show that ${\mathrm {ZFC}}$ proves that M is a transitive model of ${\mathrm {ZFC}}+{{\mathrm {V}}=M}$ that contains all ordinals. Moreover, Lemma 4.6 together with the fact that $0^{\text {long}}$ cannot be added by forcing show that M is forcing invariant.

Claim. Assume that $0^{\text {long}}$ does not exist and ${\mathrm {K}}[U_{can}]$ contains only finitely many measurable cardinals. If $\kappa $ is an uncountable cardinal, then ${\mathrm {H}}(\kappa ^+)^{{\mathrm {K}}[U_{can}]}$ is definable by a $\Sigma _1$ -formula with parameters in ${\mathrm {H}}(\kappa )\cup \{\kappa \}$ .

Proof of the claim

Set $U=U_{can}\restriction \kappa $ and ${\mathrm {K}}={\mathrm {K}}[U]$ . Then [Reference Koepke31, Theorem 3.9] shows that ${\mathrm {H}}(\kappa ^+)^{{\mathrm {K}}[U_{can}]}={\mathrm {H}}(\kappa ^+)^{\mathrm {K}}$ . Moreover, if $\kappa $ is not the successor of an element of ${{\mathrm {dom}}(U)}$ in ${\mathrm {K}}$ , then U is an element of ${\mathrm {H}}(\kappa )$ and we can repeat arguments from the proof of Theorem 4.1 to show that the class of all U-mice M (see [Reference Koepke31, Definition 2.9]) that contain $\kappa $ in their lower part $lp(M)$ (see [Reference Koepke31, Definition 2.1]) is definable by a $\Sigma _1$ -formula with parameters $\kappa $ and U. Since every element of ${\mathrm {H}}(\kappa ^+)^{\mathrm {K}}$ is contained in such a lower part, the statement of the claim follows in this case.

In the following, assume that there is $\delta \in {{\mathrm {dom}}(U)}$ with $\kappa =(\delta ^+)^{\mathrm {K}}$ . Let F be a simple predicate with ${{\mathrm {dom}}(F)}={{\mathrm {dom}}(U)}$ , and let M be an F-mouse such that $\kappa ,F\in lp(M)$ , $\kappa =(\delta ^+)^M$ and $F(\mu )$ is an ultrafilter in M for every $\mu \in {{\mathrm {dom}}(F)}$ . Since every subset of $\delta $ in ${\mathrm {K}}[F]$ is contained in an F-mouse of cardinality less than $\kappa $ , we can now apply [Reference Koepke31, Theorem 2.10] to conclude that $F(\mu )$ is an ultrafilter in ${\mathrm {K}}[F]$ for every $\mu \in {{\mathrm {dom}}(F)}$ . This shows that ${\mathrm {K}}[F]$ is a core model (in the sense of [Reference Koepke31, Definition 3.6]) and therefore [Reference Koepke31, Theorem 3.14] shows that ${\mathrm {K}}={\mathrm {K}}[F]$ holds. Since every element of ${\mathrm {H}}(\kappa ^+)^{\mathrm {K}}$ is contained in the lower part of a U-mouse M with $\kappa ,U\in lp(M)$ and $\kappa =(\delta ^+)^M$ , we now know that ${\mathrm {H}}(\kappa ^+)^{\mathrm {K}}$ consists of all sets x with the property that there exists a simple predicate F and an F-mouse M such that ${{\mathrm {dom}}(F)}={{\mathrm {dom}}(U)}$ , $F(\mu )$ is an ultrafilter in M for every $\mu \in {{\mathrm {dom}}(F)}$ , $\kappa =(\delta ^+)^M$ and $\kappa ,F,x\in lp(M)$ . This allows us to conclude that the set ${\mathrm {H}}(\kappa ^+)^{\mathrm {K}}$ is definable by a $\Sigma _1$ -formula with parameters $\delta $ , $\kappa $ and ${{\mathrm {dom}}(U)}$ in this case.

The above claim now allows us to find a $\Sigma _1$ -formula $\varphi (v_0,v_1,v_2)$ with the property that for every uncountable cardinal $\kappa $ , we have ${\mathrm {L}}_{\kappa ^+}=\{{x}~\vert ~{\varphi (\kappa ,\kappa ,x)}\}$ and, if $0^{\text {long}}$ does not exist and ${\mathrm {K}}[U_{can}]$ contains only finitely many measurable cardinals, then there exists $z\in {\mathrm {H}}(\kappa )$ with ${\mathrm {H}}(\kappa ^+)^{{\mathrm {K}}[U_{can}]}=\{{x}~\vert ~{\varphi (\kappa ,x,z)}\}$ . Finally, if the existence of n measurable cardinals is consistent with the axioms of ${\mathrm {ZFC}}$ for some natural number n, then the existence of exactly n measurable cardinals in ${\mathrm {K}}[U_{can}]$ is consistent with ${\mathrm {ZFC}}$ .

5 The lower bound for singular cardinals of uncountable cofinality

We now use ideas from [Reference Hayut and Müller21] to complete the proof of Theorem 1.2.

Proof of Theorem 1.2

Let $\kappa $ be a singular strong limit cardinal with the property that for every subset D of ${\mathcal {P}}({\kappa })$ of cardinality greater than $\kappa $ that is definable by a $\Sigma _1$ -formula with parameters in ${\mathrm {H}}(\kappa )\cup \{\kappa \}$ , there exists a perfect embedding ${\iota }:{{}^{{{\mathrm {cof}}(\kappa )}}\kappa }\longrightarrow {{\mathcal {P}}({\kappa })}$ with ${{\mathrm {ran}}(\iota )}\subseteq D$ . Assume, towards a contradiction, that there is no inner model with a sequence of measurable cardinals of length ${{\mathrm {cof}}(\kappa )}$ . Then Theorem 4.1 implies that the cofinality of $\kappa $ is uncountable. Moreover, we know that there is no inner model with a measurable cardinal of Mitchell order $1$ and therefore we can construct the canonical core model ${\mathrm {K}}$ as in [Reference Zeman53] (which is Steel’s core model [Reference Steel47] in this easier setting). Note that our hypothesis implies that, in ${\mathrm {K}}$ , the sequence of measurable cardinals below $\kappa $ is bounded below $\kappa $ . In addition, as $\kappa $ is singular in ${\mathrm {V}}$ , weak covering (see [Reference Zeman53, Theorem 7.5.1]) holds for ${\mathrm {K}}$ at $\kappa $ , that is, we have $ (\kappa ^+)^{\mathrm {K}} = \kappa ^+$ . Finally, we know that $\kappa $ is singular in ${\mathrm {K}}$ , because otherwise the fact that $\kappa $ is a singular cardinal of uncountable cofinality would allow us to apply the second part of [Reference Cox10, Theorem 1] to find an inner model in which $\kappa $ has Mitchell order greater than $0$ .

We will now construct a tree of height ${{\mathrm {cof}}(\kappa )}^{\mathrm {K}}$ that is an element of ${\mathrm {K}}$ and then argue that this tree does not have a perfect subtree in ${\mathrm {V}}$ . These arguments use ideas from [Reference Hayut and Müller21] that ultimately go back to Solovay’s argument for the consistency strength of the Kurepa hypothesis (see [Reference Jech24, Section 4]). Our tree consists of hulls of initial segments of ${\mathrm {K}}$ of size $\kappa $ , and we will argue that we can obtain such initial segments in a $\Sigma _1$ -definable way with parameters in ${\mathrm {H}}(\kappa ) \cup \{ \kappa \}$ .

In the following, we say a premouse N (in the sense of [Reference Zeman53, Section 4.1]) is good if the following statements hold:

  • N is iterable (in the sense of [Reference Zeman53, Section 4.2]).

  • $\kappa +1 \subseteq N$ and $\vert {N}\vert = \kappa $ .

  • ${{\mathrm {cof}}(\kappa )}^N={{\mathrm {cof}}(\kappa )}^{\mathrm {K}}$ .

  • $\kappa $ is the largest cardinal in N.

  • If $\gamma < \kappa $ is the supremum of the measurable cardinals below $\kappa $ in ${\mathrm {K}}$ , then

    $$\begin{align*}N \vert \gamma^{++} ~ = ~ {\mathrm{K}} \vert \gamma^{++}. \end{align*}$$
    In particular, K and N have the same measurable cardinals and the same total measures below $\kappa $ .

Claim. Let N be a good premouse. Then $N \lhd {\mathrm {K}}$ .

Proof of the Claim

Compare N and ${\mathrm {K}}$ , and suppose, towards a contradiction, that the comparison is not trivial. Consider the first measure that is used. As $N \vert \gamma ^{++} = {\mathrm {K}} \vert \gamma ^{++},$ where $\gamma < \kappa $ is the supremum of the measurable cardinals below $\kappa $ in N and ${\mathrm {K}}$ , the first measure that is used in the comparison has to be a partial measure above $\gamma $ . Say this is a partial measure $\mu $ with critical point $\nu $ on the ${\mathrm {K}}$ -side of the comparison. Then, in order to use this partial measure, we need to truncate ${\mathrm {K}}$ as $\mu $ does not measure all subsets of $\nu $ in ${\mathrm {K}}$ . By the comparison lemma (see, for example, [Reference Zeman53, Lemma 4.4.2] or [Reference Steel48, Theorem 3.11]), we obtain iterates $N^*$ of N and ${\mathrm {K}}^*$ of ${\mathrm {K}}$ (or, in fact, of a truncation ${\mathrm {K}}\vert \xi $ of ${\mathrm {K}}$ ) such that $N^* \unlhd {{\mathrm {K}}^*}$ . Note that truncations can only appear on one side of the comparison, and this side has to come out longer in the end. In particular, the iteration from N to $N^*$ can only use total measures with critical point above $\nu> \gamma $ and is therefore trivial, that is, we have $N = N^*$ .

Suppose that $\nu> \kappa $ . Note that $\nu $ is a cardinal in ${\mathrm {K}}^*$ . As $\nu < N \cap {\mathrm {On}}$ and $N \unlhd {\mathrm {K}}^*$ , this implies that there are cardinals above $\kappa $ in N, contradicting the assumption that $\kappa $ is the largest cardinal in N.

Now, suppose that $\nu < \kappa $ . The iteration from ${\mathrm {K}}|\xi $ to ${\mathrm {K}}^*$ cannot leave any total measures below $\kappa $ behind as $N \unlhd {\mathrm {K}}^*$ does not have any total measures between $\gamma $ and $\kappa $ . As we suppose that there is no inner model with a sequence of measurable cardinals of length ${{\mathrm {cof}}(\kappa )}$ , this implies that the iteration from ${\mathrm {K}}|\xi $ to ${\mathrm {K}}^*$ is a linear iteration of $\mu $ and its images. Again, as $N \unlhd {\mathrm {K}}^*$ does not have any total measures between $\gamma $ and $\kappa $ and $N \cap {\mathrm {On}} \geq \kappa $ , this iteration needs to last at least $\kappa $ -many steps by [Reference Kanamori28, Corollary 19.7.(b)] since $\kappa $ is a cardinal in ${\mathrm {V}}$ . Moreover, [Reference Kanamori28, Corollary 19.7.(b)] shows that $\kappa $ is inaccessible in ${\mathrm {K}}^*$ . As $N \unlhd {\mathrm {K}}^*$ , this contradicts the fact that $\kappa $ is singular in N. Therefore, $\mu $ is not used on the ${\mathrm {K}}$ -side of the comparison.

Similarly, we can argue that no partial measure on N gets used in the comparison and hence we can conclude that $N \lhd {\mathrm {K}}$ .

Claim. For every $x \in \left ({}^{{{\mathrm {cof}}(\kappa )}} \kappa \right )^{\mathrm {K}}$ , there is a good premouse N with $x \in N$ .

Proof of the Claim

As ${\mathrm {K}}$ satisfies the ${\mathrm {GCH}}$ , there is some $\xi < (\kappa ^+)^{\mathrm {K}}=\kappa ^+$ such that $x \in {\mathrm {K}}|\xi $ and ${\mathrm {K}}|\xi $ is a good premouse.

Following [Reference Hayut and Müller21], we say a pair $\langle M, \bar {x}\rangle $ is an active node at $\rho $ for some $\rho < {{\mathrm {cof}}(\kappa )}^{\mathrm {K}}$ if there is a good premouse N and some $x \in \left ({}^{{{\mathrm {cof}}(\kappa )}} \kappa \right )^N$ with ${{\mathrm {ran}}(x)} \subseteq \mathrm {Reg}^N$ , the regular cardinals in N, such that the following statements hold:

  • x is strictly increasing and cofinal in $\kappa $ .

  • M is equal to the transitive collapse of ${{\mathrm {Hull}}}^{N}(x(\rho ) \cup \{x\})$ and $\bar {x} \in M$ is the image of x under the transitive collapse.

  • If ${\pi }:{M}\longrightarrow {N}$ is the corresponding uncollapsing map, then ${{\mathrm {crit}}\left ({\pi }\right )} = \bar {x}(\rho )$ .

In addition, we say a pair $\langle M, \bar {x}\rangle $ is an active node if there is some ordinal $\rho <{{\mathrm {cof}}(\kappa )}^{\mathrm {K}}$ such that $\langle M, \bar {x}\rangle $ is an active node at $\rho $ .

We now let T denote the unique partial order defined by the following clauses:

  1. (i) The elements of T are triples of the form $\langle M, \bar {x}, s\rangle $ satisfying the following properties:

    1. (a) The pair $\langle M, \bar {x}\rangle $ is either an active node or equal to the pair $\langle \emptyset , \emptyset \rangle $ .

    2. (b) s is an element of $({}^{{<}{{\mathrm {cof}}(\kappa )}}\kappa )^{\mathrm {K}}$ with the property that the set

      $$ \begin{align*}\bigcup_{0 < \alpha < \kappa} s^{{-}1}(\{\alpha\})\end{align*} $$
      is finite.
    3. (c) If $\langle M, \bar {x}\rangle $ is an active node at $\rho $ , then ${{\mathrm {dom}}(s)} \geq \rho $ .

  2. (ii) The order of T is defined by

    $$ \begin{align*}\langle M_0, x_0, s_0\rangle ~ \leq_{T} ~ \langle M_1, x_1, s_1\rangle\end{align*} $$
    if and only if the following statements hold:
    1. (a) $M_0$ is the transitive collapse of ${{\mathrm {Hull}}}^{M_1}(x_1(\rho ) \cup \{x_1\})$ for some ordinal $\rho $ and $x_0$ is the image of $x_1$ under the transitive collapse, or $M_0 = x_0 = \emptyset $ . In the following, write $\rho $ for the minimal such ordinal and $\rho = -1$ if $M_0 = x_0 = \emptyset $ .

    2. (b) $s_0$ is an initial segment of $s_1$ .

    3. (c) There is no ordinal $\rho '$ between $\rho $ and ${{\mathrm {dom}}(s_0)}$ with the property that $\langle {{\mathrm {Hull}}}^{M_1}(x_1(\rho ') \cup \{x_1\}), x_1\rangle $ transitively collapses to an active node which, in case $\rho \neq -1$ , is not $\langle M_0, x_0\rangle $ .

It is now easy to see that T is a tree of height ${{\mathrm {cof}}(\kappa )}^{\mathrm {K}}$ that is contained in ${\mathrm {K}}$ and has the property that each node is splitting into $\kappa $ -many successors. Moreover, each $x \in \left ({}^{{{\mathrm {cof}}(\kappa )}} \kappa \right )^{\mathrm {K}}$ that is strictly increasing and cofinal in $\kappa $ with range contained in $\mathrm {Reg}^N$ naturally gives rise to a cofinal branch $b_x$ through T and two different such elements $x,y \in \left ({}^{{{\mathrm {cof}}(\kappa )}} \kappa \right )^{\mathrm {K}}$ give rise to different branches $b_x$ and $b_y$ . Hence, the fact that the ${\mathrm {GCH}}$ holds in ${\mathrm {K}}$ implies that the set of cofinal branches through T has cardinality at least

$$ \begin{align*}\left(\kappa^{{{\mathrm{cof}}(\kappa)}}\right)^{\mathrm{K}} ~ = ~ (\kappa^+)^{\mathrm{K}} ~ = ~ \kappa^+.\end{align*} $$

Claim. Let b be a cofinal branch through T, and let $\langle \mathcal {R}_b, x_b \rangle $ denote the direct limit of models along b. Then $\mathcal {R}_b$ is well-founded and we can identify it with its transitive collapse. Moreover, $\mathcal {R}_b \lhd K$ .

Proof of the claim

As the proof of the well-foundedness of $\mathcal {R}_b$ is easier, we focus on the argument that $\mathcal {R}_b \lhd K$ . By our first claim, it suffices to show that $\mathcal {R}_b$ is a good premouse. We obtain iterability for $\mathcal {R}_b$ by reflecting countable elementary substructures of $\mathcal {R}_b$ into models in the tree T, as in [Reference Hayut and Müller21], using the fact that

$$ \begin{align*}{{\mathrm{cof}}({{\mathrm{cof}}(\kappa)}^{\mathrm{K}})} ~ = ~ {{\mathrm{cof}}(\kappa)} ~> ~ \omega\end{align*} $$

(see [Reference Jech25, Lemma 3.7(ii)]). In the following, write $\langle {M_\rho }~\vert ~{\rho < {{\mathrm {cof}}(\kappa )}^{\mathrm {K}}}\rangle $ for the sequence of models appearing in active nodes at $\rho $ along the branch b. Then the definition of T ensures that for every $\xi < \kappa $ , there is some $\rho < {{\mathrm {cof}}(\kappa )}^{\mathrm {K}}$ such that if $M_\rho $ is the transitive collapse of ${{\mathrm {Hull}}}^{N_\rho }(x_\rho (\rho ) \cup \{x_\rho \})$ for some good premouse $N_\rho $ and some $x_\rho \in \left ({}^{{{\mathrm {cof}}(\kappa )}} \kappa \right )^{N_\rho }$ , then $x_\rho (\rho )> \xi $ . Therefore, we know that $\kappa \subseteq \mathcal {R}_b$ and elementarity implies that $\kappa +1 \subseteq \mathcal {R}_b$ . Our setup also directly ensures that $\vert {\mathcal {R}_b}\vert = \kappa $ . As $N_\rho \lhd K$ and the critical point of the inverse of the collapse embedding $\pi _\rho \colon M_\rho \rightarrow N_\rho $ is at least $x_\rho (\rho )$ , this also shows that $N_\rho | \gamma ^{++} = K | \gamma ^{++}$ , where $\gamma < \kappa $ is the supremum of the measurable cardinals below $\kappa $ in ${\mathrm {K}}$ and $N_\rho $ . Moreover, we know that $\kappa $ has cofinality ${{\mathrm {cof}}(\kappa )}^{\mathrm {K}}$ in $\mathcal {R}_b$ , as witnessed by $x_b$ . Finally, $\kappa $ is the largest cardinal in $\mathcal {R}_b$ by elementarity as $\kappa = \sup ({{\mathrm {ran}}(x_\rho )})$ is the largest cardinal in $N_\rho $ for all $\rho < {{\mathrm {cof}}(\kappa )}$ .

Claim. The set T is $\Sigma _1$ -definable with parameters in ${\mathrm {H}}(\kappa ) \cup \{\kappa \}$ .

Proof of the claim

It clearly suffices to show that the set of all good mice N is definable in the above way. As there is no inner model with ${{\mathrm {cof}}(\kappa )}$ -many measurable cardinals, the mice we consider are simple and therefore iterability for N is $\Sigma _1$ -definable from the parameter $\omega _1$ , using [Reference Zeman53, Theorem 4.5.5]. All other conditions can obviously be stated by $\Sigma _1$ -formulas using the parameters $\kappa $ and $K\vert \gamma ^{++} \in {\mathrm {H}}(\kappa )$ .

Claim. There is an injection ${i}:{T}\longrightarrow {{\mathrm {H}}(\kappa )\cap {\mathcal {P}}({\kappa })}$ that is definable by a $\Sigma _1$ -formula with parameters in ${\mathrm {H}}(\kappa )\cup \{\kappa \}$ .

Proof of the claim

It clearly suffices to construct an injection from the set of all active nodes to ${\mathrm {H}}(\kappa )\cap {\mathcal {P}}({\kappa })$ . Let $\langle M,\bar {x}\rangle $ be an active node at some $\rho <{{\mathrm {cof}}(\kappa )}^{\mathrm {K}}$ . Since M is the transitive collapse of an elementary submodel of some good premouse $N=\langle \mathrm {J}^{\vec E}_{\alpha },\vec E\rangle $ , we know that M is of the form $\langle \mathrm {J}^A_\epsilon ,A\rangle $ and there is a well-ordering $\lhd $ of M that is definable in M. Let ${\tau }:{\langle M,\lhd \rangle }\longrightarrow {\langle \lambda ,<\rangle }$ denote the corresponding transitive collapse and associate $\langle M,\bar {x}\rangle $ with the element

$$ \begin{align*} \begin{aligned} & \{{{\prec}{0},{{\prec}{\alpha},{\beta}{\succ}}{\succ}}~\vert~{\alpha,\beta<\lambda, ~ \tau^{{-}1}(\alpha)\in\tau^{{-}1}(\beta)}\} \\ & \cup ~ \{{{\prec}{1},{\alpha}{\succ}}~\vert~{\alpha<\lambda, ~ \tau^{{-}1}(\alpha)\in A}\} \\ & ~ \cup ~ \{{{\prec}{2},{\alpha}{\succ}}~\vert~{\alpha<\lambda, ~ \tau^{{-}1}(\alpha)\in\bar{x}}\} \end{aligned} \end{align*} $$

of ${\mathrm {H}}(\kappa )\cap {\mathcal {P}}({\kappa })$ . It is now easy to see that the resulting injection is definable in the desired way.

Claim. No countably closed forcing adds a cofinal branch through T.

Proof of the claim

Let ${\mathbb {P}}$ be a countably closed forcing notion, and let G be ${\mathbb {P}}$ -generic over ${\mathrm {V}}$ . Suppose, towards a contradiction, that there is a cofinal branch b through T in ${\mathrm {V}}[G]$ that is not contained in ${\mathrm {V}}$ . By considering the direct limit of the active nodes along b and using the fact that ${{\mathrm {cof}}(\kappa )}^{\mathrm {V}}$ has uncountable cofinality in ${\mathrm {V}}[G]$ , we obtain a pair $\langle \mathcal {R}_b, x_b\rangle $ such that b (modulo some choice of an almost zero sequence s) can be recovered from $\mathcal {R}_b$ and $x_b$ via the transitive collapses of models of the form

$$\begin{align*}{{\mathrm{Hull}}}^{\mathcal{R}_b}(x_b(\rho) \cup \{x_b\}), \end{align*}$$

for $\rho < {{\mathrm {cof}}(\kappa )}^{\mathrm {K}}$ . As ${{\mathrm {cof}}(\kappa )}^{\mathrm {V}}$ has uncountable cofinality in ${\mathrm {V}}[G]$ , the argument in one of our previous claims then shows that $\mathcal {R}_b \lhd {\mathrm {K}}^{V[G]}$ holds in $V[G]$ . By [Reference Zeman53, Theorem 7.4.11], the core model ${\mathrm {K}}$ is forcing absolute, that is, we have ${\mathrm {K}}^{\mathrm {V}} = {\mathrm {K}}^{{\mathrm {V}}[G]}$ . Therefore, we know that $\mathcal {R}_b$ and hence b is already an element of ${\mathrm {V}}$ , a contradiction.

Fix a strictly increasing, cofinal function ${c}:{{{\mathrm {cof}}(\kappa )}}\longrightarrow {{{\mathrm {cof}}(\kappa )}^{\mathrm {K}}}$ , and let $T_*$ denote the unique partial order defined by the following clauses:

  1. (i) The elements of $T_*$ are functions t such that ${{\mathrm {dom}}(t)}\in {{\mathrm {cof}}(\kappa )}$ and the following statements hold:

    1. (a) If $\alpha \in {{\mathrm {dom}}(t)}$ , then $t(\alpha )$ is a branch through T of order-type $c(\alpha )$ .

    2. (b) If $\alpha <\beta \in {{\mathrm {dom}}(t)}$ , then $t(\alpha )$ is an initial segment of $t(\beta )$ .

  2. (ii) The ordering of $T_*$ is given by inclusion.

It then follows that $T_*$ is a tree of height ${{\mathrm {cof}}(\kappa )}$ with the property that every node has $\kappa $ -many successors. Since the tree T has at least $\kappa ^+$ -many branches, it follows that $T_*$ also has at least $\kappa ^+$ -many branches. Moreover, by using the injection i, it is possible to construct an injection ${i_*}:{T_*}\longrightarrow {{\mathrm {H}}(\kappa )\cap {\mathcal {P}}({\kappa })}$ with $\emptyset \notin {{\mathrm {ran}}(i_*)}$ that is definable by a $\Sigma _1$ -formula with parameters in ${\mathrm {H}}(\kappa )\cup \{\kappa \}$ . Finally, the above computations also imply that forcing with a countably closed partial order does not add a new cofinal branch to $T_*$ .

Define D to be the set of all subsets of $\kappa $ of the form

$$ \begin{align*}y_b ~ = ~ \{{{\prec}{\alpha},{{\prec}{\beta},{\sup(i_*(b\restriction\alpha))}{\succ}}{\succ}}~\vert~{\alpha<{{\mathrm{cof}}(\kappa)}, ~ \beta\in i_*(b\restriction\alpha)}\}\end{align*} $$

for some function b with domain ${{\mathrm {cof}}(\kappa )}$ and the property that $b\restriction \alpha \in T_*$ for all $\alpha <{{\mathrm {cof}}(\kappa )}$ . Since the fact that the tree $T_*$ has at least $\kappa ^+$ -many cofinal branches implies that D has cardinality greater than $\kappa $ and the above computations show that D is definable by a $\Sigma _1$ -formula with parameters in ${\mathrm {H}}(\kappa )\cup \{\kappa \}$ , our assumption yields a perfect embedding ${\iota }:{{}^{{{\mathrm {cof}}(\kappa )}}\kappa }\longrightarrow {{\mathcal {P}}({\kappa })}$ with ${{\mathrm {ran}}(\iota )}\subseteq D$ . Using the fact that ${{\mathrm {cof}}(\kappa )}$ is uncountable, a routine construction now allows us to find

  • a system $\langle {u_s}~\vert ~{s\in {}^{{<}{{\mathrm {cof}}(\kappa )}}2}\rangle $ of elements of ${}^{{<}{{\mathrm {cof}}(\kappa )}}2$ ,

  • a strictly increasing sequence $\langle {\kappa _\alpha }~\vert ~{\alpha <{{\mathrm {cof}}(\kappa )}}\rangle $ that is cofinal in $\kappa $ , and

  • a system $\langle {a_s}~\vert ~{s\in {}^{{<}{{\mathrm {cof}}(\kappa )}}2}\rangle $ of bounded subsets of $\kappa $

such that the following statements hold for all $s,t\in {}^{{<}{{\mathrm {cof}}(\kappa )}}2$ :

  1. (i) If ${\mathrm {lh}}({s}){}={\mathrm {lh}}({t}){}$ , then ${\mathrm {lh}}({u_s}){}={\mathrm {lh}}({u_t}){}$ .

  2. (ii) $a_s$ is a subset of $\kappa _{{\mathrm {lh}}({s}){}}$ .

  3. (iii) If $s\subseteq t$ , then $a_s=a_t\cap \kappa _{{\mathrm {lh}}({s}){}}$ and $u_s\subseteq u_t$ .

  4. (iv) $a_{s^\frown \langle 0\rangle }\neq a_{s^\frown \langle 1\rangle }$ and $u_{s^\frown \langle 0\rangle }\neq u_{s^\frown \langle 1\rangle }$ .

  5. (v) $\iota [\{{x\in {}^{{\mathrm {cof}}(\kappa )}2}~\vert ~{x\restriction {\mathrm {lh}}({s}){}= s}\}]=\{{y\in {{\mathrm {ran}}(\iota )}}~\vert ~{y\cap \kappa _{{\mathrm {lh}}({s}){}}=a_s}\}$ .

  6. (vi) If $\alpha <{\mathrm {lh}}({s}){}$ , then there are $\gamma \leq \delta <\kappa _{{\mathrm {lh}}({s}){}}$ with ${\prec }{\alpha },{{\prec }{\gamma },{\delta }{\succ }}{\succ }\in a_s$ .

Now, let G be ${\mathrm {Add}}({{{\mathrm {cof}}(\kappa )}},{1})$ -generic over ${\mathrm {V}}$ . Set $x_G=\bigcup G\in ({}^{{\mathrm {cof}}(\kappa )}2)^{{\mathrm {V}}[G]}$ and

$$ \begin{align*}y_G ~ = ~ \bigcup\{{a_{x_G\restriction\alpha}}~\vert~{\alpha<{{\mathrm{cof}}(\kappa)}}\} ~ \in ~ {\mathcal{P}}({\kappa})^{{\mathrm{V}}[G]}.\end{align*} $$

In this situation, our construction ensures that there is a function $b_G$ in ${\mathrm {V}}[G]$ such that ${{\mathrm {dom}}(b_G)}={{\mathrm {cof}}(\kappa )}$ , $b_G\restriction \alpha \in T_*$ for all $\alpha <{{\mathrm {cof}}(\kappa )}$ and $y_G=y_{b_G}$ . By our earlier observations, the cofinal branch through $T_*$ induced by $b_G$ is contained in ${\mathrm {V}}$ and hence $b_G$ is an element of ${\mathrm {V}}$ . But this implies that $y_G$ is also contained in the ground model ${\mathrm {V}}$ . Since $x_G$ is the unique element x of $({}^{{{\mathrm {cof}}(\kappa )}}2)^{{\mathrm {V}}[G]}$ with the property that $y_G\cap \kappa _\alpha =a_{x\restriction \alpha }$ holds for all $\alpha <{{\mathrm {cof}}(\kappa )}$ , we can now conclude that $x_G$ is contained in ${\mathrm {V}}$ , a contradiction.

6 Almost disjoint families at limits of measurable cardinals

We now proceed by using the techniques developed in Section 2 to show that large almost disjoint families at cardinals with sufficiently strong large cardinal properties are not simply definable.

Proof of Theorem 1.3

Let $\kappa $ be an iterable cardinal that is a limit of measurable cardinals, let z be an element of ${\mathrm {H}}(\kappa )$ and let A be a subset of ${\mathcal {P}}({\kappa })$ of cardinality greater than $\kappa $ with the property that there exists a $\Sigma _1$ -formula $\varphi (v_0,v_1,v_2)$ with $A=\{{y\subseteq \kappa }~\vert ~{\varphi (\kappa ,y,z)}\}$ . Assume, towards a contradiction, that A is an almost disjoint family in ${\mathcal {P}}({\kappa })$ . Since $\kappa $ is an inaccessible cardinal and the collection of all bounded subsets of $\kappa $ is definable by a $\Sigma _0$ -formula with parameter $\kappa $ , we may then also assume that A consists of unbounded subsets of $\kappa $ . Fix an inaccessible cardinal $\lambda <\kappa $ with $z\in {\mathrm {H}}(\lambda )$ , and use Lemma 2.1 to obtain $x\in A$ and systems $\langle {\nu _s}~\vert ~{s\in {}^{{<}\kappa }\kappa }\rangle $ , $\langle {\kappa _s}~\vert ~{s\in {}^{{<}\kappa }\kappa }\rangle $ , $\langle {U_s}~\vert ~{s\in {}^{{<}\kappa }\kappa }\rangle $ and $\langle {I_s}~\vert ~{s\in {}^{{<}\kappa }\kappa }\rangle $ with $\lambda <\kappa _\emptyset $ and the properties listed in the lemma. Then there exists an ${\mathrm {Add}}({\lambda },{1})$ -nice name $\dot {x}$ for an unbounded subset of $\kappa $ with the property that $\dot {x}^G=i^{I_{c_G}}_{0,\infty }(x)$ holds whenever G is ${\mathrm {Add}}({\lambda },{1})$ -generic over ${\mathrm {V}}$ , $c_G=\bigcup G\in ({}^\lambda 2)^{{\mathrm {V}}[G]}$ and $I_{c_G}$ is the unique linear iteration of $\langle {\mathrm {V}},\{{U_{c_G\restriction \xi }}~\vert ~{\xi <\lambda }\}\rangle $ of length $\sup _{\xi <\lambda }{\mathrm {lh}}({I_{c_G\restriction \xi }}){}$ in ${\mathrm {V}}[G]$ with $U^{I_G}_\alpha =U^{I_{c_G\restriction \xi }}_\alpha $ for all $\xi <\lambda $ and $\alpha <{\mathrm {lh}}({I_{c_G\restriction \xi }}){}$ . Note that, by Lemma 2.2, the elementarity of $i^{I_{c_G}}_{0,\infty }$ and the upwards absoluteness of $\Sigma _1$ -statements between $M^{I_{c_G}}_\infty $ and ${\mathrm {V}}[G]$ ensures that

(7)

holds in ${\mathrm {V}}$ .

Claim. If $G_0\times G_1$ is $({\mathrm {Add}}({\lambda },{1})\times {\mathrm {Add}}({\lambda },{1}))$ -generic over ${\mathrm {V}}$ , then $\dot {x}^{G_0}\neq \dot {x}^{G_1}$ .

Proof of the claim

Given $i<2$ , the absoluteness of the iterated ultrapower construction ensures that $(I_{c_{G_i}})^{{\mathrm {V}}[G_i]}=(I_{c_{G_i}})^{{\mathrm {V}}[G_0,G_1]}$ holds, and this implies that

$$ \begin{align*}\dot{x}^{G_i} ~ = ~ (i^{I_{c_{G_i}}}_{0,\infty}(x))^{{\mathrm{V}}[G_0,G_1]}.\end{align*} $$

Since mutual genericity implies that $c_{G_0}\neq c_{G_1}$ , the desired inequality now directly follows from an application of statement (iii) of Lemma 2.2 in ${\mathrm {V}}[G_0,G_1]$ .

Pick an elementary submodel $M_0$ of ${\mathrm {H}}(\kappa ^+)$ of cardinality $\kappa $ with ${}^{{<}\kappa }M_0\subseteq M_0$ that contains ${\mathrm {H}}(\kappa )$ and all objects listed above. Since iterable cardinals are weakly compact, we can find a transitive set $M_1$ of cardinality $\kappa $ and an elementary embedding ${j}:{M_0}\longrightarrow {M_1}$ with ${{\mathrm {crit}}\left ({j}\right )}=\kappa $ (see [Reference Hauser20, Theorem 1.3]). Then $j(\dot {x})$ is an ${\mathrm {Add}}({\lambda },{1})$ -name for an unbounded subset of $j(\kappa )$ and there is a canonical ${\mathrm {Add}}({\lambda },{1})$ -name $\dot {\gamma }$ for an ordinal in the interval $[\kappa ,j(\kappa ))$ with the property that

$$ \begin{align*}\dot{\gamma}^G ~ = ~ \min(j(\dot{x})^G\setminus\kappa)\end{align*} $$

holds whenever G is ${\mathrm {Add}}({\lambda },{1})$ -generic over ${\mathrm {V}}$ .

Claim. If $G_0\times G_1$ is $({\mathrm {Add}}({\lambda },{1})\times {\mathrm {Add}}({\lambda },{1}))$ -generic over ${\mathrm {V}}$ , then $\dot {\gamma }^{G_0}\neq \dot {\gamma }^{G_1}$ .

Proof of the claim

Given an ${\mathrm {Add}}({\lambda },{1})$ -name $\dot {a}$ , let $\dot {a}_l$ and $\dot {a}_r$ denote the canonical $({\mathrm {Add}}({\lambda },{1})\times {\mathrm {Add}}({\lambda },{1}))$ -names such that $\dot {a}_l^{G_0\times G_1}=\dot {a}^{G_0}$ and $\dot {a}_r^{G_0\times G_1}=\dot {a}^{G_1}$ holds whenever $G_0\times G_1$ is $({\mathrm {Add}}({\lambda },{1})\times {\mathrm {Add}}({\lambda },{1}))$ -generic over ${\mathrm {V}}$ . Given an ${\mathrm {Add}}({\lambda },{1})$ -name $\dot {a}$ in $M_0$ , we then have $j(\dot {a}_l)=j(\dot {a})_l$ and $j(\dot {a}_r)=j(\dot {a})_r$ .

Assume, towards a contradiction, that

$$ \begin{align*}\langle p,q\rangle\Vdash_{{\mathrm{Add}}({\lambda},{1})\times{\mathrm{Add}}({\lambda},{1})}{\text{"}\hspace{0.3ex}{\dot{\gamma}_l=\dot{\gamma}_r}\hspace{0.3ex}\text{"}}\end{align*} $$

holds for some condition $\langle p,q\rangle $ in $({\mathrm {Add}}({\lambda },{1})\times {\mathrm {Add}}({\lambda },{1}))$ .

Subclaim. $\langle p,q\rangle \Vdash _{{\mathrm {Add}}({\lambda },{1}) \times {\mathrm {Add}}({\lambda },{1})}{\text {"}\hspace {0.3ex}{\dot {x}_l\cap \dot {x}_r \textit {is unbounded in } \check {\kappa }}\hspace {0.3ex}\text {"}}.$

Proof of the subclaim

Let $G_0\times G_1$ be $({\mathrm {Add}}({\lambda },{1})\times {\mathrm {Add}}({\lambda },{1}))$ -generic over ${\mathrm {V}}$ with $\langle p,q\rangle \in G_0\times G_1$ . By standard arguments, there exists an elementary embedding

$$ \begin{align*}{j_*}:{M_0[G_0,G_1]}\longrightarrow{M_1[G_0,G_1]}\end{align*} $$

with $j_*(\dot {b}^{G_0\times G_1})=j(\dot {b})^{G_0\times G_1}$ for every $({\mathrm {Add}}({\lambda },{1})\times {\mathrm {Add}}({\lambda },{1}))$ -name $\dot {b}$ in $M_0$ . Then our assumptions ensure that

$$ \begin{align*} \begin{aligned} \dot{\gamma}^{G_0} ~ = ~ \dot{\gamma}^{G_1} ~ \in ~ & ~ j(\dot{x})^{G_0} ~ \cap ~ j(\dot{x})^{G_1} ~ \cap ~ [\kappa,j(\kappa)) \\ & = ~ j_*(\dot{x}_l^{G_0\times G_1}) ~ \cap ~ j_*(\dot{x}_r^{G_0\times G_1}) ~ \cap ~ [\kappa,j_*(\kappa)) ~ \neq ~ \emptyset. \end{aligned} \end{align*} $$

In particular, if $\alpha <\kappa $ , then the elementarity of $j_*$ and the fact that $j_*(\alpha )=\alpha $ directly imply that

$$ \begin{align*}\dot{x}_l^{G_0\times G_1} ~ \cap ~ \dot{x}_r^{G_0\times G_1} ~ \cap ~ (\alpha,\kappa) ~ \neq ~ \emptyset.\end{align*} $$

This proves the statement of the subclaim.

We now use the fact that $\kappa $ is an iterable cardinal to find a transitive model M of ${\mathrm {ZFC}}^-$ of cardinality $\kappa $ with $M_0\in M$ and a weakly amenable M-ultrafilter F on $\kappa $ such that $\langle M,F\rangle $ is iterable. Pick an elementary submodel $\langle X,\in ,\bar {F}\rangle $ of $\langle M,\in ,F\rangle $ of cardinality $\lambda $ with ${}^{{<}\lambda }X\subseteq X$ that contains ${\mathrm {H}}(\lambda )$ , $M_0$ and all other relevant objects. Let ${\pi }:{X}\longrightarrow {N_0}$ denote the corresponding transitive collapse and set $F_0=\pi [\bar {F}]$ . By [Reference Kanamori28, Theorem 19.15], we know that $\langle N_0,F_0\rangle $ is iterable. Let $\langle N_1,F_1\rangle $ denote the $\kappa $ -th iterate of $\langle N_0,F_0\rangle $ and let ${i}:{N_0}\longrightarrow {N_1}$ denote the corresponding elementary embedding. Then $(i\circ \pi )(\kappa )=\kappa $ , $(i\circ \pi )(z)=z$ , $(i\circ \pi )(\langle p,q\rangle )=\langle p,q\rangle $ and ${\mathrm {H}}(\pi (\kappa ))^{N_0}={\mathrm {H}}(\pi (\kappa ))^{N_1}$ . Since ${\mathrm {Add}}({\lambda },{1})\times {\mathrm {Add}}({\lambda },{1})$ is ${<}\lambda $ -closed and a subset of $N_0$ , we know that $N_0$ contains all sequences of conditions in ${\mathrm {Add}}({\lambda },{1})\times {\mathrm {Add}}({\lambda },{1})$ of length less than $\lambda $ and therefore the fact that $\vert {N_0}\vert =\lambda $ allows us to find a filter $H_0\times H_1$ on ${\mathrm {Add}}({\lambda },{1})\times {\mathrm {Add}}({\lambda },{1})$ that contains $\langle p,q\rangle $ and is generic over $N_0$ by constructing a descending sequence of conditions in ${\mathrm {Add}}({\lambda },{1})\times {\mathrm {Add}}({\lambda },{1})$ below $\langle p,q\rangle $ that has length $\lambda $ and intersects all dense subsets of ${\mathrm {Add}}({\lambda },{1})\times {\mathrm {Add}}({\lambda },{1})$ contained in $N_0$ . Moreover, since ${\mathrm {H}}(\lambda ^+)^{N_1}\subseteq N_0$ , we know that the filter $H_0\times H_1$ is also generic over $N_1$ .

Given $i<2$ , we now define $x_i=(i\circ \pi )(\dot {x})^{H_i}$ . Set $N=(i\circ \pi )(M_0)$ . Then ${\mathrm {Add}}({\lambda },{1})\subseteq N$ , $(i\circ \pi )(\dot {x})\in N$ and $H_0\times H_1$ is $({\mathrm {Add}}({\lambda },{1})\times {\mathrm {Add}}({\lambda },{1}))$ -generic over N. Since our first claim and the above subclaim show that

$$ \begin{align*}\langle p,q\rangle\Vdash_{{\mathrm{Add}}({\lambda},{1})\times{\mathrm{Add}}({\lambda},{1})}{\text{"}\hspace{0.3ex}{\dot{x}_l\neq\dot{x}_r \textit{ and } \dot{x}_l\cap\dot{x}_r\ \textit{is unbounded in } \check{\kappa}}\hspace{0.3ex}\text{"}} \end{align*} $$

holds in $M_0$ , elementarity implies that $x_0$ and $x_1$ are distinct subsets of $\kappa $ and $x_0\cap x_1$ is unbounded in $\kappa $ . Moreover, using Equation (7), $\Sigma _1$ -upwards absoluteness and the fact that $\Sigma _1$ -statements in the forcing language can be expressed by $\Sigma _1$ -formulas, we know that

$$ \begin{align*}\langle p,q\rangle\Vdash_{{\mathrm{Add}}({\lambda},{1})\times{\mathrm{Add}}({\lambda},{1})}{\text{"}\hspace{0.3ex}{\varphi(\check{\kappa},\dot{x}_l,\check{z})\wedge\varphi(\check{\kappa},\dot{x}_r,\check{z})}\hspace{0.3ex}\text{"}}\end{align*} $$

holds in $M_0$ and therefore elementarity allows us to conclude that $\varphi (\kappa ,x_i,z)$ holds in $N[H_0,H_1]$ for all $i<2$ . By $\Sigma _1$ -upwards absoluteness, this implies that $x_0$ and $x_1$ are distinct elements of A, contradicting the fact that A is an almost disjoint family.

Now, let G be ${\mathrm {Add}}({\lambda },{\kappa ^+})$ -generic over ${\mathrm {V}}$ . Since $\lambda $ is inaccessible, the model ${\mathrm {V}}[G]$ has the same cardinals as ${\mathrm {V}}$ . Let $\langle {G_\delta }~\vert ~{\delta <\kappa ^+}\rangle $ denote the induced sequence of filters on ${\mathrm {Add}}({\lambda },{1})$ . Given $\delta <\varepsilon <\kappa ^+$ , the filter $G_\delta \times G_\varepsilon $ on ${\mathrm {Add}}({\lambda },{1})\times {\mathrm {Add}}({\lambda },{1})$ is generic over ${\mathrm {V}}$ and therefore the previous claim implies that $\dot {\gamma }^{G_\delta }\neq \dot {\gamma }^{G_\varepsilon }$ . In particular, the map

$$ \begin{align*}{\iota}:{\kappa^+}\longrightarrow{j(\kappa)};~{\delta}\longmapsto{\dot{\gamma}^{G_\delta}}\end{align*} $$

is an injection. Since $j(\kappa )<\kappa ^+$ , this yields a contradiction.

The conclusion of Theorem 1.3 provably does not generalize to $\Sigma _1$ -definitions using arbitrary subsets of $\kappa $ as parameters. If $\kappa $ is an infinite cardinal and $z\subseteq \kappa $ codes an injective sequence $\langle {s_\beta }~\vert ~{\beta <\kappa }\rangle $ of elements of ${}^{{<}\kappa }2$ with the property that the set

$$ \begin{align*}I ~ = ~ \{{x\in{}^\kappa 2}~\vert~{\forall\alpha<\kappa ~ \exists\beta<\kappa ~ x\restriction\alpha=s_\beta}\}\end{align*} $$

has cardinality greater than $\kappa $ , then the collection $\{{\{{\beta <\kappa }~\vert ~{s_\beta \subseteq x}\}}~\vert ~{x\in I}\}$ is an almost disjoint family of cardinality greater than $\kappa $ that is definable by a $\Sigma _1$ -formula with parameter z. Note that such sequences exist for every strong limit cardinal $\kappa $ , or, more generally, for every cardinal $\kappa $ that is a strong limit cardinal in an inner model M satisfying $(2^\kappa )^M\geq \kappa ^+$ .

7 Long well-orders at limits of measurable cardinals

In order to motivate the statement of Theorem 1.4, we first show how classical results of Dehornoy can easily be used to show that, if $\kappa $ is a limit of measurable cardinals, then no well-ordering of ${\mathcal {P}}({\kappa })$ is definable by a $\Sigma _1$ -formula with parameters in ${\mathrm {H}}(\kappa )\cup \{\kappa \}$ . Moreover, if $\kappa $ has uncountable cofinality, then we can also easily show that no injection from $\kappa ^+$ into ${\mathcal {P}}({\kappa })$ is definable in this way. This nondefinability result will be a direct consequence of the following theorem.

Theorem 7.1. If $\delta $ is a measurable cardinal, $z\in {\mathrm {H}}(\delta )$ and $\nu $ is a cardinal with ${{\mathrm {cof}}(\nu )}\neq \delta $ and $\mu ^\delta <\nu $ for all $\mu <\nu $ , then the following statements hold for $\kappa \in \{\nu ,\nu ^+\}$ :

  1. (i) No well-ordering of ${\mathcal {P}}({\kappa })$ is definable by a $\Sigma _1$ -formula with parameters $\nu $ , $\nu ^+$ and z.

  2. (ii) If ${{\mathrm {cof}}(\kappa )}>\omega $ , then no injection from $\kappa ^+$ into ${\mathcal {P}}({\kappa })$ is definable by a $\Sigma _1$ -formula with parameters $\kappa $ and z.

The proof of the above theorem is based on two standard results about measurable cardinals. A proof of the first of these lemmas is contained in the proof of [Reference Lücke and Schlicht38, Lemma 1.3]:

Lemma 7.2. Let U be a normal ultrafilter on a measurable cardinal $\delta $ , and let $\nu>\delta $ be a cardinal with ${{\mathrm {cof}}(\nu )}\neq \delta $ and $\mu ^\delta <\nu $ for all $\mu <\nu $ . If ${j}:{{\mathrm {V}}}\longrightarrow {{\mathrm {Ult}}({{\mathrm {V}}},{U})}$ is the induced ultrapower embedding, than $j(\nu )=\nu $ and $j(\nu ^+)=\nu ^+$ . $\Box $

Lemma 7.3. Let U be a normal ultrafilter on a measurable cardinal $\delta $ , and let

$$ \begin{align*}\langle\langle{N_\alpha}~\vert~{\alpha\in{\mathrm{On}}}\rangle, \langle{{j_{\alpha,\beta}}:{N_\alpha}\longrightarrow{N_\beta}}~\vert~{\alpha\leq\beta\in{\mathrm{On}}}\rangle\rangle\end{align*} $$

denote the system of iterated ultrapowers of $\langle {\mathrm {V}},\in ,U\rangle $ . If $\nu $ is a cardinal with ${{\mathrm {cof}}(\nu )}\neq \delta $ and $\mu ^\delta <\nu $ for all $\mu <\nu $ , then $j_{0,\alpha }(\kappa )=\kappa $ holds for $\kappa \in \{\nu ,\nu ^+\}$ and all $\alpha <\kappa $ .

Proof. We start by using induction to show that $j_{0,\alpha }(\nu )=\nu $ holds for all $\alpha <\nu $ . In the successor case, the desired conclusion follows directly from the induction hypothesis and an application of Lemma 7.2 in $N_\alpha $ . Hence, we may assume that $\alpha $ is a limit ordinal. Pick $\bar {\alpha }<\alpha $ and $\xi <\nu $ . Then elementarity allows us to apply [Reference Kanamori28, Corollary 19.7. (a)] in $N_{\bar {\alpha }}$ to conclude that $j_{\bar {\alpha },\alpha }(\xi )<\nu $ . Since every element of $j_{0,\alpha }(\nu )\geq \nu $ is of the form $j_{\bar {\alpha },\alpha }(\xi )$ for some $\bar {\alpha }<\alpha $ and $\xi <j_{0,\bar {\alpha }}(\nu )=\nu $ , these computations show that $j_{0,\alpha }(\nu )=\nu $ holds.

Next, we inductively show that $j_{0,\alpha }(\nu )<\nu ^+$ holds for all $\alpha <\nu ^+$ , where the successor step is again a direct consequence of the induction hypothesis and Lemma 7.2. In the other case, if $\alpha \in \nu ^+\cap {\mathrm {Lim}}$ and $j_{0,\bar {\alpha }}(\nu )<\nu ^+$ holds for all $\bar {\alpha }<\alpha $ , then every element of $j_{0,\alpha }(\nu )$ is of the form $j_{\bar {\alpha },\alpha }(\xi )$ with $\bar {\alpha }<\alpha $ and $\xi <j_{0,\bar {\alpha }}(\nu )$ and this shows that $\vert {j_{0,\alpha }(\nu )}\vert \leq \nu \cdot \vert {\alpha }\vert <\nu ^+$ .

Finally, we have $\nu ^+\leq j_{0,\alpha }(\nu ^+)\leq \vert {j_{0,\alpha }(\nu )}\vert ^+$ for all $\alpha <\nu ^+$ . Since the above computations show that $\vert {j_{0,\alpha }(\nu )}\vert =\nu $ holds for all $\alpha <\nu ^+$ , this shows that $j_{0,\alpha }(\nu ^+)=\nu ^+$ holds for all $\alpha <\nu ^+$ .

Proof of Theorem 7.1

Let U be a normal ultrafilter on a measurable cardinal $\delta $ , and let

$$ \begin{align*}\langle\langle{N_\alpha}~\vert~{\alpha\in{\mathrm{On}}}\rangle, ~ \langle{{j_{\alpha,\beta}}:{N_\alpha}\longrightarrow{N_\beta}}~\vert~{\alpha\leq\beta\in{\mathrm{On}}}\rangle\rangle\end{align*} $$

denote the system of iterated ultrapowers of $\langle {\mathrm {V}},\in ,U\rangle $ . Moreover, for every $\alpha \in {\mathrm {Lim}}$ , we define ${M_\alpha =\bigcap \{{N_\xi }~\vert ~{\xi <\alpha }\}}$ . Then [Reference Dehornoy11, Proposition 1.6.1] shows that each $M_\alpha $ is an inner model of ${\mathrm {ZF}}$ .

(i) Assume, towards a contradiction, that there is a $\Sigma _1$ -formula $\varphi (v_0,\ldots ,v_4)$ with the property that

$$ \begin{align*}\lhd ~ = ~ \{{\langle x,y\rangle}~\vert~{\varphi(x,y,z,\nu,\nu^+)}\}\end{align*} $$

is a well-ordering of ${\mathcal {P}}({\kappa })$ . For all $\alpha \in {\mathrm {On}}$ , we define $\lhd _\alpha =j_{0,\alpha }({\lhd })$ . Given $\alpha <\omega ^2$ , Lemma 7.3 implies that $j_{0,\alpha }(\nu )=\nu $ and $j_{0,\alpha }(\nu ^+)=\nu ^+$ . In particular, elementarity implies that $\lhd _\alpha $ is a well-ordering of ${\mathcal {P}}({\kappa })^{N_\alpha }$ and the sequence $\langle {\lhd _{\alpha +\beta }}~\vert ~{\beta <\omega ^2}\rangle $ is an element of $N_\alpha $ . By our assumptions, elementarity and $\Sigma _1$ -upwards absoluteness imply that ${\lhd _\beta }\subseteq {\lhd _\alpha }\subseteq {\lhd }$ for all $\alpha \leq \beta <\omega ^2$ . Define ${\blacktriangleleft }=\bigcap \{{\lhd _\alpha }~\vert ~{\alpha <\omega ^2}\}$ . If $\alpha <\omega ^2$ , then ${\blacktriangleleft }=\bigcap \{{\lhd _{\alpha +\beta }}~\vert ~{\beta <\omega ^2}\}$ and therefore ${\blacktriangleleft }\in N_\alpha $ . This shows that ${\blacktriangleleft }$ is an element of $M_{\omega ^2}$ , and it follows that ${\blacktriangleleft }$ is a well-ordering of ${\mathcal {P}}({\kappa })^{M_{\omega ^2}}$ . But this yields a contradiction because [Reference Dehornoy11, Theorem 5.3.4] shows that $M_{\omega ^2}$ contains a subset $\mathcal {G}_{\omega ^2}$ of ${\mathcal {P}}({j_{0,\omega ^2}(\delta )})$ with the property that $M_{\omega ^2}$ does not contain a well-ordering of the set $\mathcal {G}_{\omega ^2}$ .

(ii) Assume, towards a contradiction, that ${{\mathrm {cof}}(\kappa )}>\omega $ and there is an injection ${\iota }:{\kappa ^+}\longrightarrow {{\mathcal {P}}({\kappa })}$ that is definable by a $\Sigma _1$ -formula $\varphi (v_0,\ldots ,v_3)$ and the parameters $\kappa $ and z.

Claim. If $\alpha <\kappa $ , then $j_{0,\alpha }(\iota )=\iota $ .

Proof of the claim

Since Lemma 7.3 shows that $j_{0,\alpha }(\kappa )=\kappa $ , we also know that $j_{0,\alpha }(\kappa ^+)=\kappa ^+$ and therefore elementarity implies that $j_{0,\alpha }(\iota )$ is an injection from $\kappa ^+$ into ${\mathcal {P}}({\kappa })$ that is definable in $N_\alpha $ by the formula $\varphi $ and the parameters $\kappa $ and z. But then $\Sigma _1$ -upwards absoluteness implies that $j_{0,\alpha }(\iota )\subseteq \iota $ and this allows us to conclude that $j_{0,\alpha }(\iota )=\iota $ .

The above claim directly implies that the injection $\iota $ is an element of $ M_\kappa $ . By [Reference Dehornoy11, Theorem B. (i)], the fact that ${{\mathrm {cof}}(\kappa )}>\omega $ implies that $N_\kappa = M_\kappa =\bigcap _{\alpha <\kappa }N_\alpha $ and hence $\vert {{\mathcal {P}}({\kappa })^{N_\kappa }}\vert \geq \kappa ^+$ . Since $N_\kappa $ is a direct limit and $j_{0,\kappa }(\delta )=\kappa $ , we also know that

$$ \begin{align*}{\mathcal{P}}({\kappa})^{N_\kappa} ~ = ~ \{{j_{\alpha,\kappa}(x)}~\vert~{\alpha<\kappa, ~ x\in{\mathcal{P}}({j_{0,\alpha}(\delta)})^{N_\alpha}}\}.\end{align*} $$

But our assumptions imply that $2^\delta <\kappa $ and therefore

$$ \begin{align*}\vert{{\mathcal{P}}({j_{0,\alpha}(\delta)})^{N_\alpha}}\vert ~ \leq ~ j_{0,\alpha}(2^\delta) ~ < ~ j_{0,\alpha}(\kappa) ~ = ~ \kappa\end{align*} $$

holds for all $\alpha <\kappa $ . We can now conclude that $\vert {{\mathcal {P}}({\kappa })^{N_\kappa }}\vert =\kappa $ , a contradiction.

Corollary 7.4. Let $\kappa $ be a limit of measurable cardinals.

  1. (i) No well-ordering of ${\mathcal {P}}({\kappa })$ is definable by a $\Sigma _1$ -formula with parameters in ${\mathrm {H}}(\kappa )\cup \{\kappa ,\kappa ^+\}$ .

  2. (ii) If ${{\mathrm {cof}}(\kappa )}>\omega $ , then no injection from $\kappa ^+$ into ${\mathcal {P}}({\kappa })$ is definable by a $\Sigma _1$ -formula with parameters in ${\mathrm {H}}(\kappa )\cup \{\kappa \}$ . $\Box $

We now proceed by proving our result on the nonexistence of long $\Sigma _1$ -well-orders.

Proof of Theorem 1.4

Let $\kappa $ be a limit of measurable cardinals with ${{\mathrm {cof}}(\kappa )}=\omega $ , let D be a subset of ${\mathcal {P}}({\kappa })$ of cardinality greater than $\kappa $ and let $\lhd $ be a well-ordering of D that is definable by a $\Sigma _1$ -formula with parameter $\kappa $ . Then D is definable in the same way and we can pick $\Sigma _1$ -formulas $\varphi (v_0,v_1)$ and $\psi (v_0,v_1,v_2)$ with $D=\{{x}~\vert ~{\varphi (x,\kappa )}\}$ and $\lhd = \{{\langle x,y\rangle }~\vert ~{\psi (x,y,\kappa )}\}$ . Now, use Lemma 2.1 to find $x\in D$ and systems $\langle {\nu _s}~\vert ~{s\in {}^{{<}\omega }\kappa }\rangle $ , $\langle {\kappa _s}~\vert ~{s\in {}^{{<}\omega }\kappa }\rangle $ , $\langle {U_s}~\vert ~{s\in {}^{{<}\omega }\kappa }\rangle $ and $\langle {I_s}~\vert ~{s\in {}^{{<}\omega }\kappa }\rangle $ with the listed properties. Pick an ${\mathrm {Add}}({\omega },{1})$ -nice name $\dot {x}$ for a subset of $\kappa $ such that $\dot {x}^G=i^{I_{c_G}}_{0,\infty }(x)$ holds whenever G is ${\mathrm {Add}}({\omega },{1})$ -generic over ${\mathrm {V}}$ , $c_G=\bigcup G\in ({}^\omega 2)^{{\mathrm {V}}[G]}$ and $I_{c_G}$ is the unique linear iteration of $\langle {\mathrm {V}},\{{U_{c_G\restriction n}}~\vert ~{n<\omega }\}\rangle $ of length $\sup _{n<\omega }{\mathrm {lh}}({I_{c_G\restriction n}}){}$ in ${\mathrm {V}}[G]$ with $U^{I_G}_\alpha =U^{I_{c_G\restriction n}}_\alpha $ for all $n<\omega $ and $\alpha <{\mathrm {lh}}({I_{c_G\restriction n}}){}$ . The elementarity of $i^{I_{c_G}}_{0,\infty }$ and $\Sigma _1$ -upwards absoluteness between $M^{I_{c_G}}_\infty $ and ${\mathrm {V}}[G]$ then imply that

(8)

holds in ${\mathrm {V}}$ . Finally, let ${z}:{\omega }\longrightarrow {2}$ denote the constant function with value $0$ and for each $n<\omega $ , set $\kappa _n=\kappa _{z\restriction n}$ and $U_n=U_{z\restriction n}$ . Then the sequence $\langle {\kappa _n}~\vert ~{n<\omega }\rangle $ is strictly increasing and cofinal in $\kappa $ .

Pick a sufficiently large regular cardinal $\theta $ and a countable elementary submodel X of ${\mathrm {H}}(\theta )$ containing $\kappa $ , $\dot {x}$ , $\langle {\kappa _s}~\vert ~{s\in {}^{{<}\omega }\kappa }\rangle $ , $\langle {U_s}~\vert ~{s\in {}^{{<}\omega }\kappa }\rangle $ and $\langle {I_s}~\vert ~{s\in {}^{{<}\omega }\kappa }\rangle $ . Let ${\pi }:{X}\longrightarrow {M}$ denote the corresponding transitive collapse. Define $\bar {\kappa }=\pi (\kappa )$ and, given $n<\omega $ , set $\bar {\kappa }_n=\pi (\kappa _n)$ and $\bar {U}_n=\pi (U_n)$ . Then [Reference Steel49, Lemma 3.5] shows that the pair $\langle M,\{{\bar {U}_n}~\vert ~{n<\omega }\}\rangle $ is linearly iterable. Let $\bar {I}$ denote the unique linear iteration of $\langle M,\{{\bar {U}_n}~\vert ~{n<\omega }\}\rangle $ of length $\kappa $ with the property that

$$ \begin{align*}U^{\bar{I}}_\alpha ~ = ~ i^{\bar{I}}_{0,\alpha}(\bar{U}_{\min\{n<\omega\vert\alpha<\kappa_n\}})\end{align*} $$

holds for all $\alpha <\kappa $ . Set $N=M^{\bar {I}}_{0,\infty }$ and ${j=i^{\bar {I}}_{0,\infty }}:{M}\longrightarrow {N}$ . Then it is easy to see that $j(\bar {\kappa }_n)=\kappa _n$ for all $n<\omega $ and this implies that $j(\bar {\kappa })=\kappa $ .

Now, pick $c\in {}^\omega 2$ with the property that $G_c=\{{c\restriction n}~\vert ~{n<\omega }\}$ is ${\mathrm {Add}}({\omega },{1})$ -generic over M. Then $G_c$ is also ${\mathrm {Add}}({\omega },{1})$ -generic over N and we define

$$ \begin{align*}x_c ~ = ~ j(\pi(\dot{x}))^{G_c} ~ \in ~ {\mathcal{P}}({\kappa})^{N[G_c]}.\end{align*} $$

Claim. If $c\in {}^\omega 2$ has the property that $G_c$ is ${\mathrm {Add}}({\omega },{1})$ -generic over M, then $x_c\in D$ .

Proof of the claim

By $\Sigma _1$ -absoluteness, we know that Equation (8) implies that the given forcing statement also holds in ${\mathrm {H}}(\theta )$ . This shows that

holds in N. But this allows us to conclude that $\varphi (x_c,\kappa )$ holds in $N[G_c]$ and $\Sigma _1$ -upwards absoluteness implies that this statement also holds in ${\mathrm {V}}$ .

Let E denote the set of all pairs $\langle c,d\rangle $ in ${}^\omega 2\times {}^\omega 2$ with the property that $G_c\times G_d$ is $({\mathrm {Add}}({\omega },{1})\times {\mathrm {Add}}({\omega },{1}))$ -generic over M. Then E is a comeager subset of ${}^\omega 2\times {}^\omega 2$ and a classical result of Mycielski (see [Reference Kechris30, Theorem 19.1]) yields a continuous injection ${p}:{{}^\omega 2}\longrightarrow {{}^\omega 2}$ with $\langle p(a),p(b)\rangle \in E$ for all distinct $a,b\in {}^\omega 2$ .

Claim. The map

$$ \begin{align*}{\iota}:{{}^\omega 2}\longrightarrow{D};~{a}\longmapsto{x_{p(a)}}\end{align*} $$

is an injection.

Proof of the claim

Given an ${\mathrm {Add}}({\omega },{1})$ -name $\dot {y}$ , let $\dot {y}_l$ and $\dot {y}_r$ denote the canonical $({\mathrm {Add}}({\omega },{1})\times {\mathrm {Add}}({\omega },{1}))$ -names such that $\dot {y}_l^{G_0\times G_1}=\dot {y}^{G_0}$ and $\dot {y}_r^{G_0\times G_1}=\dot {y}^{G_1}$ holds whenever $G_0\times G_1$ is $({\mathrm {Add}}({\omega },{1})\times {\mathrm {Add}}({\omega },{1}))$ -generic over ${\mathrm {V}}$ . If $G_0\times G_1$ is $({\mathrm {Add}}({\omega },{1})\times {\mathrm {Add}}({\omega },{1}))$ -generic over ${\mathrm {V}}$ and $i<2$ , then $(I_{c_{G_i}})^{{\mathrm {V}}[G_i]}=(I_{c_{G_i}})^{{\mathrm {V}}[G_0,G_1]}$ and this shows that

$$ \begin{align*}\dot{x}^{G_i} ~ = ~ (i^{I_{c_{G_i}}}_{0,\infty}(x))^{{\mathrm{V}}[G_0,G_1]}\end{align*} $$

holds for the ${\mathrm {Add}}({\omega },{1})$ -name $\dot {x}$ fixed at the beginning of the proof of Theorem 1.3. Therefore, we can apply Lemma 2.2 to see that

holds in ${\mathrm {V}}$ and, by $\Sigma _1$ -absoluteness, this statement also holds in ${\mathrm {H}}(\theta )$ .

Now, given $a,b\in {}^\omega 2$ with $a\neq b$ , we have

$$ \begin{align*} \begin{aligned} \iota(a) ~ = & ~ x_{p(a)} ~ = ~ j(\pi(\dot{x}))^{G_{p(a)}} ~ = ~ j(\pi(\dot{x}_l))^{G_{p(a)}\times G_{p(b)}} \\ & \neq ~ j(\pi(\dot{x}_r))^{G_{p(a)}\times G_{p(b)}} ~ = ~ j(\pi(\dot{x}))^{G_{p(b)}} ~ = ~ x_{p(b)} ~ = ~ \iota(b).\\[-33pt] \end{aligned} \end{align*} $$

In the following, let $\blacktriangleleft $ denote the unique binary relation on ${}^\omega 2$ with

$$ \begin{align*}a\blacktriangleleft b ~ \Longleftrightarrow ~ x_{p(a)}\lhd x_{p(b)}\end{align*} $$

for all $a,b\in {}^\omega 2$ . Then the above claim implies that $\blacktriangleleft $ is a well-ordering of ${}^\omega 2$ .

Claim. The following statements are equivalent for all $a,b\in {}^\omega 2$ :

  1. (i) $a\blacktriangleleft b$ .

  2. (ii) There exists a countable transitive model W of ${\mathrm {ZFC}}^-$ and elements $\delta $ , $\vec {\delta }$ , $\vec {F}$ and I of W such that the following statements hold:

    • W contains M, $p(a)$ , $p(b)$ and a surjection from $\omega $ onto M.

    • $\vec {\delta }=\langle {\delta _n}~\vert ~{n<\omega }\rangle $ is a strictly increasing sequence of cardinals in W with $\delta =\sup _{n<\omega }\delta _n$ .

    • $\vec {F}=\langle {F_n}~\vert ~{n<\omega }\rangle $ is a sequence with the property that $F_n$ is a normal ultrafilter on $\delta _n$ in W for all $n<\omega $ .

    • If ${k}:{\bar {W}}\longrightarrow {W}$ is an elementary embedding of a transitive model $\bar {W}$ into W and $\mathcal {E}\in \bar {W}$ satisfies $k(\mathcal {E})=\{{F_n}~\vert ~{n<\omega }\}$ , then the pair $\langle \bar {W},\mathcal {E}\rangle $ is $\alpha $ -iterable (see [Reference Steel49, p. 131]) for all $\alpha <\omega _1$ .

    • I is the unique linear iteration of $\langle M,\{{\bar {U}_n}~\vert ~{n<\omega }\}\rangle $ of length $\delta $ with the property that

      $$ \begin{align*}U^I_\alpha ~ = ~ i^I_{0,\alpha}(\bar{U}_{\min\{n<\omega\vert\alpha<\delta_n\}})\end{align*} $$
      holds for all $\alpha <\delta $ .
    • The statement

      $$ \begin{align*}\psi(i^I_{0,\infty}(\pi(\dot{x}))^{G_{p(a)}},i^I_{0,\infty}(\pi(\dot{x}))^{G_{p(b)}},\delta)\end{align*} $$
      holds in W.

Proof of the claim

First, assume that (i) holds. Pick a sufficiently large regular cardinal $\vartheta>\theta $ and a countable elementary submodel Y of ${\mathrm {H}}(\vartheta )$ containing $\theta $ , $p(a)$ , $p(b)$ , $\langle {U_n}~\vert ~{n<\omega }\rangle $ , X and $\bar {I}$ . Let ${\tau }:{Y}\longrightarrow {W}$ denote the the corresponding transitive collapse. Given $n<\omega $ , set $\delta _n=\tau (\kappa _n)$ and $F_n=\tau (U_n)$ . Moreover, define $\delta =\tau (\kappa )$ and $I=\tau (\bar {I})$ . In this situation, [Reference Steel49, Lemma 3.5] shows that the pair $\langle W,\{{F_n}~\vert ~{n<\omega }\}\rangle $ is linearly iterable. Another application of [Reference Steel49, Lemma 3.5] allows us to also conclude that $\langle \bar {W},\mathcal {E}\rangle $ is $\alpha $ -iterable, whenever $\alpha $ is a countable ordinal, $\bar {W}$ is a transitive set, ${k}:{\bar {W}}\longrightarrow {W}$ is an elementary embedding and $\mathcal {E}\in \bar {W}$ with $k(\mathcal {E})=\{{F_n}~\vert ~{n<\omega }\}$ . Next, since we have $\delta =\sup _{n<\omega }\delta _n$ and $\tau \restriction (M\cup \{M\})={\mathrm {id}}_{M\cup \{M\}}$ , elementarity directly implies that I is the unique linear iteration of $\langle M,\{{\bar {U}_n}~\vert ~{n<\omega }\}\rangle $ of length $\delta $ with the property that

$$ \begin{align*}U^I_\alpha ~ = ~ i^I_{0,\alpha}(\bar{U}_{\min\{n<\omega\vert\alpha<\delta_n\}})\end{align*} $$

holds for all $\alpha <\delta $ . Finally, since (i) implies that

$$ \begin{align*}\psi(i^{\bar{I}}_{0,\infty}(\pi(\dot{x}))^{G_{p(a)}},i^{\bar{I}}_{0,\infty}(\pi(\dot{x}))^{G_{p(b)}},\kappa)\end{align*} $$

holds in ${\mathrm {H}}(\vartheta )$ , elementarity directly implies that

$$ \begin{align*}\psi(i^I_{0,\infty}(\pi(\dot{x}))^{G_{p(a)}},i^I_{0,\infty}(\pi(\dot{x}))^{G_{p(b)}},\delta)\end{align*} $$

holds in W. In combination, these observations show that W, $\delta $ , $\langle {\delta _n}~\vert ~{n<\omega }\rangle $ , $\langle {F_n}~\vert ~{n<\omega }\rangle $ and I witness that (ii) holds.

Now, assume that W, $\delta $ , $\langle {\delta _n}~\vert ~{n<\omega }\rangle $ , $\langle {F_n}~\vert ~{n<\omega }\rangle $ and I witness that (ii) holds. By [Reference Steel49, Lemma 3.6], our assumptions ensure that the pair $\langle W,\{{F_n}~\vert ~{n<\omega }\}\rangle $ is linearly iterable. Let $I_*$ denote the unique linear iteration of $\langle W,\{{F_n}~\vert ~{n<\omega }\}\rangle $ of length $\kappa $ with the property that

$$ \begin{align*}U^{I_*}_\alpha ~ = ~ i^{I_*}_{0,\alpha}(F_{\min\{n<\omega\vert\alpha<\kappa_n\}})\end{align*} $$

holds for all $\alpha <\kappa $ . Then we have $i^{I_*}_{0,\infty }(\delta _n)=\kappa _n$ for all $n<\omega $ and $i^{I_*}_{0,\infty }(\delta )=\kappa $ . Moreover, we know that

$$ \begin{align*}i^{I_*}_{0,\infty}\restriction(M[G_{p(a)},G_{p(b)}]\cup\{M\}) ~ = ~ {\mathrm{id}}_{M[G_{p(a)},G_{p(b)}]\cup\{M\}}.\end{align*} $$

This shows that $i^{I_*}_{0,\infty }(I)$ is a linear iteration of $\langle M,\{{\bar {U}_n}~\vert ~{n<\omega }\}\rangle $ of length $\kappa $ with the property that

$$ \begin{align*}U^{i^{I_*}_{0,\infty}(I)}_\alpha ~ = ~ i^{i^{I_*}_{0,\infty}(I)}_{0,\alpha}(\bar{U}_{\min\{n<\omega\vert\alpha<\kappa_n\}})\end{align*} $$

holds for all $\alpha <\kappa $ , and this implies that $i^{I_*}_{0,\infty }(I)=\bar {I}$ holds. In particular, it follows that

$$ \begin{align*}i^{I_*}_{0,\infty}(i^I_{0,\infty}(y)) ~ = ~ i^{\bar{I}}_{0,\infty}(y)\end{align*} $$

holds for all $y\in M$ . By our assumptions and the above observations, this shows that

$$ \begin{align*}\psi(i^{\bar{I}}_{0,\infty}(\pi(\dot{x}))^{G_{p(a)}},i^{\bar{I}}_{0,\infty}(\pi(\dot{x}))^{G_{p(b)}},\kappa)\end{align*} $$

holds in $M^{I_*}_{0,\infty }$ . Using $\Sigma _1$ -upwards absoluteness, we know that $\psi (x_{p(a)},x_{p(b)},\kappa )$ holds in ${\mathrm {V}}$ and this shows that (i) holds in this case.

Since the above claim shows that the relation $\blacktriangleleft $ is definable over ${\mathrm {H}}(\aleph _1)$ by a $\Sigma _2$ -formula with parameters, we can conclude that $\blacktriangleleft $ is a $\mathbf {\Sigma }^1_3$ -subset of ${}^\omega 2\times {}^\omega 2$ (see [Reference Jech25, Lemma 25.25]). This completes the proof of the theorem.

We end this section by proving the equiconsistency stated in Theorem 1.5. One direction is given by the following lemma that follows from arguments presented in the proof of Theorem 4.1.Footnote 6

Lemma 7.5. Assume that there is no inner model with infinitely many measurable cardinals. If $\kappa $ is a singular cardinal, then there exists an injection from $\kappa ^+$ into ${\mathcal {P}}({\kappa })$ that is definable by a $\Sigma _1$ -formula with parameters in ${\mathrm {H}}(\kappa )\cup \{\kappa \}$ .

Proof. As in the proof of Theorem 4.1 in Section 4, we know that $0^{\text {long}}$ does not exist and we let $U_{can}$ denote the canonical sequence of measures as in [Reference Koepke31]. We again set $U=U_{can}\restriction \kappa $ and ${\mathrm {K}}={\mathrm {K}}[U]$ . Then our assumptions imply that $U\in {\mathrm {H}}(\kappa )^{\mathrm {K}}$ and the results of [Reference Koepke31] show that ${\mathrm {K}}$ is an inner model of ${\mathrm {ZFC}}$ with a canonical well-ordering $<_{\mathrm {K}}$ . Since the domain of $U_{can}$ is finite, we can again combine [Reference Koepke31, Theorem 3.9], [Reference Koepke31, Theorem 3.19] and [Reference Koepke31, Theorem 3.23] to show that $\kappa ^+=(\kappa ^+)^{\mathrm {K}}$ .

Given $\kappa \leq \gamma <\kappa ^+$ , we let $y_\gamma $ denote the subset of $\kappa $ that canonically codes the $<_{\mathrm {K}}$ -least bijection between $\kappa $ and $\gamma $ . As in the proof of Theorem 4.1, we can now conclude that the unique injection ${\iota }:{\kappa ^+}\longrightarrow {{\mathcal {P}}({\kappa })}$ with $\iota \restriction \kappa ={\mathrm {id}}_\kappa $ and $\iota (\gamma )=y_\gamma $ for all $\kappa \leq \gamma <\kappa ^+$ can be defined by a $\Sigma _1$ -formula and the parameters $\kappa $ and U.

The next lemma is needed in the converse direction of our equiconsistency proof:

Lemma 7.6. Let U be a normal ultrafilter on a measurable cardinal $\delta $ , let $\alpha <\delta $ , let $\mathcal {E}$ be a set of normal ultrafilters on cardinals smaller than $\alpha $ and let I be a linear iteration of $\langle {\mathrm {V}},\mathcal {E}\rangle $ of length less than $\alpha $ . If $B\in i^I_{0,\infty }(U)$ , then there is $A\in U$ with $i^I_{0,\infty }(A)\subseteq B$ .

Proof. Using [Reference Kanamori28, Exercise 12], we find a function ${f}:{[\alpha ]^{{<}\omega }}\longrightarrow {U}$ with the property that $B\in {{\mathrm {ran}}(i^I_{0,\infty }(f))}$ . If we now define

$$ \begin{align*}A ~ = ~ \bigcap\{{f(a)}~\vert~{a\in[\alpha]^{{<}\omega}}\},\end{align*} $$

then A is an element of U with the desired properties.

In order to complete the proof of Theorem 1.5, we will now use diagonal Prikry forcing and a characterisation of generic sequences for this forcing due to Fuchs [Reference Fuchs14] to construct a model without $\Sigma _1$ -definable long well-orderings from an infinite sequence of measurable cardinals.

Proof of Theorem 1.5

Assume that $\vec {\kappa }=\langle {\kappa (n)}~\vert ~{n<\omega }\rangle $ is a strictly increasing sequence of measurable cardinals with limit $\kappa $ . Pick a sequence $\vec {U}=\langle {U(n)}~\vert ~{n<\omega }\rangle $ with the property that $U(n)$ is a normal ultrafilter on $\kappa (n)$ for each $n<\omega $ . Let ${\mathbb {P}}_{\vec {U}}$ denote the diagonal Prikry forcing associated to the sequence $\vec {U}$ (see [Reference Gitik15, Section 1.3]), that is, ${\mathbb {P}}_{\vec {U}}$ is the partial order defined by the following clauses:

  • Conditions in ${\mathbb {P}}_{\vec {U}}$ are sequences $p=\langle {p_n}~\vert ~{n<\omega }\rangle $ with the property that there exists a natural number $l_p$ such that $p_n\in \kappa (n)$ for all $n<l_p$ and $p_n\in U(n)$ for all $l_p\leq n<\omega $ .

  • Given conditions p and q in ${\mathbb {P}}_{\vec {U}}$ , we have $p\leq _{{\mathbb {P}}_{\vec {U}}}q$ if and only if $l_q\leq l_p$ , $p_n=q_n$ for all $n<l_q$ , $q_n\in p_n$ for all $l_q\leq n<l_p$ and $p_n\subseteq q_n$ for all $l_p\leq n<\omega $ .

By [Reference Gitik15, Lemma 1.35], forcing with ${\mathbb {P}}_{\vec {U}}$ does not add bounded subsets of $\kappa $ .

Given a filter G on ${\mathbb {P}}_{\vec {U}}$ , we let $c_G$ denote the unique function with domain $\sup _{p\in G}l_p\leq \omega $ and $c_G(n)=p_n$ for all $p\in G$ and $n<l_p$ . In the other direction, given a sequence c contained in the set $\prod \vec {\kappa }$ of all functions $d\in {}^\omega \kappa $ with $d(n)<\kappa (n)$ for all $n<\omega $ , we let $G_c$ denote the set of all conditions p in ${\mathbb {P}}_{\vec {U}}$ with $p_n=c(n)$ for all $n<l_p$ and $c(n)\in p_n$ for all $l_p\leq n<\omega $ . It is easy to see that $G_c$ is a filter on ${\mathbb {P}}_{\vec {U}}$ with $c_{G_c}=c$ in this situation. Given an inner model M that contains $\vec {U}$ and $c\in \prod \vec {\kappa }$ , we say that c is $\vec {U}$ -generic over M if $G_c$ is ${\mathbb {P}}_{\vec {U}}$ -generic over M. The results of [Reference Fuchs14] then show that a sequence $c\in \prod \vec {\kappa }$ is $\vec {U}$ -generic over an inner model M if and only if $\{{n<\omega }~\vert ~{c(n)\in A_n}\}$ is a cofinite subset of $\omega $ for every sequence $\langle {A_n\in U(n)}~\vert ~{n<\omega }\rangle $ in M. Using [Reference Grigorieff19, Theorem 3.5.1], this characterization can be used to show that the Boolean completion of ${\mathbb {P}}_{\vec {U}}$ is weakly homogeneous and therefore every statement in the forcing language of ${\mathbb {P}}_{\vec {U}}$ that uses only ground model elements as parameters is decided by .

Now, let G be ${\mathbb {P}}_{\vec {U}}$ -generic over ${\mathrm {V}}$ and assume that, in ${\mathrm {V}}[G]$ , there exists a well-ordering $\lhd $ of a subset D of ${\mathcal {P}}({\kappa })$ of cardinality greater than $\kappa $ that can be defined by a $\Sigma _1$ -formula $\varphi (v_0,\ldots ,v_3)$ , a parameter $z\in {\mathrm {H}}(\kappa )$ and the parameter $\kappa $ . Then we can find a $\Sigma _1$ -formula $\psi (v_0,v_1,v_2)$ with the property that, in ${\mathrm {V}}[G]$ , the set D can be defined by the formula $\psi $ and the parameters $\kappa $ and z. In this situation, we know that $z\in {\mathrm {V}}$ and the homogeneity properties of ${\mathbb {P}}_{\vec {U}}$ imply that $D\subseteq {\mathrm {V}}$ , because, given $y\in D$ , we know that y is the $\alpha $ -th element of the well-order $\langle D,\lhd \rangle $ for some ordinal $\alpha $ and therefore the set $\{y\}$ is definable in ${\mathrm {V}}[G]$ by a formula with parameters in ${\mathrm {V}}$ . In addition, we know that

(9)

Let $\mathcal {E}$ denote the set of all normal ultrafilters on cardinals smaller than $\kappa $ in ${\mathrm {V}}$ . Apply Lemma 2.1 to $\kappa $ , z and D in ${\mathrm {V}}$ to obtain an element $x_*$ of D, a system $\langle {\nu _s}~\vert ~{s\in {}^{{<}\omega }\kappa }\rangle $ of inaccessible cardinals smaller than $\kappa $ , a system $\langle {\kappa _s}~\vert ~{s\in {}^{{<}\omega }\kappa }\rangle $ of measurable cardinals smaller than $\kappa $ , a system $\langle {U_s}~\vert ~{s\in {}^{{<}\omega }\kappa }\rangle $ of elements of $\mathcal {E}$ and a system $\langle {I_s}~\vert ~{s\in {}^{{<}\omega }\kappa }\rangle $ of linear iterations of $\langle {\mathrm {V}},\mathcal {E}\rangle $ possessing the properties listed in the lemma. Next, for each $c\in ({}^\omega \kappa )^{{\mathrm {V}}[G]}$ , let $I_c$ denote the unique iteration of $\langle {\mathrm {V}},\{{U_{c\restriction n}}~\vert ~{n<\omega }\}\rangle $ of length $\sup _{n<\omega }{\mathrm {lh}}({I_{c\restriction n}}){}$ in ${\mathrm {V}}[G]$ with $U^{I_c}_{\alpha }=U^{c\restriction n}_\alpha $ for all $n<\omega $ and $\alpha <{\mathrm {lh}}({I_{c\restriction n}}){}$ . Then $M^{I_c}_{{\mathrm {lh}}({I_{c\restriction n}}){}}=M^{I_{c\restriction n}}_\infty $ and $i^{I_c}_{0,{\mathrm {lh}}({I_{c\restriction n}}){}}=i^{I_{c\restriction n}}_{0,\infty }$ for all $c\in ({}^\omega \kappa )^{{\mathrm {V}}[G]}$ and $n<\omega $ with ${\mathrm {lh}}({I_{c\restriction n}}){}<{\mathrm {lh}}({I_c}){}$ . Moreover, we have $M^{I_c}_\infty =M^{I_{c\restriction n}}_\infty $ and $i^{I_c}_{0,\infty }=i^{I_{c\restriction n}}_{0,\infty }$ for all $c\in ({}^\omega \kappa )^{{\mathrm {V}}[G]}$ and $n<\omega $ with ${\mathrm {lh}}({I_{c\restriction n}}){}={\mathrm {lh}}({I_c}){}$ . Given $c\in ({}^\omega \kappa )^{{\mathrm {V}}[G]}$ , we define $M_c=M^{I_c}_\infty $ , $\bar {c}=i^{I_c}_{0,\infty }\circ c_G$ and $x_c=i^{I_c}_{0,\infty }(x_*)$ . In this situation, Lemma 2.2 shows that $M_c$ is well-founded for all $c\in ({}^\omega \kappa )^{{\mathrm {V}}[G]}$ .

Claim. If $c\in ({}^\omega \kappa )^{{\mathrm {V}}[G]}$ , then $\bar {c}$ is $i^{I_c}_{0,\infty }(\vec {U})$ -generic over $M_c$ .

Proof of the Claim

Suppose that $i^{I_c}_{0,\infty }(\vec {U})=\langle {U^{\prime \prime }(n)}~\vert ~{n<\omega }\rangle $ and fix a sequence $\vec {C}=\langle {C_n\in U^{\prime \prime }(n)}~\vert ~{n<\omega }\rangle $ in $M_c$ . Since $I_c$ is a linear iteration of length at most $\kappa $ , we can find $n_0<\omega $ and a sequence $\vec {B}=\langle {B_n}~\vert ~{n<\omega }\rangle $ in $M^{I_{c\restriction n_0}}_\infty $ such that either ${\mathrm {lh}}({I_c}){}={\mathrm {lh}}({I_{c\restriction n_0}}){}$ and $\vec {B}=\vec {C}$ , or ${\mathrm {lh}}({I_c}){}>{\mathrm {lh}}({I_{c\restriction n_0}}){}$ and $i^{I_c}_{{\mathrm {lh}}({I_{c\restriction n_0}}){},\infty }(\vec {B})=\vec {C}$ . Now, pick $n_1<\omega $ with $\kappa (n)>\kappa _{c\restriction n_0}$ for all $n_1\leq n<\omega $ . In this situation, the conclusions of Lemma 2.1 ensure that we can apply Lemma 7.6 to find a sequence $\langle {A_n\in U(n)}~\vert ~{n<\omega }\rangle $ with $i^{I_c}_{0,{\mathrm {lh}}({I_{c\restriction n_0}}){}}(A_n)\subseteq B_n$ for all $n_1\leq n<\omega $ . Since $c_G$ is $\vec {U}$ -generic over ${\mathrm {V}}$ , we find $n_1\leq n_2<\omega $ with $c_G(n)\in A_n$ for all $n_2\leq n<\omega $ . But this shows that $\bar {c}(n)\in C_n$ holds for all $n_2\leq n<\omega $ . Using the characterization of generic sequences provided by [Reference Fuchs14], these computations prove the statement of the claim.

Claim. If $c\in ({}^\omega \kappa )^{{\mathrm {V}}[G]}$ , then $x_c\in D$ .

Proof of the claim

By the previous claim, there exists a filter H on $i^{I_c}_{0,\infty }({\mathbb {P}}_{\vec {U}})$ in ${\mathrm {V}}[G]$ that is generic over $M_c$ . Since Lemma 2.2 shows that $i^{I_c}_{0,\infty }(\kappa )=\kappa $ and $i^{I_c}_{0,\infty }(z)=z$ , we can use Equation (9) to show that

holds in $M_c$ . This shows that $\psi (x_c,z,\kappa )$ holds in $M_c[H]$ and $\Sigma _1$ -upwards absoluteness implies that this statement also holds in ${\mathrm {V}}[G]$ .

By Lemma 2.2, our definitions ensure that the map

$$ \begin{align*}{\iota}:{({}^\omega\kappa)^{{\mathrm{V}}[G]}}\longrightarrow{D};~{c}\longmapsto{x_c}\end{align*} $$

is an injection that is definable in ${\mathrm {V}}[G]$ from parameters contained in the ground model ${\mathrm {V}}$ . Since $\iota (c_G)\in D\subseteq {\mathrm {V}}$ , this shows that, in ${\mathrm {V}}[G]$ , the set $\{c_G\}$ is definable from parameters in ${\mathrm {V}}$ . Using the homogeneity properties of ${\mathbb {P}}_{\vec {U}}$ in ${\mathrm {V}}$ , we can now conclude that $c_G$ is an element of ${\mathrm {V}}$ , a contradiction.

8 Long well-orderings in ${\mathcal {P}}({\omega _1})$

We now show that both strong large cardinal assumptions and strong forcing axioms cause analogues of the above results on the definability of long well-orders to hold for $\omega _1$ . In the following, we combine well-known consequences of the axiom of determinacy ${\mathrm {AD}}$ with Woodin’s analysis of ${\mathbb {P}}_{max}$ -extensions of determinacy models (see [Reference Larson32] and [Reference Woodin52]).

Lemma 8.1 ( ${\mathrm {ZF}}$ )

Let $\kappa $ be an infinite cardinal. If there is an injection from $\kappa ^+$ into ${\mathcal {P}}({\kappa })$ , then every ${<}\kappa ^+$ -complete ultrafilter on $\kappa ^+$ is principal.

Proof. Let U be a ${<}\kappa ^+$ -complete ultrafilter on $\kappa ^+$ , and let ${\iota }:{\kappa ^+}\longrightarrow {{\mathcal {P}}({\kappa })}$ be an injection. Given $\alpha <\kappa $ , set

$$ \begin{align*}B_\alpha ~ = ~ \{{\gamma<\kappa^+}~\vert~{\alpha\in\iota(\gamma)}\}.\end{align*} $$

Since U is an ultrafilter, there is $A\subseteq \kappa $ such that

$$ \begin{align*}\alpha\in A ~ \Longleftrightarrow ~ B_\alpha\in U\end{align*} $$

holds for all $\alpha <\kappa $ . The ${<}\kappa ^+$ -completeness of U then ensures that the set

$$ \begin{align*}B ~ = ~ \bigcap\{{B_\alpha}~\vert~{\alpha\in A}\} ~ \cap ~ \bigcap\{{\kappa^+\setminus B_\alpha}~\vert~{\alpha\in\kappa\setminus A}\}\end{align*} $$

is an element of U. We now know that $\iota [B]=\{A\}$ and hence the injectivity of $\iota $ implies that B is a singleton.

Corollary 8.2 ( ${\mathrm {ZF}}+{\mathrm {DC}}+{\mathrm {AD}}$ )

There is no injection from $\omega _2$ into ${\mathcal {P}}({\omega _1})$ .

Proof. By results of Kleinberg and Martin–Paris (see [Reference Kechris29, Section 13]), the restriction of the closed unbounded filter on $\omega _2$ to the set of all ordinals of countable cofinality is a ${<}\omega _2$ -complete, nonprincipal ultrafilter on $\omega _2$ .

The following lemma will allow us to use the theory developed in [Reference Woodin52] to prove Theorem 1.6.(i).

Lemma 8.3. Assume that ${\mathrm {AD}}$ holds in ${\mathrm {L}}({\mathbb {R}})$ and ${\mathrm {V}}$ is a ${\mathbb {P}}_{max}$ -generic extension of ${\mathrm {L}}({\mathbb {R}})$ . Then no well-ordering of a subset of ${\mathcal {P}}({\omega _1})$ of cardinality greater than $\aleph _1$ is contained in ${\mathrm {OD}}({\mathbb {R}})$ .

Proof. Assume, towards a contradiction, that there exists a subset D of ${\mathcal {P}}({\omega _1})$ of cardinality greater than $\aleph _1$ and a well-ordering $\lhd $ of D that is contained in ${\mathrm {OD}}({\mathbb {R}})$ . Then the fact that ${\mathbb {P}}_{max}$ is countably closed and homogeneous in ${\mathrm {L}}({\mathbb {R}})$ (see [Reference Woodin52, Lemma 4.38] and [Reference Woodin52, Lemma 4.43]) directly implies that D and $\lhd $ are both contained in ${\mathrm {L}}({\mathbb {R}})$ . But this shows that ${\mathrm {L}}({\mathbb {R}})$ is a model of ${\mathrm {ZF}}+{\mathrm {DC}}+{\mathrm {AD}}$ that contains an injection from $\omega _2$ into ${\mathcal {P}}({\omega _1})$ , contradicting Corollary 8.2.

Proof of Theorem 1.6.(i)

Let $\lhd $ be a well-ordering of a subset of ${\mathcal {P}}({\omega _1})$ of cardinality greater than $\aleph _1$ that is definable by a $\Sigma _1$ -formula $\varphi (v_0,\ldots ,v_3)$ and parameters $\omega _1$ and $z\in {\mathrm {H}}(\aleph _1)$ .

First, assume that Woodin’s axiom $(*)$ holds, that is, ${\mathrm {AD}}$ holds in ${\mathrm {L}}({\mathbb {R}})$ and ${\mathrm {L}}({\mathcal {P}}({\omega _1}))$ is a ${\mathbb {P}}_{max}$ -generic extension of ${\mathrm {L}}({\mathbb {R}})$ . We now know that $\lhd $ and its domain are both elements of ${\mathrm {OD}}({\mathbb {R}})^{{\mathrm {L}}({\mathcal {P}}({\omega _1}))}$ because $\Sigma _1$ -statements with parameters in ${\mathrm {H}}(\aleph _2)$ are absolute between ${\mathrm {L}}({\mathcal {P}}({\omega _1}))$ and ${\mathrm {V}}$ . Since the domain of $\lhd $ has cardinality greater than $\aleph _1$ in ${\mathrm {L}}({\mathcal {P}}({\omega _1}))$ , we can now use Lemma 8.3 to derive a contradiction.

Now, assume that there is a measurable cardinal above infinitely many Woodin cardinals. Then ${\mathrm {AD}}$ holds in ${\mathrm {L}}({\mathbb {R}})$ . Note that the formula $\varphi $ and the parameters $\omega _1$ and z also define $\lhd $ in ${\mathrm {H}}(\aleph _2)$ , and this statement can be formulated by a $\Pi _2$ -formula with parameter z in the structure $\langle {\mathrm {H}}(\aleph _2),\in \rangle $ Let G be ${\mathbb {P}}_{max}$ -generic over ${\mathrm {L}}({\mathbb {R}})$ . Then the $\Pi _2$ -maximality of ${\mathrm {L}}({\mathbb {R}})[G]$ (see [Reference Larson32, Theorem 7.3]) implies that the formula $\varphi $ and the parameters $\omega _1$ and z also define a well-ordering of a subset of ${\mathcal {P}}({\omega _1})$ of cardinality greater than $\aleph _1$ in the structure $\langle {\mathrm {H}}(\aleph _2)^{{\mathrm {L}}({\mathbb {R}})[G]},\in \rangle $ . In particular, such a well-ordering is contained in ${\mathrm {OD}}({\mathbb {R}})^{{\mathrm {L}}({\mathbb {R}})[G]}$ , again contradicting Lemma 8.3.

9 Almost disjoint families in ${\mathcal {P}}({\omega _1})$

Following the structure of the arguments in the previous section, we now show that both large cardinals and forcing axioms imply that large almost disjoint families of subsets of $\omega _1$ are not simply definable. The first step in these proofs is the following unpublished result of William Chan, Stephen Jackson and Nam Trang whose proof we include with their permission. This result is an application of their work on the validity of the Kurepa hypothesis in determinacy models and continues a line of groundbreaking results on definable combinatorics at $\omega _1$ (see, for example, [Reference Chan and Jackson6], [Reference Chan and Jackson7], [Reference Chan, Jackson and Trang8] and [Reference Chan, Jackson and Trang9]).

Theorem 9.1 (Chan–Jackson–Trang, ${\mathrm {ZF}}+{\mathrm {DC}}_{\mathbb {R}}+{\mathrm {AD}}^+$ )

Assume that ${\mathrm {V}}={\mathrm {L}}({\mathcal {P}}({{\mathbb {R}}}))$ holds. If A is a set of cofinal subsets of $\omega _1$ , then one of the following statements holds:

  1. (i) A can be well-ordered and its cardinality is less than or equal to $\aleph _1$ .

  2. (ii) There are distinct $x,y\in A$ such that $x\cap y$ is unbounded in $\omega _1$ .

The proof of this result makes use of the following topological fact:

Proposition 9.2 ( ${\mathrm {ZF}}+{\mathrm {DC}}$ )

If X is a Polish space and $\langle {A_\alpha }~\vert ~{\alpha <\omega _1}\rangle $ is a sequence of pairwise disjoint nonmeager subsets of X, then there is an $\alpha <\omega _1$ such that the subset $A_\alpha $ does not have the property of Baire.

Proof. Assume, towards a contradiction, that $A_\alpha $ has the property of Baire for all $\alpha <\omega _1$ . Given $\alpha <\omega _1$ , our assumption implies that there is a nonempty open set U with the property that $U\setminus A_\alpha $ is meager. Hence, there is a sequence $\langle {N_\alpha }~\vert ~{\alpha <\omega _1}\rangle $ of nonempty basic open subsets of X such that $N_\alpha \setminus A_\alpha $ is meager. Pick $\alpha <\beta <\omega _1$ with $N_\alpha =N_\beta $ . Then $N_\alpha \setminus (A_\alpha \cap A_\beta )=(N_\alpha \setminus A_\alpha )\cup (N_\alpha \setminus A_\beta )$ is meager and hence $A_\alpha \cap A_\beta \neq \emptyset $ , a contradiction

Proof of Theorem 9.1

Assume, towards a contradiction, that both conclusions fail.

Claim. The set A cannot be well-ordered.

Proof of the claim

Assume, towards a contradiction, that A can be well-ordered. Then our assumptions imply that it has cardinality greater than $\aleph _1$ and hence we obtain an injection of $\omega _2$ into ${\mathcal {P}}({\omega _1})$ . But this yields a contradiction, because the assumption of Corollary 8.2 are satisfied in our setting.

By combining the above claim with [Reference Caicedo and Ketchersid4, Theorem 1.4], we now obtain an injection ${\iota }:{{\mathbb {R}}}\longrightarrow {A}$ . Our assumptions then ensure that the function

$$ \begin{align*} {c}:{[{\mathbb{R}}]^2}\longrightarrow{\omega_1};~{\{x,y\}} \longmapsto{\min\{{\alpha<\omega_1}~\vert~{\iota(x)\cap\iota(y)\subseteq\alpha}\}} \end{align*} $$

is well-defined. Given $\alpha <\omega _1$ , set $E_\alpha =c^{{-}1}\{\alpha \}\subseteq {\mathbb {R}}\times {\mathbb {R}}$ . Then $\bigcup \{{E_\alpha }~\vert ~{\alpha <\omega _1}\}$ is dense open in ${\mathbb {R}}\times {\mathbb {R}}$ .

Claim. There is a $\lambda <\omega _1$ such that the set $\bigcup \{{E_\alpha }~\vert ~{\alpha <\lambda }\}$ is comeager in ${\mathbb {R}}\times {\mathbb {R}}$ .

Proof of the claim

Assume that there is no $\lambda <\omega _1$ with the property that the set $\bigcup \{{E_\alpha }~\vert ~{\alpha <\lambda }\}$ is comeager. Since the ideal of meager subsets of ${\mathbb {R}}\times {\mathbb {R}}$ is closed under well-ordered unions in our setting, our assumption yields a strictly increasing function ${f}:{\omega _1}\longrightarrow {\omega _1}$ with the property that $E_{f(\alpha )}$ is a nonmeager subset of ${\mathbb {R}}\times {\mathbb {R}}$ for all $\alpha <\omega _1$ . In this situation, the sequence $\langle {E_{f(\alpha )}}~\vert ~{\alpha <\omega _1}\rangle $ consists of pairwise disjoint nonmeager subsets of ${\mathbb {R}}\times {\mathbb {R}}$ and, since we assume that ${\mathrm {AD}}$ holds, all of these sets possess the property of Baire. This contradicts Proposition 9.2.

By a classical result of Mycielski (see [Reference Kechris30, Theorem 19.1]), we can now find an injection ${e}:{{\mathbb {R}}}\longrightarrow {{\mathbb {R}}}$ such that for all $x,y\in {\mathbb {R}}$ with $x\neq y$ , there is an $\alpha <\lambda $ with $\langle e(x),e(y)\rangle \in E_\alpha $ . In this situation, we know that

$$ \begin{align*} (\iota\circ e)(x) ~ \cap ~ (\iota\circ e)(y) ~ \subseteq ~ \lambda \end{align*} $$

holds for all $x,y\in {\mathbb {R}}$ with $x\neq y$ . In particular, since the set A consists of unbounded subsets of $\omega _1$ , we know that the map

$$ \begin{align*} {i}:{{\mathbb{R}}}\longrightarrow{\omega_1};~{x}\longmapsto{\min((\iota\circ e)(x)\setminus\lambda)} \end{align*} $$

is an injection. But this shows that the reals can be well-ordered, contradicting our assumptions.

In order to transfer the above result to models of the form ${\mathrm {HOD}}({\mathbb {R}})$ of ${\mathbb {P}}_{max}$ -extensions, we make use of another axiom introduced by Woodin, called $\left ({*\atop *}\right )$ (see [Reference Woodin52, Definition 5.69]).

Lemma 9.3. Assume that ${\mathrm {AD}}$ holds in ${\mathrm {L}}({\mathbb {R}})$ and ${\mathrm {V}}$ is a ${\mathbb {P}}_{max}$ -generic extension of ${\mathrm {L}}({\mathbb {R}})$ . If $A\in {\mathrm {OD}}({\mathbb {R}})$ is a set of cardinality greater than $\aleph _1$ that consists of unbounded subsets of $\omega _1$ , then there are distinct $x,y\in A$ with the property that $x\cap y$ is unbounded in $\omega _1$ .

Proof. Assume, towards a contradiction, that the above conclusion fails.

Claim. $A\subseteq {\mathrm {L}}({\mathbb {R}})$ .

Proof of the claim

Assume that $A\nsubseteq {\mathrm {L}}({\mathbb {R}})$ . Since ${\mathrm {V}}={\mathrm {L}}({\mathcal {P}}({\omega _1}))$ holds and [Reference Woodin52, Corollary 5.83] shows that our assumptions imply that $\left ({*\atop *}\right )$ holds, we can apply [Reference Woodin52, Theorem 5.84] to find an unbounded subset U of $\omega _1$ and a function ${\pi }:{{}^{{<}\omega _1}2}\longrightarrow {[\omega _1]^\omega }$ such that the following statements hold:

  1. (i) If $s,t\in {}^{{<}\omega _1}2$ with $s\subseteq t$ , then $\pi (s)\subseteq \pi (t)$ and $\pi (s)\cap \alpha =\pi (t)\cap \alpha $ for all $\alpha \in \pi (s)$ .

  2. (ii) Given $s\in {}^{{<}\omega _1}2$ and $\alpha \in {{\mathrm {dom}}(s)}\cap U$ , we have $\alpha \in \pi (s)$ if and only if $s(\alpha )=1$ .

  3. (iii) If $x\in {}^{\omega _1}2$ , then $\bar {\pi }(x)=\bigcup \{{\pi (x\restriction \alpha )}~\vert ~{\alpha <\omega _1}\}\in A$ .

Pick $x,y\in {}^{\omega _1}2$ such that x has constant value $1$ and y is the characteristic function of $U\setminus \{\min (U)\}$ . Since $\bar {\pi }(x),\bar {\pi }(y)\in A$ and $U\setminus \{\min (U)\}\subseteq \bar {\pi }(x)\cap \bar {\pi }(y)$ , we know that $\bar {\pi }(x)=\bar {\pi }(y)$ as $\bar {\pi }(x)\cap \bar {\pi }(y)$ is unbounded in $\omega _1$ . But $\min (U)\in \bar {\pi }(x)\setminus \bar {\pi }(y)$ , a contradiction.

Claim. $A\in {\mathrm {L}}({\mathbb {R}})$ .

Proof of the claim

Using the homogeneity of ${\mathbb {P}}_{max}$ in ${\mathrm {L}}({\mathbb {R}})$ , this statement follows directly from the previous claim and the fact that the set A is contained in the class ${\mathrm {OD}}({\mathbb {R}})$ .

Since [Reference Caicedo and Ketchersid4, Corollary 2.16] shows that ${\mathrm {L}}({\mathbb {R}})$ is a model of ${\mathrm {DC}}+{\mathrm {AD}}^+$ , we can use Theorem 9.1 in ${\mathrm {L}}({\mathbb {R}})$ to conclude that, in ${\mathrm {L}}({\mathbb {R}})$ , the set A can be well-ordered and its cardinality is smaller than or equal to $\aleph _1$ . But this shows that the cardinality of A in ${\mathrm {V}}$ is at most $\aleph _1$ , a contradiction.

Proof of Theorem 1.6.(ii)

Let A be a set of cardinality greater than $\aleph _1$ that consists of unbounded subsets of $\omega _1$ and is definable by a $\Sigma _1$ -formula $\varphi (v_0,v_1,v_2)$ and parameters $\omega _1$ and $z\in {\mathrm {H}}(\aleph _1)$ .

First, assume that Woodin’s axiom $(*)$ holds. Then the $\Sigma _1$ -reflection principle implies that the formula $\varphi $ and the parameters $\omega _1$ and z also define the set A in ${\mathrm {L}}({\mathcal {P}}({\omega _1}))$ . But this shows that $A\in {\mathrm {OD}}({\mathbb {R}})^{{\mathrm {L}}({\mathcal {P}}({\omega _1}))}$ and, since ${\mathrm {AD}}$ holds in ${\mathrm {L}}({\mathbb {R}})$ and ${\mathrm {L}}({\mathcal {P}}({\omega _1}))$ is a ${\mathbb {P}}_{max}$ -generic extension of ${\mathrm {L}}({\mathbb {R}})$ , we can now apply Lemma 9.3 in ${\mathrm {L}}({\mathcal {P}}({\omega _1}))$ to find distinct $x,y\in A$ with $x\cap y$ unbounded in $\omega _1$ .

Next, assume that there is a measurable cardinal above infinitely many Woodin cardinals. Then ${\mathrm {AD}}$ holds in ${\mathrm {L}}({\mathbb {R}})$ . Note that the formula $\varphi $ and the parameters $\omega _1$ and z also define A in the structure $\langle {\mathrm {H}}(\omega _2),\in \rangle $ . Assume, towards a contradiction, that $x\cap y$ is bounded in $\omega _1$ for all distinct $x,y\in A$ . Note that, in $\langle {\mathrm {H}}(\omega _2),\in \rangle $ , the statement that $\varphi $ , $\omega _1$ and z define a set of cardinality greater than $\aleph _1$ that consists of unbounded subsets of $\omega _1$ whose pairwise intersections are countable can be expressed by a $\Pi _2$ -formula with parameter z. Let G be ${\mathbb {P}}_{max}$ -generic over ${\mathrm {L}}({\mathbb {R}})$ . Then the $\Pi _2$ -maximality of ${\mathrm {L}}({\mathbb {R}})[G]$ implies that, in the structure $\langle {\mathrm {H}}(\aleph _2)^{{\mathrm {L}}({\mathbb {R}})[G]},\in \rangle $ , the formula $\varphi $ and the parameters $\omega _1$ and z define a set of cardinality greater than $\aleph _1$ that consists of unbounded subsets of $\omega _1$ whose pairwise intersections are countable. In particular, such a subset of ${\mathcal {P}}({\omega _1})$ exists in ${\mathrm {OD}}({\mathbb {R}})^{{\mathrm {L}}({\mathbb {R}})[G]}$ , contradicting Lemma 9.3.

10 Concluding remarks and open questions

In the following, we discuss several questions raised by the above results, starting with questions about the optimality of the assumption of Theorem 1.1. By Theorem 1.2, the consistency strength of this assumption is optimal in the case of singular cardinals. In contrast, results of Schlicht in [Reference Schlicht44] show that, if $\kappa $ is an uncountable regular cardinal, $\theta>\kappa $ is inaccessible and G is ${\mathrm {Col}}({\kappa },{{<}\theta })$ -generic over ${\mathrm {V}}$ , then, in ${\mathrm {V}}[G]$ , every subset of $\kappa $ in ${\mathrm {OD}}({}^\kappa {\mathrm {On}})$ either has cardinality $\kappa $ or contains a closed subset homeomorphic to ${}^\kappa 2$ . In particular, if $\kappa $ is not weakly compact in ${\mathrm {V}}$ , then, in ${\mathrm {V}}[G]$ , the cardinal $\kappa $ is not weakly compact, the spaces ${}^\kappa 2$ and ${}^\kappa \kappa $ are homeomorphic (see, for example, [Reference Lücke, Ros and Schlicht35, Corollary 2.3]) and for every subset D of ${\mathcal {P}}({\kappa })$ of cardinality greater than $\kappa $ that is definable by a $\Sigma _1$ -formula with parameters in ${\mathrm {H}}(\kappa )\cup \{\kappa \}$ , there is a perfect embedding ${\iota }:{{}^\kappa \kappa }\longrightarrow {{\mathcal {P}}({\kappa })}$ with ${{\mathrm {ran}}(\iota )}\subseteq D$ . This shows that our question is only interesting when we also assume that the given cardinal $\kappa $ possesses certain large cardinal properties.Footnote 7

Question 10.1. Assume that $\kappa $ is a weakly compact cardinal with the property that for every subset D of ${\mathcal {P}}({\kappa })$ of cardinality greater than $\kappa $ that is definable by a $\Sigma _1$ -formula with parameters in ${\mathrm {H}}(\kappa )\cup \{\kappa \}$ , there is a perfect embedding ${\iota }:{{}^\kappa \kappa }\longrightarrow {{\mathcal {P}}({\kappa })}$ with ${{\mathrm {ran}}(\iota )}\subseteq D$ . Is there an inner model that contains a weakly compact limit of measurable cardinals?

In contrast to the singular case, we may also ask whether the conclusion of Theorem 1.1 can be established from a sequences of measurable cardinals that are bounded in a regular cardinal, but whose order type is equal their minimum.

Question 10.2. Does the assumption of Question 1.1 imply the existence of a set-sized transitive model of ${\mathrm {ZFC}}$ containing a weakly compact cardinal $\delta $ and a sequence S of measurable cardinals less than $\delta $ of order-type $\min (S)$ ?

We now consider the possibility to strengthen Theorem 1.4. Since the existence of infinitely many measurable cardinals is compatible with the existence of a $\mathbf {\Sigma }^1_3$ -well-ordering of the reals (see [Reference Mitchell41, Theorem 3.6]), it is natural to ask whether the assumption of this theorem is actually consistent. The model constructed in [Reference Mitchell41, Section 1] should be the natural candidate to look for an affirmative answer to the following question.

Question 10.3. Is it consistent that there exists a limit $\kappa $ of $\omega $ -many measurable cardinals and a well-ordering of a subset of ${\mathcal {P}}({\kappa })$ of cardinality greater than $\kappa $ that is definable by a $\Sigma _1$ -formula with parameter  $\kappa $ ?

In addition, the equiconsistency given by Theorem 1.5 motivates the question whether such implications can be extended to cardinals of higher cofinalities.

Question 10.4. Is it consistent that there exists a limit of measurable cardinals $\kappa $ and a well-ordering of a subset of ${\mathcal {P}}({\kappa })$ of cardinality greater than $\kappa $ that is definable by a $\Sigma _1$ -formula with parameters in ${\mathrm {H}}(\kappa )\cup \{\kappa \}$ ?

Next, we consider simply definable almost disjoint families. The statements of Theorem 1.3 and Theorem 1.6.(ii) are motivated by a classical result of Mathias in [Reference Mathias39] showing that no maximal almost disjoint family in ${\mathcal {P}}({\omega })$ is analytic. In contrast, Miller [Reference Miller40] showed that the existence of coanalytic maximal disjoint families in ${\mathcal {P}}({\omega })$ is consistent. This motivates the following questions:

Question 10.5.

  1. (i) Does the existence of sufficiently strong large cardinals imply that no almost disjoint family of cardinality greater than $\aleph _1$ in ${\mathcal {P}}({\omega _1})$ is definable by a $\Pi _1$ -formula with parameters in ${\mathrm {H}}(\aleph _1)\cup \{\omega _1\}$ ?

  2. (ii) Do sufficiently strong large cardinal properties of a cardinal $\kappa $ imply that no almost disjoint family of cardinality greater than $\kappa $ in ${\mathcal {P}}({\kappa })$ is definable by a $\Pi _1$ -formula with parameters in ${\mathrm {H}}(\kappa )\cup \{\kappa \}$ ?

It should be noted that our proof of Theorem 1.6.(ii) in Section 9 already shows that Woodin’s axiom $(*)$ (and therefore strong forcing axioms, see [Reference Asperó and Schindler2]) implies that no almost disjoint family of cardinality greater than $\aleph _1$ in ${\mathcal {P}}({\omega _1})$ is definable in the structure $\langle {\mathrm {H}}(\aleph _2),\in \rangle $ by a formula with parameters in ${\mathrm {H}}(\aleph _1)\cup \omega _2$ , because all such families are elements of ${\mathrm {OD}}({\mathbb {R}})^{{\mathrm {L}}({\mathcal {P}}({\omega _1}))}$ . In particular, no such family is definable by a $\Pi _1$ -formula with parameters in ${\mathrm {H}}(\aleph _1)\cup \{\omega _1\}$ in this setting.

Finally, we consider the questions whether analogues of the above results hold for other types of uncountable cardinals. The following observation uses ideas from [Reference Lücke34, Section 6] and [Reference Lücke, Schindler and Schlicht37, Section 5] to show that the results of Section 9 cannot be generalized from $\omega _1$ to $\omega _2$ . Moreover, it shows that forcing axioms outright imply the $\Sigma _1$ -definability of pathological objects at $\omega _2$ . Remember that a sequence $\langle {C_\alpha }~\vert ~{\alpha \in {\mathrm {Lim}}\cap \omega _1}\rangle $ is a C-sequence if $C_\alpha $ is an unbounded subset of $\alpha $ of order-type $\omega $ for every countable limit ordinal $\alpha $ .

Proposition 10.6.

  1. (i) If the Bounded proper forcing axiom ${\mathrm {BPFA}}$ holds and $\vec {C}$ is a C-sequence, then there exists an almost disjoint family of cardinality $2^{\aleph _2}$ in ${\mathcal {P}}({\omega _2})$ that is definable by a $\Sigma _1$ -formula with parameters $\omega _2$ and $\vec {C}$ .

  2. (ii) If $\omega _1=\omega _1^{\mathrm {L}}$ and ${\mathrm {BPFA}}$ holds,Footnote 8 then there exists an almost disjoint family of cardinality $2^{\aleph _2}$ in ${\mathcal {P}}({\omega _2})$ that is definable by a $\Sigma _1$ -formula with parameter $\omega _2$ .

  3. (iii) If there is a supercompact cardinal, then, in a generic extension of the ground model, the proper forcing axiom ${\mathrm {PFA}}$ holds and there exists an almost disjoint family of cardinality $2^{\aleph _2}$ in ${\mathcal {P}}({\omega _2})$ that is definable by a $\Sigma _1$ -formula and the parameter $\omega _2$ .

Proof. (i) By [Reference Caicedo and Veličković5, Theorem 2], our assumption implies the existence of a well-ordering of ${\mathrm {H}}(\aleph _2)$ of order-type $\omega _2$ that is definable by a $\Sigma _1$ -formula that only uses the sequence $\vec {C}$ as a parameter. In particular, there exists an injection ${\iota }:{{\mathrm {H}}(\aleph _2)}\longrightarrow {\omega _2}$ that is definable in the structure $\langle {\mathrm {H}}(\aleph _2),\in \rangle $ by a formula with parameter $\vec {C}$ . Since [Reference Lücke34, Lemma 6.4] shows that our assumption implies that the set $\{{\mathrm {H}}(\aleph _2)\}$ is definable by a $\Sigma _1$ -formula with parameter $\omega _2$ , we know that $\iota $ is definable by a $\Sigma _1$ -formula with parameters $\omega _2$ and $\vec {C}$ . Given $x\in {}^{\omega _2}2$ , we now define

$$ \begin{align*} \bar{x} ~ = ~ \{{\iota(x\restriction\gamma)}~\vert~{\gamma<\omega_2}\} ~ \in ~ {\mathcal{P}}({\omega_2}). \end{align*} $$

The above computations then show that the set $A=\{{\bar {x}}~\vert ~{x\in {}^{\omega _2}2}\}$ is definable by a $\Sigma _1$ -formula with parameters $\omega _2$ and $\vec {C}$ , and it is easy to see that A is an almost disjoint family of cardinality $2^{\aleph _2}$ in ${\mathcal {P}}({\omega _2})$ .

(ii) Assume that $\omega _1=\omega _1^{\mathrm {L}}$ and ${\mathrm {BPFA}}$ holds. Let $\vec {C}$ denote the $<_{\mathrm {L}}$ -least C-sequence in ${\mathrm {L}}$ . Then $\vec {C}$ is a C-sequence and the set $\{\vec {C}\}$ is definable by a $\Sigma _1$ -formula with parameter $\omega _2$ . Using (i), we can conclude that there exists an almost disjoint family of cardinality $2^{\aleph _2}$ in ${\mathcal {P}}({\omega _2})$ that is definable by a $\Sigma _1$ -formula with parameter $\omega _2$ .

(iii) By [Reference Asperó1, Theorem 5.2], it is possible to start in a model containing a supercompact cardinal and force the validity of ${\mathrm {PFA}}$ together with the existence of a well-ordering of ${\mathrm {H}}(\aleph _2)$ of order-type $\omega _2$ that is definable in $\langle {\mathrm {H}}(\aleph _2),\in \rangle $ by a formula without parameters. We can now proceed as in (i) to obtain an almost disjoint family of cardinality $2^{\aleph _2}$ in ${\mathcal {P}}({\omega _2})$ that is definable by a $\Sigma _1$ -formula with parameter $\omega _2$ in this generic extension.

Acknowledgements

The authors would like to thank William Chan for the permission to include his proof of Theorem 9.1 in this paper. In addition, the authors are thankful to the anonymous referees for the careful reading of the manuscript and several helpful comments.

Funding statement

This project has received funding from the European Union’s Horizon 2020 research and innovation programme under the Marie Skłodowska-Curie grant agreement No 842082 of the first author (Project SAIFIA: Strong Axioms of Infinity – Frameworks, Interactions and Applications). In addition, the research of the first author was supported by the Spanish Government under grant EUR2022-134032. The second author gratefully acknowledges funding from L’ORÉAL Austria, in collaboration with the Austrian UNESCO Commission and in cooperation with the Austrian Academy of Sciences - Fellowship Determinacy and Large Cardinals. Furthermore, the second author was supported by FWF Elise Richter grant number V844, START grant number Y1498, and international project number I6087. The author has applied a CC BY public copyright licence to any Author Accepted Manuscript version arising from this submission.

Competing interests

The authors have no competing interest to declare.

Footnotes

1 See [Reference Kanamori28, p. 5] for the definition of the Levy hierarchy of formulas.

2 Note that this directly implies that $M^{I_s}_\alpha =M^{I_t}_\alpha $ and $i^{I_s}_{\alpha ,\beta }=i^{I_t}_{\alpha ,\beta }$ holds for all $\alpha \leq \beta <{\mathrm {lh}}({I_s}){}$ . Moreover, if $1<{\mathrm {lh}}({I_s}){}<{\mathrm {lh}}({I_t}){}$ , then (v) implies that $M^{I_s}_\infty =M^{I_t}_{{\mathrm {lh}}({I_s}){}}$ and $i^{I_s}_{0,\infty }=i^{I_t}_{0,{\mathrm {lh}}({I_s}){}}$ . Finally, if ${\mathrm {lh}}({I_s}){}=1<{\mathrm {lh}}({I_t}){}$ , then $M^{I_s}_\infty =M^{I_t}_0$ and $i^{I_s}_{0,\infty }={\mathrm {id}}_{M^{I_t}_0}$ .

3 Note that ${\mathbb {P}}$ is allowed be the trivial partial order.

4 That is, there is a formula $\varphi (v)$ in the language of set theory with the property that ${\mathrm {ZFC}}$ proves the listed statements about the class $M=\{{x}~\vert ~{\varphi (x)}\}$ .

5 Here, we let ${{\prec }{\cdot },{\cdot }{\succ }}:{{\mathrm {On}}\times {\mathrm {On}}}\longrightarrow {{\mathrm {On}}}$ denote the Gödel pairing function.

6 The construction of simply definable long well-orderings in the power sets of uncountable cardinals was the original motivation for the work presented in [Reference Lücke and Müller36]. In combination with ideas contained in the proof of Lemma 7.5, the results of [Reference Lücke and Müller36] can be used to show that, if $0^\dagger $ does not exist and the cardinal $\kappa $ is either singular or weakly compact, then there exists a well-ordering of a subset of ${\mathcal {P}}({\kappa })$ of order-type ${\kappa ^+}\cdot \kappa $ that is definable by a $\Sigma _1$ -formula with parameter $\kappa $ .

7 Note that the assumption that the weak compactness of a cardinal $\kappa $ is preserved by forcing with partial orders of the form ${\mathrm {Col}}({\kappa },{{<}\theta })$ has high consistency strength (see, for example, [Reference Hayut and Müller21] and [Reference Jensen, Schimmerling, Schindler and Steel27]).

8 Note that the results of [Reference Goldstern and Shelah18] show that, if there exists a $\Sigma _1$ -reflecting cardinal in ${\mathrm {L}}$ , then these assumptions hold in a generic extension of ${\mathrm {L}}$ .

References

Asperó, D., ‘Guessing and non-guessing of canonical functions’, Annals of Pure and Applied Logic 146(2–3) (2007), 150179.10.1016/j.apal.2007.02.002CrossRefGoogle Scholar
Asperó, D. and Schindler, R., ‘Martin’s maximum ${}^{++}$ implies Woodin’s axiom $\left(\ast \right)$ ’, Ann. of Math. (2) 193(3) (2021), 793835.CrossRefGoogle Scholar
Bakke Haga, K., Schrittesser, D. and Törnquist, A., ‘Maximal almost disjoint families, determinacy, and forcing’, J. Math. Log. 22(1) (2022), Paper No. 2150026, 42.10.1142/S0219061321500264CrossRefGoogle Scholar
Caicedo, A. E. and Ketchersid, R., ‘A trichotomy theorem in natural models of ${\mathrm{AD}}^{+}$ ’, in Set Theory and Its Applications, Contemp. Math., vol. 533 (Amer. Math. Soc., Providence, RI, 2011), 227258.Google Scholar
Caicedo, A. E. and Veličković, B., ‘The bounded proper forcing axiom and well orderings of the reals’, Mathematical Research Letters 13(2–3) (2006), 393408.10.4310/MRL.2006.v13.n3.a5CrossRefGoogle Scholar
Chan, W. and Jackson, S., ‘L(R) with determinacy satisfies the Suslin hypothesis’, Advances in Mathematics 346 (2019), 305328.CrossRefGoogle Scholar
Chan, W. and Jackson, S., ‘Definable combinatorics at the first uncountable cardinal’, Transactions of the American Mathematical Society 374(3) (2021), 20352056.CrossRefGoogle Scholar
Chan, W., Jackson, S., and Trang, N., ‘Almost disjoint families under determinacy’, Preprint, 2022.Google Scholar
Chan, W., Jackson, S., and Trang, N., ‘The size of the class of countable sequences of ordinals’, Trans. Amer. Math. Soc. 375(3) (2022), 17251743.CrossRefGoogle Scholar
Cox, S., ‘Covering theorems for the core model, and an application to stationary set reflection’, Ann. Pure Appl. Logic 161(1) (2009), 6693.CrossRefGoogle Scholar
Dehornoy, P., ‘Iterated ultrapowers and Prikry forcing’, Ann. Math. Logic 15(2) (1978), 109160.10.1016/0003-4843(78)90018-9CrossRefGoogle Scholar
Fischer, V., Schrittesser, D. and Weinert, T., ‘Definable mad families and forcing axioms’, Annals of Pure and Applied Logic 172(5) (2021), 102909. 50 Years of Set theory in Toronto.10.1016/j.apal.2020.102909CrossRefGoogle Scholar
Friedman, S.-D., Holy, P. and Lücke, P., ‘Large cardinals and lightface definable well-orders, without the GCH’, J. Symb. Log. 80(1) (2015), 251284.10.1017/jsl.2013.41CrossRefGoogle Scholar
Fuchs, G., ‘A characterization of generalized Příkrý sequences’, Arch. Math. Logic 44(8) (2005), 935971, 2005.10.1007/s00153-005-0313-zCrossRefGoogle Scholar
Gitik, M., ‘Prikry-type forcings’, in Handbook of Set Theory . Vols. 1, 2, 3 (Springer, Dordrecht, 2010), 13511447.Google Scholar
Gitman, V., ‘Ramsey-like cardinals’, J. Symbolic Logic 76(2) (2011), 519540.10.2178/jsl/1305810762CrossRefGoogle Scholar
Gitman, V. and Welch, P. D., ‘Ramsey-like cardinals II’, J. Symbolic Logic 76(2) (2011), 541560.10.2178/jsl/1305810763CrossRefGoogle Scholar
Goldstern, M. and Shelah, S., ‘The bounded proper forcing axiom’, J. Symbolic Logic 60(1) (1995), 5873.CrossRefGoogle Scholar
Grigorieff, S., ‘Intermediate submodels and generic extensions in set theory’, Annals of Mathematics (2) 101 (1975), 447–490.CrossRefGoogle Scholar
Hauser, K., ‘Indescribable cardinals and elementary embeddings’, J. Symbolic Logic 56(2) (1991), 439457, 1991.CrossRefGoogle Scholar
Hayut, Y. and Müller, S., ‘Perfect subtree property for weakly compact cardinals’, Israel J. Math. 253(2) (2023), 865886.CrossRefGoogle Scholar
Holy, P. and Lücke, P., ‘Locally ${\varSigma}_1$ -definable well-orders of $\mathrm{H}\left({\kappa}^{+}\right)$ ’, Fund. Math. 226(3) (2014), 221236.10.4064/fm226-3-2CrossRefGoogle Scholar
Horowitz, H. and Shelah, S., ‘On the non-existence of mad families’, Arch. Math. Logic 58(3–4) (2019), 325338.CrossRefGoogle Scholar
Jech, T. J., ‘Trees’, J. Symbolic Logic 36 (1971), 114.CrossRefGoogle Scholar
Jech, T. J., Set Theory, Springer Monographs in Mathematics (Springer-Verlag, Berlin, 2003). The third millennium edition, revised and expanded.Google Scholar
Jensen, R. B. and Steel, J. R., ‘K without the measurable’, J. Symb. Log. 78(3) (2013), 708734.CrossRefGoogle Scholar
Jensen, R., Schimmerling, E., Schindler, R. and Steel, J., ‘Stacking mice’, J. Symbolic Logic 74(1) (2009), 315335.CrossRefGoogle Scholar
Kanamori, A., The Higher Infinite, second edn., Springer Monographs in Mathematics (Springer-Verlag, Berlin, 2003).Google Scholar
Kechris, A. S., ‘ ${\mathrm{AD}}$ and projective ordinals’, in Cabal Seminar 76–77 (Proc. Caltech-UCLA Logic Sem., 1976–77), Lecture Notes in Math., vol. 689 (Springer, Berlin, 1978), 91132.Google Scholar
Kechris, A. S., Classical Descriptive Set Theory, Graduate Texts in Mathematics, vol. 156 (Springer-Verlag, New York, 1995).CrossRefGoogle Scholar
Koepke, P., ‘Some applications of short core models’, Ann. Pure Appl. Logic 37(2) (1988), 179204.CrossRefGoogle Scholar
Larson, P. B., ‘Forcing over models of determinacy’, in Handbook of Set Theory. Vols. 1, 2, 3 (Springer, Dordrecht, 2010), 21212177.CrossRefGoogle Scholar
Lücke, P., ‘ ${\varSigma}_1^1$ -definability at uncountable regular cardinals’, J. Symbolic Logic 77(3) (2012), 10111046.CrossRefGoogle Scholar
Lücke, P., ‘Partition properties for simply definable colourings’, Israel Journal of Mathematics 236(2) (2020) 841898.CrossRefGoogle Scholar
Lücke, P., Ros, L. M. and Schlicht, P., ‘The Hurewicz dichotomy for generalized Baire spaces’, Israel J. Math. 216(2) (2016), 9731022.CrossRefGoogle Scholar
Lücke, P. and Müller, S., ‘Closure properties of measurable ultrapowers’, J. Symb. Log. 86(2) (2021), 762784.CrossRefGoogle Scholar
Lücke, P., Schindler, R., and Schlicht, P., ‘ ${\varSigma}_1\left(\kappa \right)$ -definable subsets of $\mathsf{H}\left({\kappa}^{+}\right)$ ’, The Journal of Symbolic Logic 82(3) (2017), 11061131.10.1017/jsl.2017.36CrossRefGoogle Scholar
Lücke, P. and Schlicht, P., ‘Measurable cardinals and good ${\varSigma}_1\left(\kappa \right)$ -wellorderings’, Mathematical Logic Quarterly 64(3) (2018), 207217.CrossRefGoogle Scholar
Mathias, A. R. D., ‘Happy families’, Ann. Math. Logic 12(1) (1977), 59111.CrossRefGoogle Scholar
Miller, A. W., ‘Infinite combinatorics and definability’, Ann. Pure Appl. Logic 41(2) (1989), 179203.CrossRefGoogle Scholar
Mitchell, W. J., ‘Sets constructible from sequences of ultrafilters’, J. Symbolic Logic 39 (1974), 5766.CrossRefGoogle Scholar
Neeman, I. and Norwood, Z., ‘Happy and mad families in $L\left(\mathbb{R}\right)$ ’, J. Symb. Log. 83(2) (2018), 572597.CrossRefGoogle Scholar
Schimmerling, E., ‘A core model toolbox and guide’, in Handbook of Set Theory. Vols. 1, 2, 3 (Springer, Dordrecht, 2010), 16851751.10.1007/978-1-4020-5764-9_21CrossRefGoogle Scholar
Schlicht, P., ‘Perfect subsets of generalized Baire spaces and long games’, J. Symb. Log. 82(4) (2017), 13171355.CrossRefGoogle Scholar
Schrittesser, D. and Törnquist, A., ‘The Ramsey property implies no mad families’, Proc. Natl. Acad. Sci. USA 116(38) (2019), 1888318887.CrossRefGoogle ScholarPubMed
Sharpe, I. and Welch, P. D., ‘Greatly Erdős cardinals with some generalizations to the Chang and Ramsey properties’, Ann. Pure Appl. Logic 162(11) (2011), 863902.CrossRefGoogle Scholar
Steel, J. R., The Core Model Iterability Problem, Lecture Notes in Logic, vol. 8 (Springer-Verlag, Berlin, New York, 1996).CrossRefGoogle Scholar
Steel, J. R., ‘An outline of inner model theory’, in Handbook of set theory. Vols. 1, 2, 3 (Springer, Dordrecht, 2010), 15951684.CrossRefGoogle Scholar
Steel, J. R., ‘An introduction to iterated ultrapowers’, in Forcing, Iterated Ultrapowers, and Turing Degrees, Lect. Notes Ser. Inst. Math. Sci. Natl. Univ. Singap., vol. 29 (World Sci. Publ., Hackensack, NJ, 2016), 123174.Google Scholar
Törnquist, A.. ‘ ${\varSigma}_2^1$ and ${\varPi}_1^1$ mad families’, J. Symbolic Logic, 78(4):11811182, 2013.CrossRefGoogle Scholar
Welch, P. D., ‘Stably measurable cardinals’, J. Symb. Log. 86(2) (2021), 448470.CrossRefGoogle Scholar
Woodin, W. H., The Axiom of Determinacy, Forcing Axioms, and the Nonstationary Ideal, De Gruyter Series in Logic and Its Applications, vol. 1 (Walter de Gruyter & Co., Berlin, 1999).Google Scholar
Zeman, M., ‘Inner Models and Large Cardinals, De Gruyter Series in Logic and Its Applications, vol. 5 (Walter de Gruyter & Co., Berlin, 2002).CrossRefGoogle Scholar